Está en la página 1de 154

Los problemas de la antigedad clsica La duplicacin del cubo Consiste en hallar el lado de un cubo que tenga volumen doble

que otro cubo de lado dado. Es decir, dado un cubo de arista a y volumen V, hallar la arista de un cubo de volumen 2V. La tradicin cuenta que una epidemia de peste arras Atenas hacia el 428 a.C.. Para acabar con ella se envi un mensajero al orculo de Apolo en Delos. El orculo indic que para acabar con la epidemia Los problemas de sera necesario duplicar el volumen del altar cbico de Apolo. Los la antigedad atenienses duplicaron las dimensiones del altar, pero evidentemente clsica fueron: no se duplic el volumen. Ni que decir tiene que la epidemia no La duplicacin del desapareci! cubo. La cuadratura del Hipcrates de Chios demostr que el problema puede reducirse a encontrar dos medias proporcionales entre la arista dada y el doble crculo. La de la misma. triseccin del ngulo. cada uno de ellos resueltos Con la notacin actual resulta solamente con y a partir de ellas regla sin marcas (no graduada) y x2 = a y ; y2 = 2a x comps. Ninguno Si consideramos a = unidad podemos obtener el valor de la arista del de ellos tiene cubo buscado mediante la interseccin de las parbolas x2 = y e y2 = solucin con 2 x tal como aparece en la grfica dichas condiciones.

Tambin Menecmo en el siglo IV a.C ya haba hallado la solucin mediante la interseccin (con el lenguaje actual) de una parbola y una hiprbola (y = 2/x) al considerar los miembros primero y tercero de la proporcin a : x = y : 2a y primero y segundo a : x = x : y que para a = unidad son la parbola e hiprbola de la grfica

La cuadratura El problema construir un tenga igual rea dado. Lindeman demostr que construccin marcas y es un nmero Veamos cmo

del crculo consiste en cuadrado que que el crculo Ferdinand (1852-1939) era imposible tal con regla sin comps pues trascendente. podemos "cuadrar" un tringulo ACB. Dicha rea es S T = 1/2 AC h Sobre la prolongacin de h llevamos la mitad del lado AC y contruimos la circunferencia cuyo dimetro es BB'. Consideramos el tringulo rectgulo QB"B y aplicando el teorema de la altura resulta

Por tanto x es el lado del cuadrado equivalente al tringulo ACB Hipcrates logr cuadrar algunas lnulas y este texto es el nico fragmento de las matemticas griegas que ha llegado a nosotros en el original. Sobre el tringulo rectgulo issceles ABC prob que el rea de la lnula ALBL' es igual que el rea del trigulo T. Para ello hace uso del hecho de que dos crculos son entre s como los cuadrados construidos sobre sus dimetros. Veremos que no podemos cuadrar el crculo mediante una construccin geomtrica En la circunferencia de radio r consideramos una cuerda AB = r Trazamos el radio que pase por el punto medio de dicha cuerda que determina en la circunferencia el punto T y por l la recta tangente t. A partir de A' llevamos tres veces sobre t el radio r y obtenemos el punto L. Finalmente unimos L con P.

Como el trigulo PTL es rectngulo resulta PL 2 = PT 2 + TL 2 PT = 2 r PT 2 = 4 r 2

Si hacemos resulta TL = m r por tanto 2 2 PL = 4 r + m 2 r 2 = r 2 (4 + m 2) = 9,869231718 r 2 PL = 3,141533339 r = r ' La diferencia entre y ' es d = - ' = 0,00005931. Si deseamos "cuadrar" el crculo cometeremos, por ejemplo, un error menor que 0,00001. Para ello basta con que llevemos a continuacin de PL el radio del crculo y aplicar el teorema de la altura a la altura relativa a la hipotenusa

De esta forma x es el lado del cuadrado que "cuadra" el crculo (con un error menor que 0,0001 -por ejemplo-)

La triseccin del ngulo La trisecin del ngulo fu el tercero de los problemas clsicos de la antigedad griega. Se pretenda, slo con regla (no graduada) y comps, trisecar un ngulo. Esto no es posible y los matemticos introdujeron curvas auxiliares que le sirvieran de ayuda (por ejemplo la trisectriz de Hipias). El matemtico fracs Pierre Wantzel (1814-1848) prob que un ngulo es trisecable con regla y comps si el polinomio 4x3 - 3x - cos() es reductible. He aqu un mtodo para obtener la tercera parte de cualquier ngulo debido a Arqumedes. Sea AOB = x el ngulo dado. Trazamos por el vrtice O una circunferencia de radio r, prolongamos el otro lado y por B una secante a la circunferencia tal que MP = r (es necesario una regla graduada, para transportar dicha medida). Dicha secante determina un punto M sobre la circunferencia. Entonces el ngulo NOM = z es la tercera parte del dado. En efecto, el ng(OMB) = 2z (es el ngulo exterior del tringulo OPM que es issceles). Por otra parte ng(BOM) = 180 - 4z (el tringulo OMB tambin es issceles) y resulta x + ng(OMB) + z = 180 de donde x + 180 - 4z + z = 180 es decir x = 3z El sofista Hipia de Elis intentando trisecar el ngulo descubri una nueva curva que,

desafortunadamente, no es construcitible con regla y comps. La denomin trisectriz Se genera de la siguiente forma

Suponemos el segmento AB que gira en sentido de las agujas del reloj (con velocidad constante) hasta ocupar la posicin AM. A la vez, se desplaza hacia abajo, tambin con velocidad constante, el segmento BB' y ocupa en el mismo tiempo la posicin AM. Un punto de la trisectriz viene dado por la interseccin en cada instante de dichos segmentos. T es un punto de la trisectriz (que est dibujada en color azul en la figura) Podemos dividir un ngulo agudo cualquiera (ayudndonos con la trisectriz) de la siguiente forma Consideremos el ngulo BOA = Dividimos en tres partes iguales AH = A'B' y tracemos paralelas a OB' que corten a la trisectriz en P2 y P1. Por ltimo unamos O con dichos puntos y obtenemos los tres ngulos /3 En efecto, teniendo en cuenta la construccin de la trisectriz resulta

Como B"B' = AH/3

Por otra parte

y de ambas

Federico Villarreal 1850-1923 Julio A. Miranda Ubaldo (Per) Hace mucho tiempo leyendo un texto de historia del Per de Gustavo Pons Muzzo en el capitulo XIV: "Las Ciencias en la Epoca Republicana" encontr un personaje llamado Federico Villarreal V. (1850-1923) y segn su biografa se trat de un reconocido matemtico de finales del

siglo XIX y comienzos del siglo XX que descubri en 1873 un novedoso mtodo para "elevar un polinomio cualquiera a una potencia cualquiera" en ese instante me entr una curiosidad por saber en que consista ese mtodo. Pasaron los aos y en el mes de enero de este ao (2002) me enter por medio de una conversacin informal con mi amigo el futuro ilustre matemtico huaralino Edward W. Morales que l haba encontrado en al biblioteca de la Universidad Nacional Mayor de San Marcos el trabajo original de Villarreal relativo a dicho mtodo y fue l mismo quin me proporcion esa valiosa informacin que le he dado forma y lo plasmo en este trabajo, ahora bien espero sinceramente, que el contenido de este trabajo sea de tu completo agrado. Julio A. Miranda Ubaldo

Federico Villarreal naci el 3 de agosto de 1850 en Tcume, departamento de Lambayeque (Per) (El departamento de Lambayeque tiene como capital departamental a la ciudad de Lambayeque) A los 14 aos fue cajero en una empresa despepitadora de algodn, pero no dej de lado sus estudios que lo llevaran hacer profesor y as fue: a los 20 aos obtuvo el ttulo de preceptor otorgado por la comisin departamental de Instruccin pblica de Trujillo el cual le permiti dirigir la escuela oficial de Tcume de 1870 a 1874 y entre 1875 y 1876 dirigi un colegio de instruccin media en la ciudad de Lambayeque, ense all matemticas y ocup en l el cargo de vicerrector. Entre 1876 y 1877 tuvo bajo su cargo una escuela primaria en Lambayeque. La experiencia de Villarreal como maestro elemental seal slo una primera etapa. Su vocacin de matemtico bulla desbordando su enseanza humilde. Ya en 1873 cuando contaba con tan slo 23 aos descubri un mtodo para elevar un polinomio cualquiera a una potencia cualquiera. Entre 1877 y 1880 estudi en la seccin de ciencias matemticas de la Facultad de Ciencias de la Universidad Nacional Mayor de San Marcos (UNMSM) gradundose como bachiller en 1879 con la tesis: "Frmulas y mtodos que deben completarse en matemticas puras" y como licenciado con la tesis: "Efectos de la Refraccin sobre el Disco de los Astros" (1880). En 1881 se gradu de doctor en ciencias matemticas mediante la tesis: "Clasificacin de Curvas de Tercer Grado" destacando por su originalidad y conclusiones lo cual le mereci a Villarreal la medalla de oro, otorgada por la Facultad de Ciencias al primer doctor de su poca, quien a la vez, se constituye en el primer matemtico profesional del siglo XX en el Per. Su labor docente universitaria la inicia como profesor adjunto en la Facultad de Ciencias de la UNMSM en 1880, donde dict su primer curso: Astronoma; luego en esa misma casa de estudio se encarga de los cursos: Revisin de Matemticas, Mecnica Racional, Geodesia y Teora General de Motores y Mquinas.

Biografa

Por su gran prestigio y sus dotes profesionales e intelectuales, llegara a ser decano de la Facultad de Ciencias en dos oportunidades: de 1903 a 1917 y luego de 1919 a 1923. Sigui estudios en la Escuela nacional de Ingenieros desde 1882 hasta graduarse de ingeniero civil y de minas en 1886. En este centro docente ense los cursos de fsica, clculo infinitesimal, teora de caminos, puentes y ferrocarriles, Topografa y luego los cursos de Resistencia de Materiales e Hidrulica. Tambin fue profesor en la Escuela Militar de Chorrillos (1890) en donde ense los cursos de: Cosmografa, Trigonometra Esfrica, Construccin de Cartas Topogrficas y Clculo de Probabilidades. Fund la Revista de Ciencias en 1897. F. Villarreal particip activamente formando parte del contingente sanmarquino en la Guerra con Chile especficamente en la Resistencia de Chorrillos y en la Batalla de Miraflores (enero de 1881) donde fue distinguido con el grado de subteniente- En 1893 se enrola en la Guardia Nacional y en 1884 fue nombrado primer jefe del batalln "Defensores de la Patria". Tambin incursion en la poltica. En1871 fue presidente de la Junta Directiva del Partido Civil en el distrito de Muchumi (Lambayeque). Posteriormente, en el ao 1892 fue elegido senador suplente de su departamento. Ms tarde, es elegido nuevamente senador por Lambayeque, actuando en las legislaturas de 1913 y 1914 en donde alcanzan mucha significacin sus discursos sobre la "Ley de Enfiteusis" y sobre la "Ley Relativa a los Bancos Hipotecarios". Fue uno de los iniciadores de la ley que estableci el examen de ingreso a la universidad. Villarreal tambin posea una notable cultura filosfica de manifiesta preferencia por la corrientes mecanicistas propias de aquella poca y sostenidas entre otros por Wronski, corrientes que parecan tener la posibilidad de lograr una sntesis entre la filosofa y la Mecnica Celeste como sistema de descripcin causalista del equilibrio universal cualesquiera que fuera le estructura y consistencia del Universo. Sobre el lado humano de Vilarreal, Basadre dice al respecto: "Villarreal no fue un sabio pacfico e inofensivo. Muchas veces refut a presuntos expertos e inventores y polemiz con ellos implacablemente sin desdear su poca jerarqua intelectual. Tuvo tambin veleidades lingsticas. A pesar de su genio, Villarreal no tuvo brillo como profesor. En sus lecciones, su gran dificultad de expresin levant un muro ante sus alumnos, dando lugar, de un lado a monlogos acompaados por complicados clculos en la pizarra y, de otro a escenas cmicas o grotescas. Hombre apasionado como decano en la Facultad de Ciencias de la UNMSM ejerci una verdadera dictadura. A pesar de humanas debilidades y de deficiencias ahondadas por la falta de una educacin adecuada o por las limitaciones del ambiente, Villarreal es todo un personaje en la historia del Per" El Dr. Federico Villarreal fallece en Barranco (Lima) el 3 de enero de 1923.

Trabajos del Dr. Villarreal

Federico Villarreal dej un aproximado de 538 trabajos en diversos campos de la ciencia y tecnologa fundamentalmente en matemticas, ingeniera, fsica, pedagoga, geografa, historia y lingstica.

En matemticas sus principales trabajos fueron: 1. "Elevacin de polinomios a una potencia cualquiera" (1879) 2. "Clasificacin de las curvas de tercer grado" (tesis doctoral de 1881) En este trabajo Villarreal logra obtener y clasificar matemticamente 80 curvas de tercer grado 3. "Aportes a la teora de los nmeros" (1897) La teora de los nmeros atrajo siempre la atencin de Villarreal tal es as que le dedic numerosos artculos. Entre ellos se destacan dos teoremas referentes a criterios de divisibilidad que el descubri: La diferencia de dos nmeros que son representados por las mismas cifras en dos sistemas de numeracin de bases diferentes es divisible por la diferencia de las bases Un nmero es divisible por un cierto divisor si lo es la suma de sus cifras cuando se le escribe en el sistema de numeracin cuya base es el divisor aumentado en la unidad; o bien si los es la suma de sus cifras de lugar par menos la suma de las de lugar impar cuando se le escribe en el sistema de numeracin cuya base es el divisor disminuido en al unidad 4. "Geometra no Euclideana" (1898) Este trabajo fue presentado por Villarreal en el Primer Congreso Cientfico Latinoamericano realizado en Buenos Aires (Argentina) en 1898. Aqu describe los fundamentos de las geometras de Lobatschewsky y Riemann. 5. "Poliedros Regulares y semiregulares" (1906-1907) Esta obra contiene una exposicin histrica y el clculo de volmenes de los poliedros regulares y semiregulares empleando los principios de la trigonometra esfrica. 6. "Integracin por Traspasos" (1920) Trabajo que apareci por primera vez como parte de su tesis de bachiller en 1879 en que valindose del mtodo de integracin por partes obtiene una frmula que generaliza la llamada frmula de integracin de Bernouilli. 7. "Resolucin general de las ecuaciones de quinto grado" Estudio crtico de un mtodo propuesto por Wronski en 1827 para la resolucin de las ecuaciones de quinto grado , traducido, analizado y corregido por Villarreal. ste llega a la conclusin que Wronski hace en este trabajo el empleo de una funcin que llama "funcin Shin" que corresponde a los actuales determinantes, explica los errores de Wronski y concluye con la imposibilidad de la solucin algebraica de las citadas ecuaciones En Ingeniera: 1. "Tratado de resistencia de Materiales" (1911) En este importante trabajo de Villarreal estan insertos dos trabajos originales: "Clculos de los momentos de flexin en una viga empotrada en sus dos extremos" En este trabajo Villarreal analiza los problemas de las vigas empotradas ya sea en ambos lados o empotradas en un extremo y libre en el otro descubriendo los llamados "momentos de empotramiento" que hasta esa poca no se haba podido calcular. "Deformacin de las vigas que trabajan a la flexin" Aqu el problema de la flexin de una viga Villarreal lo reduce a una ecuacin

diferencial de cuarto grado y sus integrales sucesivas dan: la primera, el esfuerzo cortante; la segunda, los momentos de flexin; la tercera, la deflexin de una viga; y la cuarta y ltima, la ecuacin del eje deformado. 2. "Teora de Mquinas y Motores" (No se conoce ao de publicacin) En la que hace una exposicin sistemtica y rigurosa de todas las condiciones referentes al equilibrio de las mquinas. En Fsica: 1. "Principio de la Relatividad" (1909) Raro trabajo de Villarreal en la que logra interpretar el principio de relatividad restringida formulado por Einstein en 1905 y expone un desarrollo metdico de las modificaciones que debido a este principio experimentan las leyes clsicas de la mecnica. Es de advertir que en aquella poca no fue tarea fcil la interpretacin inmediata del principio de la relatividad para muchos hombres de ciencia, debido en gran parte a que la mentalidad clsica se mostr hermtica ante la consideracin de las condiciones epistemolgicas en le tcnica de la observacin de los fenmenos. 2. "Descarga Oscilante en un Condensador" (1916) Interesante trabajo de electrodinmica en el que resuelve el problema terico de la descarga Disruptiva obteniendo la frmula de Thompson para el periodo de las oscilaciones. 3. "Dinmica Analtica" (1917) En esta obra est incluida el importante trabajo sobre " Choques de un nmero cualquiera de Cuerpos". 4. "Trabajo mecnico del Hombre" (No se conoce ao de publicacin) En l se refiere a cuestiones realmente curiosas y algunas de ellas muy tiles , tales como: el equilibrio del hombre, la marcha de un hombre con carga, la fatiga mnima del cargador, la condicin para que se haga el mximo camino antes del cansancio, etc todo en base a datos experimentales y resultados matemticos. En Pedagoga: 1. "Memorias Pedaggicas" (No se conoce ao de publicacin) 2. "Recreaciones matemticas" (No se conoce ao de publicacin) En Geografa: 1. "Mtodo para determinar la latitud y longitud de los lugares del Per" (No se conoce ao de publicacin) Este trabajo lo inicia con una introduccin sobre la metodologa a seguir para la medicin De coordenadas geogrficas de un lugar y expone a continuacin una tcnica simple para la determinacin de meridianos, la hora solar, las latitudes y longitudes geogrficas. Contiene una tabla de latitudes y longitudes de 700 lugares del Per. 2. "Trazo del meridiano por la Cruz del Sur" (No se conoce ao de publicacin) 3. "Coordenadas geogrficas del Departamento de Lambayeque y Cuzco" (1905) 4. "Extensin Superficial del Per" (No se conoce ao de publicacin) En historia:

1. "Historia de las matemticas en el Per" (No se conoce ao de publicacin) Este trabajo comprende una introduccin y estudios sobre la numeracin, la geometra, la mecnica, la astronoma y la hidrulica en el Imperio de los Incas ; sigue con un estudio sobre la enseanza acadmica de las matemticas en el virreinato y finalmente se ocupa de las matemticas en la Repblica. 2. "Los cometas en los tiempos de Huayna Capac" (1894) Utilizando como fuente principal al cronista Inca Garcilazo de la Vega, Villarreal realiza una confrontacin entre las observaciones realizadas por la ciencia occidental desde la aparicin de los primeros instrumentos de observacin y los datos proporcionados por Garcilazo, llegando a identificar los cometas descritos en las crnicas de la conquista 3. "El Archivo de Raymondi" (No se conoce ao de publicacin) 4. "Orgenes del Sistema mtrico"(No se conoce ao de publicacin) En lingstica: 1. " Manual y Diccionario de Esperanto" (1900) Idioma nuevo y universal, el esperanto al que Villarreal le prodig lastimosamente tiempo, dinero y energa, y a dirigir y redactar como colaborador nico la revista "Antuanen esperantistoj!" (Adelante Esperantistas) que fundara en 1903. 2. "La Lengua Yunga" (1921) Villarreal public una gramtica y un vocabulario de la lengua mochica o Yunga. Esta lengua se hablaba en los departamentos de la costa norte del Per En la actualidad esta lengua esta completamente extinguida.

Problemas de Villarreal

Formulado por Villarreal en 1906 y denominado por l como: "El Problema del Nio" y dice as: "Un mvil se desplaza en lnea recta con una velocidad constante y otro mvil se mueve tambin a velocidad constante, de modo que la tangente a su trayectoria pasa constantemente por el primer mvil. Hallar la ecuacin de la curva descrita por el segundo mvil". En una nota de 1908 Villarreal plantea y resuelve los dos problemas siguientes: "Hallar dos nmeros terminados en la misma cifra y tales que las dos ltimas cifras de su producto constituyan el cuadrado de la cifra en que terminan los dos nmeros dados". "Hallar tres nmeros terminados en la misma cifra cuyo producto termina en tres cifras que constituyan el cubo de la cifra en que terminan los nmeros dados". Puede Ud. resolverlos?.
Una ancdota en la vida del Dr. Villarreal

Esta es una de las muchas ancdotas de Villarreal que a continuacin les relato: "En la Maison de Sant (que es un hospital) falleci en diciembre de 1909, a la edad de 86 aos, Jos Sebastian Barranca, antiguo catedrtico de Botnica en la Facultad de Ciencias de la UNMSM, fillogo

naturalista, Sebastian Barranca vivi para sus estudios e investigacin. Su sepelio fue modestsimo. El estado y la universidad estuvieron en l ausentes. Los colegas que acudieron no pasaron de media docena. No estuvo representada la juventud estudiantil. El mayor porcentaje de oyentes que tuvo Villarreal cuando pronunci su discurso fnebre fue el de unos 40 negritos de humilde condicin que ni conocan al muerto pues ellos haban asistido a otro entierro. Segn se dijo,la Beneficencia neg un nicho perpetuo al sabio Barranca por no haber pagado el precio respectivo" Se imaginan al ilustre Dr. F Villarreal pronunciando un discurso fnebre a personas que en su mayora eran negritos y donde casi ningn catedrtico y alumno asistieron y para colmo los negritos eran de otro entierro.!!!!!!!. Y surge una pregunta en mi mente: porqu no asistieron? acaso pocos fueron avisados de la muerte del sabio? o quiz fue un psimo profesor y nadie quiso asistir a su sepelio?........ahhhh cosas de la Vida.
Comentario acerca de la vida del Dr. Villarreal

Villarreal fue un personaje multifactico y dinmico le entr a casi todo desde modesto profesor de primaria y secundaria, a profesor universitario, matemtico, ingeniero, soldado, poltico y hasta lingista, que tipo! muy pocas veces se encuentra en la historia de un pas latino un personaje como ste. Es notable que encontrndose lejos de la influencia de los grandes matemticos de la poca, Villarreal haya podido arribar a importantes estudios y descubrimientos como los que efectu, lo que resalta su talento. Su dominio en el campo de la ciencia es bastante amplio pues ense varios cursos, algunos sin relacin directa como: astronoma, mecnica racional, hidrulica, teora de probabilidades, topografa, clculo infinitesimal, fsica, etc.. Siendo un sencillo profesor de secundaria, con slo 23 aos y sin haber estudiado en una universidad, Villarreal descubre el mtodo para elevar un polinomio cualquiera a una potencia cualquiera, asombroso verdad?. Sin embargo lo mas interesante de su vida cientfica es el hecho de que efectu contribuciones originales al desarrollo de las matemticas e ingeniera,algo pocas veces visto en los matemticos de habla espaola. Es por todas estas razones que a Villarreal se le puede decir con toda justicia: "El Newton del Per"

En 1873, encontrndose en su pueblo natal Tcume del departamento de Lambayeque (Per), Federico Villarreal V. (1850-1923) descubre un mtodo para elevar un polinomio cualquiera a una potencia cualquiera. Este Huaral,30 de junio de hecho provoc que otro matemtico peruano 2002 Cristbal de Losada y Puga (1894-1961) Julio A. Miranda Ubaldo estudiase a profundidad este descubrimiento y Profesor de bautizase el desarrollo de la potencia del Matemticas polinomio como el "Polinomio de Villarreal". El jmiub@hotmail.com historiador peruano Jorge Basadre en su

"Historia de la Repblica del Per" (Tomo X, pag.28) dice: "Es tan perfecto que aun para el caso de un binomio resulta fcil y seguro y rpido que el mtodo del binomio de Newton". En su tesis de 1879 para optar el grado de bachiller en ciencias matemticas titulado:"Frmulas y mtodos que deben completarse en matemticas puras" Villarreal inserta su mtodo pasando desapercibido segn l - "por el estado de las matemticas en el Per". Este novedoso mtodo Villareal lo publica por primera vez el 31 de marzo de 1886 en la revista "La Gaceta Cientfica" (2 tomo) pero como siempre sucede en nuestro medio muy pocas personas le dieron la debida importancia a su trabajo. En 1919 Vilarreal nuevamente publica su mtodo esta vez en la "Revista de Ciencias" bajo el ttulo de: "Elevacin de polinomios a una potencia cualquiera" que es justamente el ttulo de este trabajo. Veamos. . .

Tpicos Problemas Sangakus Los problemas sangakus normalmente implican problemas de crculos dentro de crculos tangentes entre s o bien crculos inscritos en otras figuras, como por ejemplo elipses. Tambin hay problemas que tratan de esferas dentro de otras esferas u otras figuras tambin tangentes entre s. Este problema es de un sangaku que data de 1788 y fue hallado en la prefectura de Tokio. Nos pide el radio del n-simo crculo azul en funcin del radio r del crculo verde. Obsrvese que los crculos rojo oscuro son idnticos cada uno de radio r/2. Este problema muestra el equivalente japons de la "frmula de los crculos tangentes de Descartes" Conocido problema que ha sobrevivido desde 1824 en una tablilla de la prefectura de Gumma. Los crculos anaranjado y azul se tocan en un slo punto y son tangentes a una misma recta. El pequeo crculo rojo toca a ambos crculos y es tambin tangente a la misma recta.

Curioso problema que fue escrito en una tablilla aproximadamente en 1913 en la prefectura de Miyagi. Tres cuadrados anaranjados se trazan segn se muestra en el tringulo rectngulo grande de color verde. Cmo se relacionan los radios de los tres crculos azules? Este problema es de un sangaku encontrado en la prefectura de Gumma y que data de 1803. La base de un tringulo issceles descansa sobre el dimetro del crculo grande de color verde. Este dimetro tambin biseca al crculo rojo el cual esta inscrito de modo que toque exactamente el interior del crculo verde y un vrtice del tringulo issceles, como se muestra en la figura anterior. El crculo azul est inscrito de modo que sea tangente al crculo rojo, al tringulo e interiormente al crculo verde. Un segmento une el centro del crculo azul en el punto de interseccin entre el crculo rojo y el tringulo issceles. Demuestre que este segmento es perpendicular al dimetro del crculo verde. Interesante problema que proviene de una tablilla ubicada en la prefectura de Gumma y esta fechada en 1874. Un crculo grande de color azul esta dentro de un cuadrado. Cuatro crculos anaranjados ms pequeos, cada uno de ellos de diferente radio, son tangentes al crculo azul asi como a las caras adyacentes del cuadrado. Cul es la relacin de los radios de los cuatro crculos pequeos y la longitud del lado del cuadrado? Este problema que data de 1822 est inscrito en una tablilla localizada en la prefectura de Kanagawa. Dos esferas rojas son tangentes exteriormente y ambas son tangentes interiormente a la esfera grande de color verde. Un collar de esferas azules de diferentes tamaos rodea el "cuello" entre

las esferas rojas. Cada esfera azul en el "collar" es tangente a sus vecinos prximos, a la vez que son tangentes a las dos esferas rojas y a la esfera verde. Cuntas esferas azules conforman el collar? Cmo los radios de las esferas azules se relacionan entre s? Se anima alguien a resolver estos problemas? Tmese su tiempo y buena suerte Para que no sufras mucho, la solucin de estos seis problemas sangaku lo encontrars en: http://www.pourlascience.com/numeros/pls-249/rothman/rothmanbox1.htm de donde extraje valiossima informacin para la elaboracin de este trabajo. Un detalle... est en francs. Apuntes y notas sobre problemas sangaku Tres cuadrados anaranjados se trazan segn se muestra en el tringulo rectngulo grande de color verde. Cmo se relacionan los radios de los tres crculos azules?

Todos los tringulos rectngulos formados son semejantes. En el trigulo APQ se verifica

En PBS

de donde que corresponde al lado del primer cuadrado que deseamos inscribir, siendo c = AB el cateto sobre el que se apoya el vrtice P (cateto contiguo del ngulo ). Radio del crculo inscrito en el tringulo QRC En el tringulo QMO Como L = QM + r y resulta

Repitiendo este procedimiento se llega a solucionar el problema. Si r 1, r 2, y r 3 son los radios de los crculos, el radio del segundo es media proporcional de los otros dos, es decir r22 = r1 r3 como a continuacin vamos a comprobar para el tringulo rectngulo (3, 4, 5)

Primer cuadrado y primer crculo

Segundo cuadrado y segundo crculo

Por lo tanto

... o bien

Segundo radio

Tercer cuadrado y tercer crculo

Por lo tanto

... o bien

Tercer radio

Resultando, efectivamente, que r 2 2 = r 1 r 3 Frmula de los crculos tangentes de Descartes Julio A. Miranda Ubaldo (Per) En una carta fechada en noviembre de 1643 dirigida a la princesa Elizabet de Bohemia el matemtico francs Ren Descartes (1596-1650) desarroll una frmula que relacionaba la curvatura1 de cuatro crculos cada uno tangente a los otros tres. Descartes defini la curvatura de un crculo como el recproco de su radio. As por ejemplo si el radio de un crculo es 1/5 de otro crculo entonces su curvatura es 5 veces el del crculo grande. Una lnea se considera un crculo de radio infinito y por tanto de CURVATURA CERO.

Pero que dice la frmula? Dado cuatro crculos de curvaturas Ra, Rb, Rc, y Rd, cada uno tangente a los otros tres, entonces se cumple que:

donde por definicin de curvatura Rc, y Rd, los radios de los crculos tangentes.

siendo Ra, Rb,

En 1842, Philip Beecroft, matemtico aficionado ingls descubri de manera independiente la misma frmula. Nuevamente en 1936 esta misma frmula es redescubierta esta vez por nada menos que Frederick Soddy (1877-1956) quin en 1921 haba ganado un premio Nbel en fsica por su descubrimiento de los istopos. Soddy expres esta frmula en forma de un poema llamado " el beso exacto",que fue publicado en la revista cientfica NATURE el 20 de Junio de 1936 en la pgina 1021. A continuacin se incluyen un extracto del poema original en ingls y la traduccin que aparece en el artculo Esferas y semiesferas de Martn Gardner en su libro Circo Matemtico (Alianza Editorial). Veamos: The smaller are the benter. The bend is just the inverse of The distance form the center. Though their intrigue left Euclid dumb There's now no need for rule of thumb. Since zero's bend's a dead straight line And concave bends have minus sign, The sum of the squares of all four bends Is half the square of their sum. Cuatro crculos llegaron a besarse, cuanto menores tanto ms curvados, y es su curvatura tan slo la inversa de la distancia desde el centro. Aunque este enigma a Euclides asombrara, ninguna regla emprica es necesaria: al ser las rectas de nula curvatura y ser las curvas cncavas tomadas negativas, la suma de los cuadrados de las cuatro curvaturas es igual a un medio del cuadrado de su suma.

Ntese que los dos primeros versos hacen clara referencia a los cuatro crculos cada uno tangentes a los otros tres. El tercer y cuarto verso definen la curvatura del crculo. Y el noveno y dcimo verso es la expresin literal de la frmula de Descartes. Cabe resaltar el hecho de que los crculos tangentes mostrados en la figura 1 se les suele denominar los "Crculos de Soddy". Como dato adicional fu Apolonio de Perga (260-180 A.C) quin estudi las propiedades de los crculos tangentes hace ms de 2000 aos. Aplicacin Usando la frmula de los crculos tangentes de Descartes demostrar el tercer teorema japons

Tomando en cuenta que la lnea tangente le corresponde curvatura cero entonces tendremos:

donde R1, R2, y R3, son las curvaturas de los tres crculos tangentes y por definicin de curvatura

Sustituyendo estas curvaturas

Efectuando operaciones resulta

y teniendo en cuenta que

Operando y simplificando resulta

y finalmente

l.q.q.d.

Primer Teorema Japons (Primer Teorema de Mikami y Kobayashi) Este teorema tambin es conocido con el nombre de "Antiguo Teorema Japons" y dice as: Sea un polgono convexo de n lados inscrito en una circulo de radio R. Hagamos la triangulacin del polgono trazando todas las diagonales a partir de uno de sus vrtices, ahora en cada tringulo as formado inscribamos crculos, entonces demustrese que la suma de los inradios (el inradio de un tringulo es el radio de la circunferencia inscrita en l) de cada tringulo es independiente de la triangulacin elegida. Demostracin: Enfoquemos nuestra atencin en el caso de un polgono de seis lados (n = 6), para esto

efectuemos dos triangulaciones diferentes (ver figuras 1 y 2)

Siendo r 1, r 2, r 3, y r 4, los inradios2 de cada tringulo mostrado en la figura 1, demostremos primeramente que: 4R + r 1 + r 2 + r 3 + r 4 = m 1 + m 2 + m 3 + m 4 + m 5 + m 6 donde m 1, ... m 6 son las distancias del centro O del crculo (de radio R) a cada uno de los lados del polgono. Para nuestra demostracin primero tracemos desde O perpendiculares de longitudes m a, m b, y m c, a las diagonales AC, FC y CE del polgono respectivamente. Usando el teorema de Carnot: En el tringulo ABC: m1 + m2 - ma = R + r1 En el tringulo ACF: m6 + ma - mb = R + r2 En el tringulo FCE: mb + mc + m5 = R + r3 En el tringulo ECD: m3 + m4 - mc = R + r4 Sumando miembro a miembro 4R + r 1 + r 2 + r 3 + r 4 = = m1 + m2 + m3 + m4 + m5 + m6 Anlogamente efectuando el mismo procedimiento usando la figura 2 se demuestra que:

4R + r' 1 + r' 2 + r' 3 + r' 4 = m 1 + m 2 + m 3 + m 4 + m 5 + m 6 Obsrvese que los segundos miembros de (2) y (3) son iguales luego se deduce r 1 + r 2 + r 3 + r 4 = r' 1 + r' 2 + r' 3 + r' 4 es decir que la suma de los inradios es independiente de la triangulacin elegida.

Segundo Teorema Japons (Segundo Teorema de Mikami y Kobayashi) Demostrar que al unir los incentros M, N, P y Q de los tringulos ABC, BCD, CDA y ABD formados al trazar las diagonales de un cuadriltero inscrito ABCD se forma un rectngulo (vase figura). Demostracin: Efectuemos algunos trazos auxiliares (de color rosa) como se aprecia en la figura. En el punto Q: (#1) Como M es el incentro del tringulo ABC entonces: (#2) (el ngulo C referido al tringulo ABC; ntese que BM es bisectriz del ngulo B). Anlogamente como Q es el incentro del tringulo ADB: (#3) (el ngulo D referido al tringulo ADB). Pero como el cuadriltero ABCD est inscrito en un crculo el &angulo C y el ngulo D son iguales. Los ngulos C y D referidos a los tringulos ABC y ADC respectivamente, se concluye que los ngulo AMB y AQB son iguales. Por tanto el cuadrilatero AQMB es inscriptible por lo que (#4) Anlogamente se demuestra que el cuadriltero AQPD es tambin inscriptible por lo que (#5) Remplazando (#4) y (#5) en (#1):

Sin embargo en el cuadriltero inscriptible ABCD se cumple Por tanto el ngulo MQP es recto. De modo igual se demuestra que:

Por tanto el cuadriltero PQMN es un RECTGULO

l.q.q.d.

Tercer Teorema Japons (Tercer Teorema de Mikami y Kobayashi) Considrese tres crculos tangentes entre s y tangentes a una misma recta, donde: r 1 < r 2 < r 3 Entonces

Demostracin Unamos los centros de los tres crculos tangentes, ahora desde estos mismos centros levantemos tres perpendiculares O 2A, O 1B y O
3C

a la recta tangente. Desde el centro O 1 del crculo ms pequeo trazemos perpendiculares a los radios O 2A y O 3C en D y E respectivamente.

Por construccin: DE = AC. Aplicando el Teorema de Pitagoras en los tringulos rectngulos O 2DO 1 y O 1EO 3:

Sumando (#2) y (#3) obtenemos DE que sabemos es igual AC. Por tanto:

Dividiendo cada uno de los trminos de la expresin anterior por l.q.q.d. Huaral, 16 de mayo 2002

Julio A. Miranda Ubaldo Email: jmiub@yahoo.com

Puedes ver una demostracin del teorema de Carnot en Notas Matemticas

Teorema de Carnot

Sea O el circuncentro del tringulo y X, Y y Z los puntos medios de los lados a, b y c respectivamente, R y r los radios de las circunferencias circunscrita e inscrita respectivamente. El teorema dice que OX + OY + OZ = R + r siendo OX, OY y OZ positivos si el circuncentro esta en el mismo semiplano respecto al lado correspondiente que el vrtice opuesto, y negativo en caso contrario.

Utilizamos S = (ABC), Sa = (OBC), Sb = (OCA) y Sc = (OAB) para las reas de los respectivos tringulos, y s = (a + b + c)/2 para el semipermetro del ABC.

1) Tringulo es acutngulo (circuncentro interior) Hagamos OX = x, OY = y, OZ = z. Aplicando el Teorema de Ptolomeo a los cuadrilteros cclicos OYAZ, tenemos: (c/2) y + (b/2) z = (a/2) R cy + bz = aR (#1) Igualmente para OZBX y OXCY, az + cx = bR (#2) bx + ay = cR (#3) S = Sa + Sb + Sc ax + by + cz = 2S (#4)

Sumando (#1), (#2), (#3) y (#4), resulta x (a + b + c) + y (a + b + c) + z (a + b + c) = R (a + b + c) + 2S (x + y + z) 2s = R 2s + 2S (x + y + z) s = Rs + rs x+y+z=R+r OX + OY + OZ = R + r 2) El tringulo es obtusngulo (circuncentro exterior)

Hagamos OX = x, OY = y, OZ = -z. Aplicando el Teorema de Ptolomeo a los cuadrilteros cclicos OZYA, tenemos: (a/2)R + (b/2)z = (c/2)y aR + bz = cy aR = cy - bz (#1) Igualmente para OZXB y OXCY, az + bR = cx bR = cx - az (#2) bx + ay = cR cR = bx + ay (#3) S = Sa + Sb - Sc ax + by - cz = 2S (#4) Sumando #1, #2, #3 y #4, x (a + b + c) + y (a + b + c) - z(a + b + c) = R (a + b + c) + 2S (x + y - z) 2s = R 2s + 2S (x + y - z) s = R s + r s x+y-z=R+r OX + OY + OZ = R + r 3) Tringulo rectngulo (circuncentro en la hipotenusa) Basta hacer z = 0 en cualquiera de los casos anteriores. Ignacio Larrosa

Nota El teorema de Ptolomeo dice: Si el cuadriltero ABCD est inscrito en una circunferencia la suma de los productos de los lados opuestos es igual al producto de las diagonales

AB DC + BD AC = AD BC Si el cuadriltero es un rectngulo resulta el teorema de Pitgoras El ngulo de Brocard Julio A. Miranda Ubaldo (Per) Sea un tringulo ABC cualquiera y ubiquemos en el interior un punto P de manera que

De acuerdo con este criterio construimos la figura adjunta. El ngulo se denomina ngulo de Brocard y el punto P punto de Brocard Existe una relacin trigonomtrica entre las medidas de los ngulos interiores del tringulo ABC y el ngulo de Brocard. Si entonces Demostracin La figura adjunta muestra los trazos adecuados que indica el procedimiento siguiente: Por B se traza una recta paralela al lado AC. Se prolonga AP de modo que corte a dicha paralela en el punto F entonces

Luego unimos F con C y por F trazamos una perpendicular a la prolongacin de AC en el punto G. Anlogamente desde el punto B trazamos una perpendicular a AC en el punto H. Observemos BH = FG = h Al ser el cuadriltero PBFC es inscriptible. luego adems que

Ahora bien del tringulo APB

Por tanto al ser el cuadriltero PBFC inscriptible

En el tringulo rectngulo AGF resulta sustituyendo en (#1)

(#1) Como AG = AH + HC + CG

(#2) Por otra parte: En el trigulo AHB En el trigulo BHC En el trigulo CGF

Sustituyendo estas expresiones en (#2) resulta la expresin buscada. Estimado lector, qu te pareci la desmostracin?. Fcil verdad?

Aplicacin En la siguiente figura hallar el ngulo de Brocard Solucin geomtrica De la figura dada se observa que (recurdese que el tringulo rectngulo ABC es issceles AB = BC) y adems que y Prolongamos BP y por el punto A trazamos una perpendicular a dicha prolongacin en el punto Q. Entonces Por tanto el tringulo AQP es rectngulo issceles en y en consecuencia AQ = PQ = a. El tringulo BPC es congruente con el tringulo BQA (criterio ALA). En consecuencia: AQ = BP = a Finalmente el tringulo BQA es rectngulo notable pues BQ = 2 AQ, por tanto = 26 33' 54.184"

Solucin trigonomtrica Teniendo en cuenta que y sustituyendo en resulta

de donde

y por tanto = 26 33' 54.184"

Como pueden notar la solucin geomitrica es ms laboriosa mientras que la solucin trigonomtrica es bastante prctica y sencilla. Buen provecho! Julio A. Miranda Ubaldo Nmeros capicas. Francisco Javier Asencor Una carta de nuestro amigo Francisco Javier Asencor Querido amigo: Resulta simptica la introduccin con anlisis-felicitacin con que introduces la actual (se refiere a la anterior lgicamente) edicin de la Gacetilla. Se menciona el hecho de que el nmero del ao iniciado es capica. Este tipo de clasificacin de los nmeros, atendiendo a sus cifras genera una "paramatemtica" con inters ldico pero que no me resisto a tocar. Resulta que en los ltimos mil aos, slo nosotros (los que el 1 de enero ltimo vivamos con edad de 11 aos o ms) y algn longevo (?) hemos tenido la oportunidad de vivir dos aos capicas, (1991 y 2002). Y esto no volver a ocurrir hasta dentro de otros mil aos salvo para los pocos longevos que superen los 110 aos. (Bueno, tal vez puedan ser muchos si la esperanza de vida se sigue alargando...). Todo esto es vlido slo para las personas que nos regimos por el calendario de la Era Cristiana. Incluir el resto es una labor que se me escapa. Volviendo a la clasificacin de los nmeros por consideraciones sobre sus cifras. Establecer algoritmos que supongan operar con las cifras permite relaciones de equivalencia y de orden entre ellas que suponen un entretenimiento. Permiten establecer clasificaciones y ordenaciones muchas veces sorprendentes, as como propiedades de algunos nmeros. Si contemplamos algn valor didctico, resulta un ejercicio formativo tratar de describirlos con lenguaje simblico matemtico riguroso. Adjunto una posibilidad, resulta simptico admitir que todos nos entendemos cuando decimos "son los nmeros que se leen igual de atrs para adelante". Salvo el 1 (uno), que es un "pesao" trivial que sale por todos los sitios, el 81 es el nico nmero que es igual al cuadrado de la suma de sus cifras... con el cubo les ocurre a varios, etc. Definir "nmero capica" Como convencisn admitimos el idioma espaol en las expresiones literales y los smbolos arbigos, utilizados en base decimal, para las expresiones numricas Simbolizamos con N el conjunto de nmeros naturales, N = {1, 2, ...} y N# = {2, 3, 4 ...} el de los nmeros naturales excluyendo el 1. Sean los nmeros n N y b N#. Sea el conjunto D = {0, 1, 2, ... b - 1} Para cada n y cada b existe un nico conjunto ordenado e infinito de nmeros, Sn, b: que cumple

y lo denominamos cifras de n expresado en base b Al valor lo denominamos nmero de cifras de n.

Con lo que puede expresarse Los valores de n para los que a i = am - i - 1 para todo i < m,se denominan capicas en base b. Por antonomasia, los capicas en base 10, se denominan capicas. Por ejemplo: Si tomamos n el nmero que adjudicamos al ao en curso en la Era Cristiana. Para b = 7 n = 070 + 671 + 572 + 573 + 074 + 075 + ... Sn, 7 = {0, 6, 5, 5, 0, 0, ...} con lo que m = 4. Puesto que a 0 = 0 y a 3 = 5, se tiene que a 0 es distinto de a 3 y en consecuencia n no es capica en base 7. Para b = 10 n = 2100 + 0101 + 0102 + 2103 + 0104 + 0105 + ... Sn, 10 = {2, 0, 0, 2, 0, 0, ...} con lo que m = 3. Puesto que a 0 = a 3 = 2, tambin a 1 = a 2 = 0, en consecuencia n es capica en base 10 o simplemente capica. Un cordial saludo. Javier

PALNDROMOS Conjetura del Capica Aqu te te dejamos un mtodo para obtener nmeros capicas. Dicho mtodo indica que dado un nmero y sumndolo con el que resulta de invertir sus cifras, despus de un nmero finito de pasos, obtenemos un capica. Por ejemplo: N = 42 N = 28 N = 87 42 + 24 = 66 28 + 82 = 110 110 + 011 = 121 87 + 78 = 165 165 + 561 = 726 726 + 627 = 1353 1353 + 3531 = 4884

Pero cuidado, es slo una conjetura. Si quieres medio comprobarlo prueba con el nmero 89 y si lo consigues para el nmero 196 ... Enhorabuena! Inversiones Geomtricas. Francisco Javier Asencor Dado un punto del espacio, O, que denominamos polo, decimos que un punto del espacio, P, es inverso de otro, Q, de potencia m, si O, P y Q estn alineados y cumplen que la distancia OP multiplicada por la distancia OQ es igual a m > 0. La transformacin tiene la propiedad simtrica. Se exponen aqu algunos resultados interesantes de esta transformacin. Como espacio tomamos el plano, y sin perder generalidad tomaremos el origen de coordenadas como polo, y la unidad como potencia. Cualquier conjunto de puntos exteriores a un circulo de radio unidad centrado en el origen

tendr todos sus inversos en este circulo, y viceversa. Utilizando las coordenadas cartesianas, un punto determinado por el par (x, y) tiene su inverso en el punto (u, v) si:

Singularidades El nico punto que carece de inverso es propio polo. (Cuya inverso corresponde al infinito al anularse el denominador) Los puntos de la circunferencia centrada en el origen de radio unidad son inversos de s mismos. (al hacerse uno el denominador). Figuras Cualquier figura, mediante la inversin de los puntos que la componen, determina su figura inversa. Rectas La inversa de cualquier recta que pase por el origen es ella misma. Esto no es cierto para todos los segmentos de dichas rectas. Slo son inversos de s mismos los segmentos comprendidos entre un punto y su inverso. Cualquier otra recta, no pasando por el origen, admite como expresin: 2 ax + 2 by = 1 y su inversa que operando puede escribrirse (u - a) 2 + (v - b) 2 = a 2 + b 2 que es la ecuacin de los puntos (u, v) que componen una circunferencia centrada en el punto (a, b) y que contiene el origen ( ). La inversa de la recta, azul, es una circunferencia, roja, que pasa por el polo. Recprocamente, la inversa de una circunferencia que pasa por el polo es una recta.)

Circunferencias Los puntos (x, y) que forman una circunferencia centrada en el origen y de radio R, cumplen que x 2 + y 2 = R 2. La inversa de esta figura la componen los puntos (u, v) que cumplen

por tanto constituyen una circunferencia centrada en el origen de radio 1/R.

Es posible hallar la inversa de una circunferencia en general. Los puntos (x, y) que forman una circunferencia centrada en (a, b) y de radio R, cumplen que (x - a) 2 + (y - b) 2 = R 2 La inversa de esta figura la componen los puntos (u, v) que cumplen

Operando en esta expresin puede obtenerse

una circunferencia centrada en

pue no es inverso de (a, b) y de radio

Un ejemplo de aplicacin Tomemos dos circunferencias de radio unidad C g, tangentes entre s en el origen, y tangentes a una recta C 0. Consideremos la sucesin de circunferencias tal que la primera es tangente a C 0 y a las C g. Las siguientes, cada una tangente a la anterior y a las Cg. Como ilustra la figura.

La linea de centros de esta serie contiene el punto de tangencia de la primera con C 0 a una distancia unidad del origen. Se puede comprobar que los restantes puntos de tangencia se encuentran a distancias 1/2, 1/3, 1/4, ... formando la serie armnica. De manera ms o menos laboriosa puede comprobarse para los primeros resultados utilizando rudimentos de geometra, pero se ofrece esta propuesta:

La figura de la izquierda representa las C g y la C 0 en azul mas una circunferencia de inversin en negro. La figura de la derecha representa su inverso. C 0 como una circunferencia interior, y las C g como rectas verticales. Si encajamos entre las lneas verticales circunferencias sucesivamente tangentes a modo de pila, los puntos de tangencia se encuentran necesariamente a distancias 1, 2, 3, .... Puesto que al invertir esta figura reencontramos la sucesin propuesta, las inversas de estos valores son las distancias buscadas.

Frmula de Pick. George Alexander Pick fu un matemtico austriaco nacido en Viena (1859) que muri en un campo de concentracin nazi durante la II Guerra Mundial (se cree que en 1943). G.A.Pick estableci la relacin que existe entre los nudos de una malla y la rea de un polgono dibujado sobre ella. Malla cuadrada de 10 10 Pueden construirse, evidentemente, mallas de muy diversas pixels. La interseccin de dos rectas de las malla se denomina maneras. Aqu vamos a considerar una malla construda a partir de rectas paralelas y verticales. Cada punto de nudo (en rojo en la figura) interseccin de una recta horizontal y una vertical se denomina nudo. Un cuadrado de dicha malla ser la unidad de superficie.

No existen nudos interiores en los polgonos dibujados sobre la malla. Entonces, si NP son los nudos sobre el permetro Para la Fig 1 es NP = 12 con lo que A =(1/2) 12 - 1 = 5 Para la Fig 2 es NP = 5 con lo que
No existen nudos interiores en ninguno de los polgonos

A =(1/2) 5 - 1 = 1,5

Para la Fig 3 es NP = 12 con lo que A =(1/2) 12 - 1 = 5

Generalizacin Definimos un lado de la malla como el segmento determinado por dos nodos consecutivos. Entonces, el rea de una regin poligonal con los vrtices sobre los nodos de la malla viene dado por

donde NT es el nmero total de nodos de la malla y L los lados de la malla. Para la Fig 4 es NT = 15, L = 12 por lo que A = 8 Para la Fig 5 es NT = 11, L = 8 por lo que A = 6 Para la Fig 6 es NT = 18, L = 14 por lo que A = 10
Existen nudos interiores.

Composiciones de figuras.
Puede utilizarse tambin la expresin A = (NP/2) + NI - 1 donde NP es el nmero de nodos de la frontera y NI el nmero de nodos interiores. Esta expresin es lgicamente equivalente a la dada. Para la Fig 6 resulta NP = 14, NI =4y A = 7 + 4 - 1 = 10

Para la Fig 7 resulta: NT = 25, L = 24, A = 12 Para la Fig 8 resulta: NT = 13, L = 12, A = 6 Para figuras ms complicada

NT = 33; L = 19; A = 22,5

Figuras con "agujeros" Es vlida la expresin

en donde NT y L son los mismos parmetros anteriores y C es el nmero de "agujeros" que tiene la figura.

NT = 42; L = 12 + 14 (interiores); C = 2; A = 30

NT = 31; L = 20 + 5 (interiores); C = 1; A = 18,5

La Conjetura de Goldbach.
Posible demostracin a la conjetura Matemtica de Goldbach Prof. Mario Peral Manzo. U.P.N. Mxico.

(...) todo nmero, sea cual fuere, no es sino el nmero nueve o su mltiplo ms un excedente, pues los signos de los nmeros no tienen ms que nueve caracteres.

Avicena (1)
Clawson apunta: "Junto con Goldbach, Euler fue miembro de la Academia Rusa de Ciencias. Ambos hombres sentan pasin por las series infinitas y los nmeros primos. Goldbach, en una carta que le envi a Euler el 7 de junio de 1742, especul que todo nmero par es la suma de dos primos [Conjetura binaria] y que todo nmero impar mayor que 2 es la suma de tres primos [Conjetura ternaria]" (2) . El presente trabajo intenta ser base para una posible demostracin de la "Conjetura Binaria de Goldbach" (CBG) que dice: cualquier nmero par mayor o igual a cuatro es resultado de cuando menos una suma de dos nmeros primos. El planteamiento, aparentemente simple, no ha sido demostrado, desde su formulacin, dada la infinidad

de nmeros naturales; de suerte que no se ha llegado a una generalizacin plausible que de una vez y por todas erija esta conjetura (presuncin fundada) en teorema (proposicin que afirma una verdad demostrable). La demostracin de la CBG, significara un portentoso avance en la comprensin de los nmeros primos (nmeros solamente divisibles por s mismos y por la unidad): de las razones de por qu se presentan con una aparente irregularidad o azar, adems de cmo producirlos mediante algn algoritmo simple o complejo. La utilidad de los nmeros primos ya ha sido comprobada en la elaboracin de cada vez ms sofisticados cdigos (la caja fuerte de la informacin restringida). Esto lo saben muy bien y de manera especial los pases anglosajones (La Gran Bretaa y Los Estados Unidos de Norteamrica) quienes han tenido que lidiar con el problema de cmo esconder informacin "estratgica"; an ahora cuando presumimos que ya no hay "guerra fra". Por otro lado, supondra la elaboracin de algoritmos ms eficientes para el manejo de grandes volmenes de informacin dado la inusitada evolucin de los sistemas informticos, en cuya base, desde luego, se encuentran las computadoras ms poderosas, todava muy lejos de ser accesibles para quienes contamos con las interfases (las computadoras personales). No resulta extrao, pues, que a raz de la publicacin del libro de Apstolos Doxadis (3) los editores de ste (Faber & Faber) hayan ofrecido un milln de dlares a quien dentro de los prximos dos aos demuestre la CBG. Pero lo que no es de extraar no es tanto la gran cantidad de dinero que se ofrece, sino el que se limite el jugoso reto a los residentes legales de la Gran Bretaa y de los Estados Unidos de Norteamrica. (4) Es posible que una especie de fervor nacionalista semejante a la de Europa durante la poca en la que, como ilustre ejemplo, el propio Fermat enunciara su clebre (ltima) conjetura, intente reservar el logro de esta hazaa nicamente a los pases anglosajones mencionados?. (5) No lo creemos, pero resulta un poco incmodo que la mencionada editora evite la participacin de los matemticos de origen latino (europeo o americano). En todo caso sera deseable, tal vez, que alguna casa editorial abra un reto semejante para los matemticos de todo el mundo. Lo que s es cierto, es que urge conocer mejor el "comportamiento" de los nmeros primos en un mundo en el que es necesario una competente produccin, manejo y circulacin de un creciente volumen de informacin y ante la perspectiva de ms eficientes ordenadores. Las propuestas que se ofrecen en este escrito, tienen como base la idea de la existencia de una infinidad de sistemas en la naturaleza. Infinidad de sistemas que, aparentemente en relacin catica entre s, requieren de una eficiente inversin de energa para obtener de ellos un razonable volumen de informacin (en forma de conocimiento y de tecnologa). Establecemos una analoga entre estos sistemas naturales y el conjunto de los nmeros naturales (de hecho la operacin con nmeros han permitido describir coherentemente los procesos naturales; al cuantificar o medir hacemos que la incertidumbre se "colapse" en informacin til para nuestros humanos fines). Esta analoga se hace extensiva a la nocin de entropa (la medida del caos); suponemos que tratando el conjunto de los nmeros naturales como una infinidad de sistemas en interrelacin, sus relaciones, particularmente las de los nmeros primos, nos permitirn idear un modelo recursivo que nos ayude en el "colapso" para obtener

informacin sobre lo enunciado en la CBG. Para comenzar, suponemos que los nmeros pares y los impares representan, cada uno, el cincuenta por ciento en el contexto del conjunto de los nmeros naturales. Nuestro punto de partida es un conjunto de doce presunciones. (6) 1. El nmero de sistemas existentes en la naturaleza tiende al infinito. 2. Los sistemas presentes en la naturaleza, aunque en nmero tiendan al infinito, de igual modo (dado que no estn aislados unos de otros) tienden a igualar sus diferencias relativas, es decir, a alcanzar el mximo grado de entropa o equilibrio trmico dado que estn inextricablemente interrelacionados mediante la informacin que portan. "La segunda ley de la termodinmica dice que la entropa de un sistema cerrado, es decir aislado, siempre aumenta o se conserva pero nunca disminuye. Cuando un sistema cerrado alcanza el equilibrio t>;rmico se encuentra en un estado de mxima entropa." (7) 3. Todo numeral (representacin grfica de un nmero) constituido por una o ms cifras puede ser tratado como un sistema y, por consiguiente, como portador de informacin. Al respecto, Hayles afirma: "Supongamos que envo un mensaje que contiene la serie 2, 4, 6, 8... y le pido que contine la secuencia. Como usted capta el modelo subyacente, puede ampliar la serie indefinidamente, aun cuando slo se especifiquen unos pocos nmeros. La informacin que tiene un modelo puede ser comprimida en formas ms compactas. Yo podra haber enviado el mensaje en la forma: "cite los nmeros enteros pares, empezando con el 2". Supongamos por el contrario, que le envo a usted la salida de un generador de nmeros al azar. Cualquiera que sea la cantidad de nmeros que yo transmita, usted no podr continuar la secuencia. Cada nmero es una sorpresa; cada nmero transmite nueva informacin. Segn este razonamiento, mientras ms aleatorio o catico es un mensaje, ms informacin contiene." (8) 4. Limitndonos al conjunto de los nmeros naturales, sabemos que ste est constituido por una infinidad de sistemas: los sistemas del subconjunto de los nmeros pares (2n) y los del subconjunto de los impares (2n + 1). Al igual que estos dos subconjuntos, los nmeros primarios por s solos conforman una infinidad de sistemas. 5. La sustraccin es una operacin que hace evidente la diferencia entre dos sistemas (numerales) que comparamos entre s. 6. Toda diferencia (producto de la sustraccin) entre dos numerales puede ser cuantificada en trminos de las desigualdades entre los valores absolutos de los dgitos que los constituyen. Consideremos por ejemplo la resta 87 - 78; restemos las diferencias entre sus valores absolutos... Para las decenas: 8 - 7 = 1 y para las unidades: 7 - 8= -1; vemos que la diferencia entre los valores absolutos de estos resultados es cero debido a la operacin (1) + (-1) pues tanto el minuendo como el sustraendo de la sustraccin ejemplificada comparten los mismos dgitos pero ordenados de distinto modo. Pero si consideramos los valores relativos de esas diferencias (1 decena menos una unidad) el resultado es de 9. En trminos "absolutos" la informacin contenida en estos sistemas es la misma y, por consiguiente, el cambio es de cero para dichos sistemas considerados en conjunto; empero, en trminos "relativos" hay un cambio desde el punto de vista de un sistema en relacin con el otro. Si consideramos la resta como un todo, es decir, como un sistema aislado entonces, como afirma Abbot: "...esto debe dar a entender que el estado de equilibrio de un sistema aislado es aqul en el cual la entropa alcanz su valor

mximo con respecto a todas las variaciones posibles. La condicin matemtica para este mximo es: dSSISTEMA = 0 (sistema aislado)". (9) 7. De acuerdo con la anterior presuncin: las diferencias entre valores absolutos de los dgitos de un numeral pueden ser expresadas en trminos de la suma algebraica de esos mismos valores: (da) (suma de las diferencias entre los valores absolutos de los dgitos de los numerales involucrados en una sustraccin). Para un mayor abundamiento, tomemos la resta: 231 - 123; vemos que para el caso de las centenas 2 - 1 = 1; para las decenas 3 - 2 = 1 y para las unidades 1 - 3 = -2. De este modo y para esta sustraccisn en particular, da = 1 + 1 + (-2) =0 8. Las diferencias entre valores absolutos de los dgitos de un numeral pueden ser expresadas como una suma de los valores relativos de dichos dgitos (dr) y su resultado, si es mayor a 9, puede ser expresado en trminos de da. Retomemos el ejemplo propuesto en la anterior presuncin: vimos que da = 1 + 1 + (-2) = 0 y al realizar la suma algebraica de los valores relativos (dr) de estos sumandos, tenemos que dr = 100 + 10 + (-2) = 108; como 108 es mayor a 9, tendramos que interpretarlo en trminos de da nuevamente, o sea: da = 1 + 0 + 8 = 9. De esta suerte afirmamos que 9 es complemento de 0 dentro de una topografa recursiva, lo que nos lleva a... 9. La dr de un numeral complementa a da y permite la determinacin del conjunto de dN (suma de los valores absolutos de los dgitos de cualquier nmero natural) que asume (en calidad de informacin inicial) los valores comprendidos entre el 1 y el 9 inclusive. Como ejemplo: consideremos el numeral 24857; la suma de los valores absolutos de sus dgitos sera expresado as: da = 2 + 4 + 8 + 5 + 7 = 26 y como el resultado debe expresarse como un solo nmero dgito, entonces continuamos la suma, 2 + 6 = 8, resultado que debe ser tratado como informacin inicial pues, por razones expresadas en las siguientes presunciones, el numeral 24857 (aunque asume el valor de 8 del conjunto dN) se colapsara en la "Resultante" -1 (R - 1) por ser nmero impar dentro de una "topografa recursiva" constituida por los valores comprendidos entre -9 y 9 inclusive y por ser 8 y -1 complementarios dentro de esta topografa. El conjunto de "Resultantes" o parejas complementarias, se expresa como: R={(0,-9), (1,-8), (2,-7), (3,-6), (4,-5), (5,-4), (6,-3), (7,-2), (8,-1), (9,0)} por razones que se explican ms abajo. 10. Las R sealan, dentro de una topografa recursiva, los estados ms probables de un nmero infinito de numerales y es deducida del conjunto de las dN; lo que, enlazando con la nocin de "mxima entropa", nos lleva a la expresin: en el universo los sistemas tienden a equilibrarse, a eliminar sus diferencias relativas. Pues, tal y como subraya Csarman: "La posibilidad de utilizar el lenguaje de la termodinmica y el concepto de entropa como un comn denominador en el caleidoscopio del conocimiento humano se debe al carcter general y universal de sus leyes, a que utiliza parmetros macroscpicos que no necesitan definirse en funcin de las innumerables variables microscpicas que los determinan, a que se manejan con pocas variables, a que es aplicable al estudio de los sistemas y a que todo en la naturaleza son sistemas compuestos por las

continuas transformaciones de la materia y de la energa, a que todos los sistemas desde los ms simples hasta los ms complejos, son el resultado del arreglo de las partes ms elementales de energa y de materia con que se ha estructurado el Universo y a que, por ltimo, se trata de un lenguaje dinmico y no esttico que corresponde al carcter continuamente cambiante de la naturaleza." (10) 11. Si el estado final de los sistemas en general pueden ser deducidos a partir de la igualacin de sus diferencias relativas, entonces sus propiedades pueden ser deducidas, a su vez, de esos estados finales. En otras palabras, se puede determinar con probabilidad 1 la posicin de cualquier nmero natural dentro de la topografa recursiva constituida por las R, cuya construccin se explica lneas ms abajo. Esto es muy significativo para nuestro propsito de acercarnos a la conjetura binaria de Goldbach.

La Conjetura de Goldbach.
Posible demostracin a la conjetura Matemtica de Goldbach Prof. Mario Peral Manzo. U.P.N. Mxico.
(...) todo nmero, sea cual fuere, no es sino el nmero nueve o su mltiplo ms un excedente, pues los signos de los nmeros no tienen ms que nueve caracteres.

Avicena (1)
Los resultados de la aplicacin de nuestras presunciones. Tras la aplicacin de las doce presunciones expuestas en la resolucin de cien sustracciones en trminos de da y dr, se determinaron las R o resultantes. Los resultados idnticos para da y dr se obviaron con el fin de encontrar las parejas de "resultantes" que nos serviran para elaborar nuestra topografa. Despus de revisar los mencionados casos, pudimos determinar el conjunto: R = {(0,-9), (1,-8), (2,-7), (3,-6), (4,-5), (5,-4), (6,-3), (7,-2), (8,-1), (9,0)} Con estos datos se procedi a realizar la siguiente figura que es una topografa recursiva que nos permite representar las R y operar con el presupuesto nmero once; es decir, ubicar las parejas resultantes de la determinacin de la dN de cualquier nmero en una recta numrica cerrada (0,9; -9, 0).

Bsicamente esta grfica es una recta numrica "doblada" de tal manera que permite que se traslapen los puntos extremos (como una serpiente que se mordiera la cola). Las resultantes, en realidad seran las parejas de puntos "antpodos" evidenciados ms que como opositores, como complementarios, producto de este ejercicio mental de representar en forma de "rizo" rectas numricas; y sustentado por las presunciones arriba enunciadas. La recta numrica, tal es esta topografa, nos permite realizar conteos a partir del punto cero. De esta suerte, si se desea saber en cul posicisn "caer" el nmero impar 163, podramos hacer tres cosas: contar mediante pequeos saltos sobre los puntos numerados hasta completar 163 giros, o bien hacer una divisin (163/18) que indicara cuantas vueltas debe drsele al crculo junto con cuntos giros o, mejor an, determinar la dN de 163. Cualquiera de los procedimientos nos conducira al punto ocupado por la resultante 1. Veamos: si dividimos 163 entre 18, el resultado sera 9 vueltas ms un giro (posicisn final: R1 (resultante uno) el residuo indica la posicin final del conteo; por otro lado, si realizamos la suma de los valores absolutos de 163, tendramos da163 = 1 + 6 + 3 =10; como 10 > 9, entonces da10 = 1 + 0 = 1, es decir dN163 = 1 (que arroja nuestra informacin inicial) y que decidimos que es la indicada para expresarse como R1 dado que 163 es nmero impar; de este modo, como conclusin expresamos: R163 = 1 (obviamente el mismo resultado). Este ltimo procedimiento es muy prctico para determinar las R de nmeros muy grandes. Por ejemplo: Cul es la R del nmero 4 597 863 282? Resolveramos as: da4 597 863 282 = 4 + 5 + 9 + 7 + 8 + 6 + 3 + 2 + 8 + 2 = 54 da54 = 5 + 4 = 9, por lo tanto... dN4 597 863 282 = 9 (informacin inicial) y dado que 4 597 863 282 es par, decidimos que la resultante para l es cero (R0), o bien R4 597 863 282 = 0 Observemos que si antes de contar se eliminan los dgitos 9 del numeral, la suma resultar ser ms rpida. Veamos: si a 4 597 863 282 le cancelamos los dgitos 9 que contenga, queda el numeral 457863282 si, adems, observamos que los dgitos subrayados suman nueve y los eliminamos, tendremos un nuevo numeral: 78282 si ahora observamos que los dgitos subrayados suman 18 que es mltiplo de nueve y los eliminamos tendremos el

nmero 72 cuyo da = 7 + 2 = 9. Con el fin de inferir algunas regularidades en la aplicacin de nuestras doce presunciones, recurrimos al simple conteo de los "saltos" y las "vueltas" que se completaran con los primeros 36 nmeros naturales (dado que 36 es mltiplo de 9). Imaginemos una rana que realiza saltos a partir de cero de acuerdo con las instrucciones que nosotros le demos; por ejemplo: "rana, salta dos veces"; entonces nosotros procedemos a verificar el lugar dentro del crculo donde la rana ha llegado despus de cumplir con la instruccin dada. Registramos el resultado en nuestra tabla de instrucciones en donde basicamente se da respuesta a las preguntas: si dentro de nuestra topografa la rana da saltos en nmero impar, hasta dnde llegar?, cuntos saltos (giros de 20) tuvo que dar y cuntas vueltas (giros de 360) complet para llegar hasta el lugar al que lleg (o sea la "resultante")?; a iguales preguntas tendramos que dar respuesta cuando nuestras instrucciones involucren saltos en nmero par. Hemos tenido cuidado de que nuestra tabla nos permita visualizar tanto las instrucciones en nmero par como las de nmero impar con el fin de visualizar las preguntas bsicas que nos hemos planteado, junto con sus respuestas ("colapsos") para derivar algunas consecuencias que podamos expresar de manera formal acerca del comportamiento de la rana de acuerdo con nuestras instrucciones precisas. CONTEO DE "N" SOBRE LA RECTA NUMRICA CERRADA (0, -9; 9, 0) Numeros Naturales (N) Giro de 360 o "vueltas" (V) Giro de 20 o "saltos" (n) Punto final de conteo o "Resultante" (R)

pares impares pares impares pares impares pares impares

2 4 6 8 10 12 14 16 18 20 22 24 26 28 30 32 34 36 (...)

1 3 5 7 9 11 13 15 17 19 21 23 25 27 29 31 33 35 (...)

0 0 0 0 0 0 0 0 1 1 1 1 1 1 1 1 1 2 (...)

0 0 0 0 0 0 0 0 0 1 1 1 1 1 1 1 1 1 (...)

2 4 6 8 10 12 14 16 0 2 4 6 8 10 12 14 16 0 (...)

1 3 5 7 9 11 13 15 17 1 3 5 7 9 11 13 15 17 (...)

2 4 6 8 -8 -6 -4 -2 0 2 4 6 8 -8 -6 -4 -2 0 (...)

1 3 5 7 9, -9 -7 -5 -3 -1 1 3 5 7 9, -9 -7 -5 -3 -1 (...)

En la columna "N" de la tabla de conteos expuesta arriba se presentan los 2n y los 2n + 1 que se sometieron a giros, mediante conteos sencillos, dentro de nuestra topografa de la curva cerrada. (Las instrucciones u rdenes dadas a la rana) Los conteos se realizaron comenzando por el punto cero de esta figura, en el sentido de las manecillas del reloj y se registraron: en la columna de "giros de 360" las "vueltas" completas (18/18) (realizadas por la rana), en tanto que en la de "giros de 20", los "saltos" (n/18) (dados por esa misma rana). En las dos ltimas columnas se registraron los "resultantes" (R), que indican el punto en el cual el nmero cae o, si se prefiere, "se colapsa" despus de haber realizado las da correspondientes (el lugar en el que aparece nuestra rana despus de cumplir con nuestras instrucciones). De la anterior tabla, derivamos las proposiciones (expresiones formales acerca del comportamiento de nuestra rana en un contexto preciso dadas una instrucciones tambin precisas): A. El conteo de nmeros naturales (N): V(18) + n = N N/18 = V + n/18 Significado: podemos enterarnos de las instrucciones dadas a la rana a partir de saber cuntas vueltas y giros complet y viceversa. B. Los "R" del conteo de Pares: n <= 8 > 0 -> n + 0 = R n > 8 < 18 -> n + (-18) = -R n = 0 -> R= 0 Significado: cuando nuestras instrucciones a la rana estn expresadas en nmeros pares, la rana no tendr otra opcin que "colapsarse" en una posicin par (negativa o positiva) o cero dentro de nuestra topografa, sin importar que el dN del nmero par se exprese como impar. C. Los "R" del conteo de Impares: n <= 8 > 0 -> n + 0 = R n = 9 -> R, -R = 9, -9 n > 8 < 18 -> n + (-18) = -R n = 1 -> R= 1 Significado: cuando nuestras instrucciones a la rana estn expresadas en nmeros impares, la rana no tendr otra opcin que "colapsarse" en una posicin impar (negativa o positiva) dentro de nuestra topografa, sin importar que el dN del nmero impar se exprese como par. Podemos obtener las siguientes "consecuencias" de las proposiciones enunciadas: 1. 2. 3. 4. Las "R" para los 2n, siempre sern pares (positivos o negativos) o cero. Las "R" para los 2n + 1, siempre sern impares (positivos o negativos). Las 2n + 1 no expresarn R = 0 al ser reducidos por sus da sucesivas. (R, -R) = (9, -9), expresa la recursividad del conjunto N y por tanto de sus propiedades. 5. Para el caso de la "Conjetura Binaria de Goldbach", las proposiciones enunciadas permiten operar de manera ms eficiente, mediante las dN. Recordemos que la mencionada conjetura expresa: "Puede escribirse a todo nmero par igual o mayor a 4 como la suma de dos primos?" (11) Si suponemos como vlida la presuncin once enunciada en este trabajo, entonces podemos suponer que los "R" representan a todos los 2n (incluido el 2; nico primo par) y los 2n + 1 (incluidos los nmeros primos mayores a 2) de los N.

As, por ejemplo: 2860 (par) puede descomponerse (entre otras opciones) en la suma de los primos: 2819 + 41. Ahora bien, si aplicamos nuestro procedimiento tenemos: Para 2n = 2860; da (2 + 8 + 6 + 0 = 16); da (1 + 6 = 7) dado dN (7,-2) R= -2 (pues 2860 es un nmero par) Para 2n + 1 = 2819; da (2 + 8 + 1 + 9 = 20); da (2 + 0 = 2) dado dN (2, -7) R= -7 (pues 2819 es un nmero impar). Para 2n + 1 = 41; da (4 + 1 = 5) dado dN (5,-4) R= 5 (pues 41 es un nmero impar) De este modo: (-7) + 5 = -2 Por lo tanto R = 2n > 2 (positivos o negativos) puede expresarse mediante las sumas de dos "R primos" presentes en nuestro modelo (2, 3, 5, 7 y -7, -5, -3, -2) y siempre y cuando la suma algebraica sea nicamente entre dos "R" del mismo signo. Veamos: (R = 4) se deduce de (R = 2) + (R = 2) como (-R = -4) se deduce de (-R = -2) + (-R= -2) (R = 6) se deduce de (R = 3) + (R = 3) como (-R = -6) se deduce de (-R = -3) + (-R = -3) (R = 8) se deduce de (R = 5) + (R = 3) como (-R = -8) se deduce de (-R = -5) + (-R =-3) Como vimos, "R primos" expresa los valores que asumen los N cuando se "colapsan" segn la frmula dN en la recta numrica cerrada (curva numrica cerrada si se prefiere, pues toda recta es en realidad una curva). De este modo las "consecuencias 4 y 5" enunciadas en este trabajo son verdaderas y por lo tanto, creemos, tambin la conjetura binaria de Goldbach. Actualmente estamos ocupados en agrupar nmeros primos dentro de la topografa propuesta en este trabajo, as como en evidenciar sus "distancias" relativas verticales y horizontales para intentar descubrir alguna regularidad en su aparicin dentro de la topografa mencionada. Adelantamos que hasta ahora los resultados nos hacen sospechar de la presencia de una simetra subyacente en la aparente disimetra con la que ocurre la aparicin de los nmeros primos.

1. ABBOT, Michael M. y Hendrick C. Vanness. TERMODINMICA. Ed. Mc. Graw Hill. Mxico 1991. 2. CSARMAN, Eduardo. ORDEN Y CAOS. El complejo Orden de la Naturaleza. 2 Ed. Ediciones Gernika. Mxico 1986.

3. CLAWSON, Calvin C.. MISTERIOS MATEMTICOS.zbrZ Ed. Diana. Mxico 1999. 4. DAZ, Jos Luis. EL BACO, LA LIRA Y LA ROSA. Las Regiones del Conocimiento. Ed. Fondo de Cultura Econmica. Mxico 1997. 5. DOXADIS, Apstolos. UNCLE PETROS AND GOLDBACHS CONJETURE. Ed. Faber & Faber. United Kingdom, Canada, Australia, New Zealand, South Africa. 2000. 6. GMEZ, Marn Edgar. ESTO ES EL CAOS. Ed. Consejo Nacional para la Cultura y las Artes. Mxico 1995. 7. HAYLES, N. Katherine. LA EVOLUCIN DEL CAOS. El Orden dentro del Desorden en las Ciencias Contemporneas. Ed. GEDISA, Barcelona 1998. 8. INTERNET http://www.edicionesb.es "EL TO PETROS Y LA CONJETURA DE GOLDBACH". 9. INTERNET. http://www.cs.unb.ca/~alopez-o/math-faq/node23.html Fermat, Pierre de - History of Fermats Last Theorem 10. INTERNET. www.faber.co.uk RULES OF THE GOLDBACHS CONJECTURE CHALLENGE (15.3.00). 11. RANC, Enrquez Clairette (editora de la revista Ciencia y Desarrollo). "Copia ciega" del dictamen del 26 de febrero del 2001 para este artculo.

La maldicin de Ssifo.
Una propuesta didctica para Comprender la Segunda Ley de la Termodinmica Prof. Mario Peral Manzo. U.P.N. Mxico. Ssifo, hijo de Eolo, de la estirpe de Deucalin, era el ms astuto de los mortales y el menos escrupuloso. Cuando Zeus hubo raptado a Egina, la hija del Asopo, al llevarla de Fliunte a Enone, pas por Corinto y fue visto por Ssifo. Cuando el Asopo se le present, en busca de la doncella, Ssifo le prometi revelarle el nombre del raptor a condicin de que el dios-ro hiciese brotar una fuente en la ciudadela de Corinto. Tras consentirlo el Asopo, Ssifo revel a Zeus como culpable, lo cual le vali la ira del seor de los dioses. Zeus lo fulmin y lo precipit en los Infiernos, condenndolo a empujar una roca enorme hasta lo alto de una pendiente. Tan pronto como la roca llegaba a la cumbre volva a caer, impedida por su propio peso y Ssifo tena que empezar de nuevo. Este castigo aparece ya contado en la Odisea, si bien existen episodios que justifican su castigo de modo distinto. ____________________________________

La maldicin de Ssifo. En su delicioso ensayo (Los Motivos de Ssifo) Csarman y Estaol apuntan:
"Para Albert Camus, el mito de Ssifo es smbolo de lo absurdo y trgico de la condicin humana. Condenado a un trabajo perenne, cuesta arriba, el hombre al final se da cuenta de la futilidad de su esfuerzo. A todo ascenso corresponde una cada. Toda la naturaleza parece estar irremisiblemente condenada a la obsolescencia (...). No otra cosa dice la segunda ley de la termodinmica: para realizar un trabajo se requiere invertir energa y parte de esta energa se desperdicia o disipa en 1 calor que ya no puede usarse para realizar ms trabajo"

La analoga que los mencionados autores establecen entre el trabajo/maldicin de Ssifo 2 y la segunda ley de la termodinmica 3, adems de interesante, nos hizo preguntarnos si dicha analoga sera pertinente para elaborar un modelo que pudiera servir a los alumnos del nivel de secundaria (tal vez tambin para nios de sexto de primaria) para comprender de manera ms prctica la mencionada ley. Nuestra postura al respecto es que s la es, razn por la cual hemos sealado como objetivo de este escrito el de demostrar la pertinencia de esta analoga. Replanteamos en boca de Ssifo el problema de su maldicin y lo hacemos negociar con los "jueces infernales" para

que (cuando menos) stos modifiquen las estipulaciones con el fin de que nuestro personaje pueda ensayar diversas situaciones que posiblemente le permitan terminar con su trabajo. Adelantamos que s hay una salida para Ssifo pero...

I. Lana Sube, Lana Baja... Ssifo, cansado de tanto subir y bajar con su piedra y haciendo gala de su fama de astuto, logra que los "jueces infernales" le concedan una audiencia para revisar su caso. El dilogo entre aqul y stos, se desarrolla de la siguiente manera: Ssifo: Estoy consciente de mi error y no pido concesiones; pero me gustara tener la oportunidad de poder tomar decisiones en lo referente a mi castigo. Juez "A": No podemos modificar tu castigo, tendrs que cumplirlo. Qu tipo de decisiones, pues, has de tomar en relacin con una sentencia inapelable? Ssifo: No me refiero al castigo en s, tan solo pido que, para cumplirlo de mejor manera y ms justa, se me permita (por as decirlo) modificar las "condiciones ambientales" que me han impuesto y ms libertad para buscar formas de cumplir con mi castigo. Ante las muestras de impaciencia de los jueces, Ssifo recurre al pizarrn de la sala de audiencias para explicar de mejor manera su peticin. Dibuja los siguientes grficos:

Ssifo (al terminar de dibujar): Como ustedes saben, mi castigo consiste en subir cuesta arriba una piedra y, justo cuando voy a alcanzar la cima, las fuerzas me abandonan, sta se me sale de control y rueda hacia abajo haciendo intil todo mi esfuerzo (como pueden ustedes ver en las dos primeras vietas). Si represento esto en la dimensin temporal (vean la ltima vieta) resulta que mi trabajo es equivalente a subir un nmero infinito de alturas; situacin que me parece injusta, puesto que se me conden a intentar llegar a la cima

de una sola cuesta. Esto, a mi parecer, se debe a que el ambiente al que ustedes me condenaron est "abierto". Yo lo que pido es que se me permita cumplir mi castigo en un ambiente "cerrado" y con un mximo de libertad permitido que no contradiga, claro, este castigo que me tengo merecido

A pesar de las falacias subyacentes en el discurso de nuestro personaje, los jueces se dejaron convencer por la elocuencia de ste. Para sorpresa de Ssifo, los jueces le pidieron que se retirara a seguir cumpliendo con su castigo en tanto deliberaban acerca de su peticin. Le dijeron que se presentara al siguiente da para que supiera del dictamen. Se cumpli el plazo y Ssifo acudi puntual a la cita. Juez "B": Despus de deliberar en relacin con la peticin que formulaste ayer, hemos decidido lo siguiente... Juez "C", haga el favor de leer nuestro dictamen. Juez "C": Garcias Juez "B". Pues bien, hemos decidido acceder a tu peticin bajo las siguientes bases: PRIMERA: DE LAS LIBERTADES QUE TENDR EL REO SSIFO. 1. Podr elegir la trayectoria que ms le convenga; ya sea que sta represente un polgono regular o irregular de "N" lados. 2. Podr determinar los "conteos" que desee a partir de "N + 2" desde que "N = 0". 3. Podr combinar, si as lo desea, los conteos que ms le convengan, por ejemplo: (N + 2) + (N + 6)... SEGUNDA: DE LAS RESTRICCIONES PARA EL REO SSIFO. 1. Los vrtices de los polgonos sern nombrados de manera sucesiva y en el sentido de las manecillas del reloj comenzando con las letras el alfabeto latino (de la "A" a la "Z") y si por el nmero de los vrtices del polgono elegido stas no bastaran, se recurrir a los diversos alfabetos pasados, presentes o futuros; por ejemplo el griego, el cirlico, el hebreo, etc., a discrecin del reo. 2. El "punto de partida" (el punto cero) siempre ser el vrtice "A" y el "punto destino" siempre ser el vrtice "B" para no contravenir la sentencia para el castigo original. 3. Siempre habr avance en un solo sentido: el de las manecillas del reloj. 4. Al final de cada conteo han de quedar expresadas todas y cada una de las letras correspondientes a los vrtices del polgono elegido y sin que alguna de ellas aparezca ms de una vez. 5. Al ser cumplidas las anteriores restricciones, cuando quede expresada la serie cuya letra final sea la "B" (el punto del vrtice destino) se considerar terminada la condena. Como ves -agreg dirigindose a Ssifo- ya no tendrs que subir cuesta alguna. Estas bases entran en vigor el da de hoy y terminar su vigencia el da en que sea cumplido tu trabajo

tal como lo indica la quinta restriccin de este documento-. Concluy su intervencin el juez "C" despus de dar el mazazo de rigor. Ssifo se retir contento de cumplir una condena que se le antojaba muy corta. Se haba salido con la suya (o al menos eso crea).

II. Manos a la Obra o "A la Piedra las manos" Armado de papel y lpiz para registrar su avance y la serie de letras que lo conduciran hacia la libertad, Ssifo se dispuso a realizar su tarea. Le pareci que sera una buena idea elaborar un cuadro de registro cuyas columnas permitieran registrar: el nmero de conteos, el polgono que representara la "trayectoria cerrada" los puntos de destino final de cada serie y, por ltimo, el nmero de traslaciones. As mismo decidi hacer uso de polgonos regulares a partir del tringulo y de conteos sencillos (sin combinar). Presentamos los resultados parciales de su avance.

Ssifo ha realizado, adems, sus conteos para los polgonos regulares de seis, siete, ocho y nueve lados.. Sus clculos estn en el fichero comprimido Cuaderno de notas.zip. Si deseas comprobar sus resultados con los tuyos puedes bajarte el fichero. Como puede observarse, en todos los casos el destino final de la serie es siempre "A", el "punto de partida" (o punto cero) con el que se inicia el conteo y, en estos casos, se reinicia de manera indefinida. De hecho, a lo largo de los conteos progresivos se van definiendo "periodos" que se reproducen tambin, a su vez, de manera "indefinida"; slo que, para llegar a la reproduccin de estos periodos, Ssifo tiene que realizar un mayor nmero de conteos y de traslaciones. Sospechamos que (an realizando combinaciones de conteos) nuestro amigo obtendr los mismos resultados hasta ahora observados en la tabla de arriba. Pero como "la esperanza muere al ltimo"...

III. La Trampa de los Jueces o "Ms Sabe el Diablo por Viejo..." Se nos antoja cuatro maneras que podran servir a Ssifo para terminar con su intil actividad:

1. Pedir a Zeus que interceda a favor del condenado para que lo perdonen. Pero... esto es muy peligroso; sabemos cun vengativos son los dioses griegos. No sea que los "jueces infernales" tengan reservadas por ah algunas piedras para nosotros. 2. Dar permiso a Ssifo para que realice "pasos dobles". Pero... vindolo bien es imposible pedirle a ste que d el segundo paso sin antes haber dado el primero. 3. Permitirle "movimientos reversibles". Sin embargo, siendo congruentes con la Segunda Ley de la Termodinmica (que es equivalente a ser congruente con los procesos de la naturaleza que esta ley describe) an en nuestro modelo no hay posibilidad para la reversibilidad de los procesos. En otras palabras: Ssifo podra terminar su trabajo si se le permitiera partir del punto "B", pero ste es el "punto destino" y, como sabemos, no se puede terminar lo que no se ha comenzado; desde esta perspectiva, nuestro personaje est condenado a realizar su pesado trabajo por toda la eternidad. Semat afirma:
"[...] podemos imaginar tres objetos A, B, y C, tales que A est en equilibrio trmico con B y

A tambin est en equilibrio trmico con C. Esta idea de que dos objetos separadamente en equilibrio trmico con un tercer objeto estarn en equilibrio trmico entre s, se llama ley cero de la termodinmica."

(y ms adelante)
"[...]En la operacin de una mquina de Carnot hay tres sistemas que tenemos que considerar: (la substancia activa, el depsito caliente y el depsito fro)" "[...] La entropa de la substancia activa no cambia en un ciclo: Sws = 0 "[...] El depsito caliente transfiere calor Q 1 reversiblemente a la temperatura T I, por lo que el cambio de entropa del depsito caliente S H se da por S H = Q 1/ T I "[...] El depsito fro absorbe el calor Q 2 a la temperatura T 2 por lo que el cambio de entropa del depsito fro S c se da por S c = Q 2/ T 2 "[...] El cambio de entropa del universo se da por la suma de S = S ws + S H + S c = Q 2/T 2 - Q 1/ T I "[...] De la ecuacin (e = 1- T 2/ T I) tenemos Q 1/ Q 2 = T I / T 2 o bien Q 1/ T I = Q 2 / T 2 e introduciendo esto en la ecuacin del cambio de entropa en el universo tenemos S = 0 "[...] En la operacin de la mquina de Carnot el cambio total de entropa del universo es 4 igual a cero."

Lo que nos lleva a la nica manera mediante la que Ssifo podr liberarse de la maldicin, como se ve lneas abajo...

4. Todo sistema que deseemos que se mantenga sin cambios, y en un estado de mxima eficiencia, ha de recibir de manera constante un suministro de energa; pero resulta que la mayor parte de la energa que se gasta en un trabajo, se desperdicia en forma de calor, sin posibilidad de ser reaprovechado por el sistema que lo gener.

De dnde obtienen los "jueces infernales" la energa para mantener la maldicin de Ssifo?: lo ignoramos. Lo que s sabemos es que Ssifo tiene una y solo una posibilidad de liberarse de esta absurda tarea. Ha de alcanzar el grado mximo de equilibrio trmico de que es capaz todo ser vivo: la muerte. Si a la luz de las leyes de la Fsica la vida es una broma, la muerte entonces es la madre de todas las bromas. "La vida es orden, organizacin, informacin, replicacin. La nocin de orden domina en medio de un universo de desorden".5 Tambin del mismo autor Sphirolandia: un lugar muy formal

NOTAS 1 CSARMAN, Eduardo y Bruno Estaol. LOS MOTIVOS DE SSIFO. Ed., Miguel ngel Porra, Mxico 1995, pp 5 y 6. 2 "Como se dio cuenta (Ssifo) del robo que hizo Zeus de la ninfa Egina y cont a su padre el hecho, el dios lo mand al Trtaro para que fuera castigado con duros tormentos... (Astutamente Ssifo orden a Merope, su esposa), que no quemara su cadver, como era usual. Y cuando lleg al Hades se fue a ver a Persefone (para que le permitiera regresar al mundo a cumplir con el rito funerario con la promesa de que regresara al tercer da. Como no cumpliera con su palabra lo hizo volver Hermes). Por tal incumplimiento los jueces infernales resolvieron que estuviera dando vueltas constantemente a fuerza de empujones a una enorme roca, hacindole rodar hacia la altura y que ella volva a caer y su obra interminable era la mayor fatiga." En GARIBAY, K. ngel Mara. MITOLOGA GRIEGA (Dioses y Hroes) Ed. Porra, Mxico 1996 pp 219 y220. 3 "(...) la segunda ley de la termodinmica establece la existencia de una propiedad comn a los cambios de estado de los procesos, conocida con el nombre de entropa, la cual puede ser considerada como la tendencia a la desaparicin de las diferencias de temperatura que es el estado ms probable entre todos los posibles. La entropa se incrementa con los cambios de estado que son irreversibles, y permanece constante en aquellos cambios que son reversibles. El incremento de la entropa no afecta la constancia cuantitativa de la energa, pero s influye en su capacidad para ejecutar trabajo". En DE GORTARI, Eli. DIALCTICA DE LA FSICA. Ed. Ocano, Mxico 1986 pp 108 y 109. 4 SEMAT, Henry y Phillip Baumel. FUNDAMENTOS DE FSICA. Ed. Interamericana. Mxico pp 208, 209 y 250. 5 SCHATZMAN, Evry. LOS NIOS DE URANIA. Ed. Salvat. Barcelona 1987 p 69.

Tringulos rectngulos. cuyos catetos, enteros, se diferencian en la unidad y tienen la


hipotenusa entera. Francisco Javier Asencor

Resulta un tpico, ilustrar el teorema de Pitgoras mediante el tringulo de tres y cuatro unidades en los catetos y cinco en la hipotenusa. Sin duda es la terna de nmeros ms simples que cumplen la relacin de Pitgoras, salvo la trivial (0,1;1) que, en definitiva, no permite ver un tringulo. Existen otras ternas que cumplan esta relacin? Desde luego que s, pero cuntas? y qu relaciones mantienen entre ellas? No es un gran problema pero se presta al entretenimiento si se est abierto a dejarse sorprender por algunos aspectos que aparecen. Todo puede hacerse con un nivel de algebra simple (el cuadrado del binomio es lo ms complicado. Si puede uno pasar el rato haciendo un crucigrama, tambin con esto ...) En principio puede armarse de una calculadora y ensayar (o con un sencillisimo programa hacer que ensaye el ordenador). Se puede encontrar que las ternas (0,1;1) (3,4;5) (5,12;13) (8,15;17) (20,21;29) (12,35;37) (9,40;41) (28,45;53) ... y muchas ms, cumplen la condicin. Por supuesto las obtenidas como producto de estos valores por cualquier factor entero tambin cumplen, pero carecen de inters. Consideramos slo aquellas cuyo mximo comn divisor sea la unidad. Desde luego resulta ms interesante buscar una forma general que nos proporcione todas las ternas. (En rigor no se demuestra que todas las ternas vlidas queden recogidas con este mtodo ...)

Consideremos dos enteros nones cualesquiera, primos entre s Las expresiones

Demostracin Si n 1 y n 2 son primos entre s, C 1 y C 2 son primos entre s y en consecuencia lo son con H. Ya que si C 1 y C 2 admiten como factor comn el primo p, este ha de ser factor de n 1 o n 2; admitamos que lo es de n 1, de esta forma C1 = p m1 n2 y

proporcionan nmeros enteros, que que slo es entero si n 2 es mltiplo de p es cumplen la relacin que nos interesa y decir, n 2 no es primo con n 1 cuyo m.c.d. es uno, como es fcil de demostrar. Los valores n 1 = 1 y n 2 = 3 proporcionan la terna (3,4;5).

Tenemos un conjunto doblemente infinito de ternas fcilmente clasificable por su origen por ejemplo con n 1 = 1 cada uno de los impares mayores distintos de uno nos proporciona un conjunto en el que la hipotenusa es mayor que el cateto mayor en una unidad (|H - C 2| = 1)... Pueden buscarse multitud de subconjuntos, con slo imponer alguna condicin sobre los nmeros generadores (n 1, n 2) o entre los valores de las ternas. Analizamos el caso en que la diferencia entre ambos catetos sea la unidad: |C 1 - C 2| = 1

Los casos de signo negativo de la raz carecen de inters pues slo resulta compatible el valor n 1 = 1 con la opcin negativa en el interior de la raz, y supone la anulacin de esta. As requerimos encontrar los valores n 1, que aplicados a esta expresin nos proporciones un n 2 entero, primo con n 1 Hagamos algunas transformaciones. Sea n 1, un valor que cumple la condicin

El valor obtenido para n 2, cumple las mismas condiciones y proporciona otro nmero

entero. Esto constituye una ley de recurrencia. As en la sucesin cuyos primeros trminos son: 1, 1, 3, 7, 17, 41, 99, 239, 577, 1393, 3363, 8119,... tomando dos de ellos consecutivos cualesquiera, generaremos una terna que nos indique el valor de los catetos enteros diferenciados en una unidad que determinan hipotenusa entera. * Es posible establecer una forma general no recursiva para los trminos de esta sucesin, pero requiere un clculo algo superior y no es relevante para lo que presentamos. Valores de las n Llamando

La obtencin de estos enteros a partir de irracionales, no presenta ninguna particularidad. En el desarrollo binomial (aqu aparece el triangulo de Tartaglia) de las potencias de a se alternan trminos racionales (potencias pares de la raiz), con irracionales. Estos se anulan dos a dos en la suma de los dos desarrollos. *

Los catetos obtenibles y las hipotenusas constituyen de por sm otras sucesiones de enteros. Fijmonos, por ejemplo en las hipotenusas

Ambas relaciones "cuelgan" la sucesin de hipotenusas de la sucesin de nmeros generadores, y resulta "desagradable" la presencia de elementos no lineales. Con un poco de algebra muy sencilla podemos tratar de obtener una ley de recurrencia "autoconsistente" para las hipotenusas. Si tenemos en cuenta la primera relacin entre las n

que podemos elevar al cuadrado y anular el segundo miembro

y sumarlas agregando este cero a la expresin de la hipotenusa

que puede reescribirse como

que es lo que nos proponamos una ley de recurrencia para las hipotenusas: con los trminos H 0 = 1 y H 1 = 5, queda determinada la sucesin cuyos primeros elementos son: 1, 5, 29, 169, 985, 5741, 33461,... * Tambien es posible establecer una forma general no recursiva para los trminos de esta sucesin, pero requiere un clculo algo superior y no es relevante para lo que presentamos Valores de las H Manteniendo el valor de a

que est relacionado con el anterior.... Los exponentes son nones. * Para terminar una observacin para consideraciones posteriores, que ahora no haremos. Si con la ley de recurrencia obtenida cambiamos el segundo trmino,

obtenemos otra sucesin simptica, (1, 6, 35, 204, 1189, 6930, 40391,...). * Que tambin tiene su forma general Valores de las G

en que los exponentes son pares. * Los cuadrados de sus elementos son a la vez nmeros triangulares. * Nmeros triangulares Si denominamos nzmeros cuadrados [z 2] al nmero de objetos que permiten ser colocados

en z filas de z objetos cada una, formando un cuadro, podemos llamar triangulares [z(z+1)/2] al nmero de objetos que permiten ser colocados en z filas de modo que la primera tenga z objetos y las dems, sucesivamente, uno menos que la anterior cada una, formando un tringulo. Estos nmeros aparecen en el tringulo de Tartaglia en la segunda lnea paralela a cualquiera de las filas de 1...
1 1 1 1 1 1 ... 1 2 3 4 5 ... 1 3 6 10 ... 1 4 10 ... 1 5 ... 1 ... ..

El primero (el 1) es un cuadrado, hay otros en la lmnea y vienen recogidos en la sucesisn que hemos llamado G * Como no creemos en las casualidades cuando se trata de relaciones numricas, dejamos la relacin abierta... pero cuidado las sucesiones enganchan. Plimpton 322. El teorema de Pitgoras es, sin duda, el teorema ms popular de toda la matemtica. Ya se conoca desde tiempo de los babilonios y aparece por primera vez impreso en la tablilla (aprox. 1900-1600 a.C.) denominada Plimpton 322 (por tener ese nmero de la coleccin del mismo nombre) que se encuentra en la Columbia University Library (N.Y.). En ella, aparecen cuatro columnas de nmeros entre las que se desprende un aceptable dominio de las ternas pitagricas. La tabla fu descifrada por Neugebauer y Sachs (Mathematical Cuneiform Texts -1945-) y ah estn las 6 primeras filas

Los nmeros de la columna primera, tercera y cuarta (con fondo amarillo) son ternas pitagricas. Parece que los babilonios llegaron a calcular estos valores segn un elaborado procedimiento algebraico, hecho que no es en absoluto descartable. Pastor y Babini, refirindose a los pitagricos, dicen: [...aunque en el estudio de los tripletes no lograron la generalidad de los babilonios] A partir de la expresin a2 + b2 = c2 dividiendo ambos miembros por b 2 resulta:

y haciendo el cambio de variable Haciendo el cambio de variable

tenemos u 2 + 1 = v 2 expresin equivalente a (v - u)(v + u) = 1

obtenemos

De esta forma podemos obtener ternas pitagricas sin ms que dar valores a x e y

Adems de las tres columnas con las ternas pitagricas, aparece una 120 119 169 1,9834028 45 14 23.038 12 5 cuarta columna que es la relacin, al 3456 3367 4825 1,9491586 45 44 50.389 64 27 cuadrado, que existe entre la 4800 4601 6649 1,9188021 46 12 45.553 75 32 hipotenusa y uno de los catetos. b a c (c/b) 2 x y 13500 12709 18541 1,8862479 46 43 43.28 125 54 72 360 960 600 60 240 90 65 319 799 481 45 161 56 97 481 1,8150077 47 55 29.921 9 4 1,7851929 47 39 53.962 20 9 --- ----- ---

2700 2291 3541 1,7199837 --1249 1,6927094 --769 75 289 106 1,6426694 --1,562500 1,4500174 1,3871605

6480 4961 8161 1,5861226 2400 1679 2929 1,4894168 2700 1771 3229 1,4302388

--- --- De esta forma podan conocer los ----- --- ngulos de los tringulos rectgulos considerados. Podemos observar que la ----- --- tabla parte de un ngulo de ----- --- aproximadamente 45 y va aumentando hasta aproximadamente 60. Sobre ----- --fondo gris estn los valores de y los 56 44 17.133 50 27 de x e y para calcular los lados del 58 6 33.15 9 5 tringulo (En la tabla slo aparecen los valores sobre fondo amarillo)

No es probable que los babilonios conocieran estas relaciones trigonomtricas, pero pudieron llegar a dicho resultado a partir de los valores de x e y teniendo en cuenta que

Nota Los valores que aparecen en la tabla marcados por --- quedan a cargo del lector interesado Sellos y Matemticas. El teorema de Pitgoras.

El Ao 2000 fu prdigo en acontecimientos dedicados a las matemticas. El correo postal nicaragense puso en circulacin una serie de 10 valores dedicado a Las 10 frmulas que cambiaron la faz de la Tierra. Este valor de 30 centavos est dedicado a la LEY DE PITGORAS El segundo valor, de gran belleza matemtica, fu publicado por el servicio postal de la Repblica Checa. Dedicado a Fermat y a Andrew Wiles. Sobre el smbolo de la igualdad aparece la desigualdad en la que figura la inscripcin Andrew Wiles 1995. Dicha igualdad slo tiene soluciones enteras cuando n = 2 es decir, el teorema de Pitgora.

Sello de San Marino de 1000 Liras. En la parte superior puede apreciarse la interpretacin geomtrica de dicho teorema. El segundo valor pertence al estado de Israel. Conmemora el Da de la Educacin y en l figuran varias expresiones matemticas, entre ellas el teorema de Pitgoras.

En 1955 Grecia puso en circulacin este serie de cuatro valores. Dos estatuas de valores 2 y 5 dracmas representando a Pitgoras, la isla de Samos y la interpretacin intuitiva del teorema 25 cuadrados = 16 cuadrados + 9 cuadrados es decir 5 2 = 3 2 + 4 2 Si desea conocer ms cosas sobre este apasionante teorema visita El teorema de Pitgoras.

El teorema de Pitgoras. Euclides I, 47 Euclides, en el Libro I de los Elementos proposicin 47 demuestra el teorema de Pitgoras: En los tringulos rectngulos el cuadrado sobre el ngulo opuesto al ngulo recto es equivalente a los cuadrados sobre los lados que forman el ngulo recto. Prueba que el rea del cuadrado NMBC es igual a la suma de las reas de los cuadrados ABPQ y CAED. Para ello, trazamos por A una perpendicular a CB hasta que corte a NM en A y que divide al cuadrado NMBC en dos rectngulos AMBA y NAAC. A continuacin unimos A con M y C con P. Los tringulos MBA y CBP son iguales pues tienen el mismo ngulo B = 90 + t e iguales los lados que lo determina (BP = AB y BM = BC) Se verifica: [rea tringulo MBA] = 1/2 MB.MA = 1/2 (MB.MA) = = 1/2 [rea rectngulo AMBA] Por otra parte: [rea tringulo CBP] = 1/2 BP.QP = 1/2 (BP.QP) = = 1/2 [rea cuadrado BPQA] Por tanto: [rea tringulo MBA] = [rea tringulo BPC] = = 1/2 [rea cuadrado BPQA] = 1/2 [rea rectngulo AMBA] Es decir el cuadrado BPQA y el rectngulo AMBA son equivalentes. Anlogamente demuestra que el

Manuscrito rabe del s.XIII

rectngulo NAAC es equivalente al cuadrado CAED.

En la proposicin 48 del Libro I de los Elementos, Euclides demuestra que Si el cuadrado construido sobre uno de los lados de un tringulo es equivalente a los cuadrados, juntos, de los otros dos lados, el ngulo formado por esos dos lados es recto es decir el recproco de la Proposicin 47. (Esta es la demostracin que hace Euclides en los Elementos, aunque se han adoptado algunas notaciones actualizadas). Sea el tringulo ABC y supongamos a 2 = b 2 + c 2 Tracemos por A una perpendicular a AC y sobre ella tomamos AD igual a AB. Unamos D con C. Como DA = AB = c tambin lo sern sus cuadrados, es decir DA 2 = AB 2 = c 2 2 Si sumamos b , tendremos DA 2 + b 2 = c 2 + b 2 Pero m 2 = DA 2 + b 2 (pues DAC es recto; p47) y a2 = b2 + c2 (por hiptesis), luego el cuadrado sobre el lado DC (es decir m 2) es equivalente al cuadrado sobre BC (es decir a 2), por lo que el lado DC ser igual al lado BC. Puesto que DA es igual a AB y AC es comn DA y y AC sern iguales a BA y AC y la base DC igual a BC por lo que el ngulo DAC ser igual a BAC, y como DAC es recto, el BAC tambin es recto.

OTRAS DEMOSTRACIONES, COMPROBACIONES, COMENTARIOS, etc. RELACIONADOS CON EL TEOREMA DE PITGORAS. del griego : fijar, sujetar fuertemente una cosa a otra. (cateto) perpendicular, lnea que cae a plomo.

Apunte (#1). Facilitada por Jos Carrin. Demostracin atribuida a Leonardo da Vinci. Siguiendo la construccin resulta que el trigulo ABC es simtrico del tringulo ABC respecto al punto O centro del cuadrado mayor. Por otra parte, los cuadrilteros ABCA y ABCA son congruentes (tienen el mismo aspecto y rea) La figura formada por los dos cuadrados menores, el tringulo ABC y su simtrico ABC tiene un eje de simetra DE y se compone de dos cuadrilteros iguales DBCE y DECB Si se gira DBCE un ngulo de 90 a la derecha con centro en B y se hace coincidir con ABCA, los hexgonos BCECBD y ABCABC son equivalentes. Si restamos de ambos los dos tringulos que forman parte de ellos, obtenemos el teorema de Pitgoras.

Apunte (#2).

Una de las demostraciones ms antiguas e intuitivas sobre el teorema de Pitgoras es la siguiente, que puede seguirse fcilmente a partir de la construccin grfica que se muestra Partimos del tringulo rectngulo R cuya rea es 1/2 a b. A continuacin construimos un cuadrado cuyo proceso se describe en el grfico anterior El lado del cuadrado as obtenido es a + b y su rea (a + b) 2. Dicho cuadrado consta de cuatro tringulos rectngulos cuya rea es 4 ( 1/2 a b) = 2 a b y un cuadrado interior de lado c y rea c 2. Igualando ambas reas tendremos: (a + b) 2 = c 2 + 2 a b de donde a 2 + b 2 = c 2.

Apunte (#3). Generalizacin del teorema de Pitgoras. Para los semicrculos de la figura, a partir de la expresin c 2 = a 2 + b 2 multiplicando ambos miembros por resulta

Si las superficies S, S y S son semejantes, entonces rea (S) = rea (S) + rea (S)

de donde rea (Semicrculo S) = rea (SemicrculoS) + rea (SemicrculoS)

Apunte (#4). Relaciones mtricas en el tringulo rectngulo.

El tringulo ABC es rectngulo en C.

Teorema del cateto. En el tringulo ADC

En el tringulo BCA

Multiplicando miembro a miembro ambas expresiones

Teorema de la altura. En los tringulos rectngulos ADC y DBC resulta: h = m.tag(A) h = n.tag(B) Multiplicando miembro a miembro ambas expresiones h 2 = m.n.tag(A).tag(B) = m.n (pues tag(A).tag(B) = tag(A).tag(90 - A) = 1). Es decir: "En todo tringulo rectngulo la altura relativa a la hipotenusa es media proporcional entre los dos segmentos que determina sobre ella"

Es decir: "En todo tringulo rectngulo un cateto es media proporcional entre la hipotenusa y su proyeccin sobre ella" Teorema de Pitgoras. Aplicando el teorema del coseno a cada uno de los catetos del tringulo ABC y sumando resulta: a 2 = c.n b 2 = c.m

a 2 + b 2 = c.n + c.m = c (n + m) = >c 2

Apunte (#5). En T1 resulta sen (x) = b/c y en T4 sen (x) = PN /a de donde PN = ab/c y por construccin TV = PN = ab/c Por otra parte, en T4 cos (x) = MN/a y en T1 cos (x) = a/c de donde MN = a 2/c. Adems en T3 sen (x) = VM/b = b/c de donde VM = b 2 /c
A partir del tringulo rectngulo MPT trazamos por M una paralela a la hipotenusa (PT) y por P y T perependicualres a dicha paralela de forma que se determinan los tringulos rectngulos MNP y VMT. Tambin construimos sobre la hipotenusa el tringulo rectngulo PST.

(A partir de estos datos podemos comprobar que A T2 = A T3 + A T4 pues los tringulos T2, T3 y T4 son semejantes; ver Apunte (#3)). La figura as construida podemos mirarla de dos formas: formada por el rectngulo VNPT y el tringulo rectngulo T2 o bien por el rectngulo MPST y los tringulos T3 y T4. Evidentemente: rea (VNPT) + rea (T2) = rea (MPST) + rea (T3) + rea (T4) Calculando el rea de cada una de estas composiciones, identificando y simplificando las expresin obtenida obtendremos el Teorema de Pitgoras.

Apunte (#6). Diseccin de Perigal En Wennington (Essex) est la abandonada tumba del matemtico ingls Henry Perigal (1801/1898). En ella puede adivinarse la inscripcin: "[...] estudioso e ingenioso geometrista. Investig y enunci las leyes del movimiento circular compuesto. Querido y admirado por un gran nmero de parientes y amigos" Se le atribuye una ingeniosa comprobacin del teorema de Pitgoras. Sobre el mayor de los cuadrados construidos sobre los catetos se determina el centro (no necesariamente ha de ser este punto) y se trazan dos rectas paralela y perpendicular a la hipotenusa del tringulo. Con las cuatro piezas obtenidas ms el cuadrado construido sobre el otro cateto podemos cubrir el cuadrado contruido sobre la hipotenusa.

Apunte (#7). Una demostracin del teorema de Pitgoras atribuida a J. A. Garfield (vigsimo Presidente de los EEUU) Uniendo los puntos M y N obtenemos un trapecio cuya rea es: (a + b)/2 . (a + b) = a 2/2 + b 2/2 + a.b Por otra parte, dicha rea es la suma de los tres tringulos rectgulos que lo determinan. Sumado dichas reas: (a.b)/2 + (a.b)/2 + c 2/2 = a.b + c 2/2 Igualando ambas expresiones y simplificando obtenemos que a 2/2 + b 2/2 = c 2/2 y simplificando resulta el teorema de Pitgoras.

Apunte (#8). Con centro en O trazamos una semicircunferencia de radio c; consideramos un punto P y a partir de l construimos el tringulo rectngulo de lados a, b y c. (Primera figura) A continuacin construimos el tringulo MNP. Dicho tringulo es rectngulo y en l la altura relativa a la hipotenusa es a. Dicha altura determina sobre la hipotenusa los segmentos c + b y c - b. Aplicando el teorema de la altura (Apunte #4) resulta: a 2 = (c + b).(c - b) = c 2 - b 2 2 es decir c = a 2 + b 2

Apunte (#9). El Teorema de Pitgoras en el espacio. Facilitada por Jos Carrin

D2 = c2 + d2 d2 = a2 + b2 D2 = a2 + b2 + c2

Apunte (#10). Expresin vectorial del teorema de Pitgoras (En lo que sigue designaremos en negrita las magnitudes vectoriales y las operaciones efectuadas respecto a un sistema de referencia ortonormal) Dados dos vectores x e y la condicin necesaria y suficiente para que dichos vectores sean ortogonales es que || x + y || 2 = || x || 2 + || y || 2 siendo la norma del vector x. De dicha definicin de la norma resulta que || x || 2 = x.x (es decir, la norma al cuadrado de un vector es el producto escalar del vector por l mismo). Demostracin || x + y || 2 = (x + y) (x + y) = x . x + y . y + 2 x . y = || x || 2 + || y || 2 + 2 x .y Luego

Apunte (#11). Comparando cuadrados A partir del tringulo rectngulo inicial de hipotenusa c y catetos a y b, construimos los cuadrados de lados a + b tal como aparecen en la figura primera. El primer cuadrado est formado por cuatro tringulos iguales (T1, T2, T3, T4) y por un cuadrado de lado c, por lo que su rea es

c 2 + 4 A(T) siendo A(T) el rea de uno cualquiera de los tringulos. El segundo cuadrado est formado por dos cuadrados de lados a y b y los tringulos T1, T2, T3 y T4 y su rea es a 2 + b 2 + 4 A(T) Igualando ambas expresiones y simplificando obtenemos que c2 = a2 + b2

Esta es una de las ms intuitivas demostraciones del teorema de Pitgoras y, posiblemente, una de las que utilizaran los pitagricos.

Apunte (#12) Rompecabezas. Si trazamos en cada uno de los cuadrados construidos sobre los catetos una diagonal y una paralela a la hipotenusa c del tringulo rectngulo obtenemos ocho tringulos con los que podemos recubrir el cuadrado construido sobre la hipotenusa. Expresemos las reas de cada uno de estos tringulos en funcin de los lados a, b y c del tringulo rectngulo. Por definicin sen () = b/c; cos () = a/c Por otra parte, debido a la igualdad de los tringulos obtenidos bastar calcular las reas de, por ejemplo, los tringulos 1, 2, 5 y 6. Aplicando el teorema del seno al tringulo 1 resulta

de donde

Desarrollando sen (135 - ) obtenemos sen (135 - ) = sen (135) cos () - sen () cos (135) = sen (45) cos () + sen () cos (45) es decir

Sustituyendo en el valor de x obtenido anteriormente

y el rea del tringulo 1 es

El rea del tringulo 2 es

Analogamente resulta:

Puede comprobarse que

a=b

Apunte (#13) Kou ku. Uno de los primeros libros chinos dedicados a la matemtica y la astronoma es el Chou Pei (aprox. 300 a.C.). En l se hece referencia al teorema de Pitgoras (kou ku) mediante una comprobacin. A partir de la figura, un cuadrado de lado 7, si retiramos los cuatro tringulos de las esquinas (dos rectngulos de rea total 2 (3 4) = 24 unidades cuadradas) queda un cuadrado de lado 25 unidades cuadradas. Por lo tanto su lado es 5. Entonces (3 + 4) 2 - 2 (3 4) = 3 2 + 4 2 = 5 2 Durante el siglo III esta comprobacin fu fundamentada por varios matemticos chinos. Una de ellas es la siguiente. Si el lado ms corto (kou) es a, el ms largo (ku) es b y la diagonal (shian) es c resulta: c 2 = rea(GHEF) = rea(LIJK) + 2 (a b) = = rea(LIJK) + rea(GLFD) + rea(KECF) = = rea(APLG) + rea(PBEK) = a 2 + b 2

Apunte (#14) Tringulos rectngulos en la proporcin urea Supongamos un tringulo rectngulo ABC y a partir de l construimos otro tringulo rectngulo tomando como hipotenusa el cateto mayor del dado y como cateto mayor el cateto menor del tringulo rectngulo considerado. Impongamos la condicin de que ambos tengan los mismos ngulos. Entonces ambos tringulos rectngulos sern semejantes y se verificar

Determinando R Puesto que el tringulo 2 ha de ser rectngulo a2 = c2 + p2 y de (#1)

Mediante el cambio de variable R 2 = t resulta t 2 - t - 1 = 0 cuyas soluciones son Entonces siendo el nmero ureo.

Por la naturaleza del problema consideramos Supongamos b = k 0 > 0. De (#1) tendremos

de donde

Los tringulos de la forma

son tringulos rectngulos. Valor del gulo En el tringulo 1

independientemente del valor de b = k 0 Nuevamente de (#1) resulta por lo que

Tambin los trigulos de la forma

son rectngulos, tienen los mismos ngulos que los anteriores y estn con ellos en la proporcin 1/R. Si k 0 = los tringulos rectngulos y

son semejantes y estn en la proporcin 1/R.

Una apostilla de Fco. Javier Asencor En el Apunte #14 se hace un estudio interesante sobre el tringulo rectngulo cuyos lados toman los valores donde representa el nmero ureo. Donde aparece el nmero ureo siempre hay sorpresas en forma de relaciones con cierta belleza. Es un nmero que por s solo puede sostener una publicacin. Un ejemplo es este tringulo. Puede agregarse alguna consideracin que aumenta la fascinacin de este tringulo (o sus semejantes). Si multiplicamos los lados por , podemos escribir la terna como

Una forma en que los exponentes nos recuerdan definitivamente a la ms famosa terna de los tringulos rectngulos, (3, 4, 5) Esto admite la siguiente consideracin: Si nos proponemos encontrar el tringulo rectngulo cuyo cateto mayor sea la media aritmtica entre la hipotenusa y el otro cateto encontramos el familiar (3, 4, 5) o semejantes. Si el propsito es idntico, pero con la media geomtrica, encontramos ste. Una aparicin geomtrica: Sobre el primer cuadrante de unas coordenadas cartesianas trazamos el semicrculo de radio unidad y centro en eje Y distante la unidad del centro. Por supuesto resulta tangente al eje X. Este semicirculo admite infinitas tangentes, de las cuales las que se tracen "por arriba" es decir con pendiente negativa se cortan con ambos ejes. La longitud del segmento determinado por estos cortes dependen del punto de tangencia. Pueden tener cualquier longitud por encima de cierto valor, es decir existe una que es mnima. El tringulo formado por el origen de coordenadas y los cortes en los ejes, forman un tringulo rectngulo que se corresponde exactamente con la terna propuesta arriba. El nmero aparece tambin en las coordenadas del punto de tangencia.... El triangulo superior, por encima de la ordenada se corresponde con la primera terna aqu mencionada: ... y si seguimos mirando: ms cosas

El matemtico indio Bhaskara Apunte (#15) Bhaskara (1114-1185) hizo una reconstruccin del teorema de Pitgoras a la que slo le aadi la palabra MIRA! de forma que a partir de la observacin de la figura se pudiera reconstruir el teorema. Esta reconstruccin aparece en su obra VijaGanita (en la que por primera vez aparece la divisin de un nmero (distinto de 0) por cero. Su obra Lilavati contiene varios problemas relacionados con el teorema de Pitgoras como los siguientes: El bamb roto Si un bamb de 32 codos de altura ha sido roto por el viento de tal manera que su extremo superior queda apoyado en el suelo a una

distancia de 16 codos de su base, a qu altura del suelo se rompi? El pavo real y la culebra Un paco real se encuentra posado en el extremo de un poste vertical en cuya base hay un agujero de culebra; observando la culebra a una distancia del pie del poste igual a tres veces su altura, el pavo real se lanza sobre ella en linea recta mientras la culebra intenta ganar su agujero. Si el paco real captura a la culebra cuando ambos han recorrido exactamente la misma distancia, a cuntos codos de distancia del agujero se produjo la captura? Las abejas y las matemticas.
Jos Carrin Beltran.

Analicemos, brevemente, el comportamiento de algunos polgonos. Con tringulos equilteros, cuadrados y hexgonos regulares pondemos enlosar una superficie.

Las abejas ..., en virtud de una cierta intuicin geomtrica ..., saben que el hexgono es mayor que el cuadrado y que el tringulo, y que podr contener ms miel con el mismo gasto de material.

Pappus de Alejandra

Tringulo Lado = 4 u Permetro = 12 u rea = 6,928 u 2 Cuadrado Lado = 3 u Permetro = 12 u rea = 9 u 2 Hexgono Lado = 2 u Permetro = 12 u rea = 10,392 u 2

Las abejas construyen sus panales como prismas hexagonales regulares apuntados en el fondo por tres rombos inclinados respecto a la horizontal un ngulo determinado para que, almacenando la misma cantidad de miel, tengan la Con el mismo mnima cantidad de materia (cera); es decir, el permetro, la mayor superficie se recubre con rea sea mnima. un hexgono Este problema de las abejas ya admir a los clsicos y fue estudiado por importantes matemticos, entre otros Colin McLaurin (16981746) y Gabriel Cramer (1704-1752) obteniendo para dicha inclinacin valores de 70 32 y 70 31 respectivamente.

Observando la figura vemos que la abeja construye el rombo GBHF de modo que el volumen que quita del prisma, el GABF, equivale al que aade, el HBJF. Pero aunque el volumen del panal equivale al del prisma hexagonal, sin embargo el rea total del panal es la menor posible para tal propsito; si la abeja hubiese dado al panal la forma de prisma, ste no habra perdido capacidad, pero habra sido necesaria ms cera para su construccin. En la naturaleza rige la ley del mnimo/mximo. Vamos a calcular el ngulo x de inclinacin del rombo que hace mnima dicha rea. Identidades AB = a arista bsica del prisma, que por tratarse de un problema afn se puede sustituir por la unidad de longitud; luego lo haremos) x = HKJ ngulo de inclinacin que deseamos determinar KJ = a/2 apotema del tringulo equiltero BFD HJ = KJ.tang(x) = (a/2).tang(x) = AG por simetra HG = 2.HK =a.sec(x) HJ = KJ.tang(x) = (a/2).tang(x) = AG por simetra lado del tringulo BDF HK = KJ.sec(x) = (a/2).sec(x)

El rea que estudiamos ser mnima cuando sea mxima la diferencia y = (ABF) + (BFJ) + (ABG) + (AFG) - (GBHF) Tendremos: Haciendo a = 1 se tiene:

derivando

Igualando a 0 y aplicando el criterio de la derivada segunda se obtiene finalmente

El doble de dicho ngulo es 70 31 43.606"

Veamos otro camino para la resolucin del problema Imaginemos las colmenas compuestas por tres listones romboidales como indica la figura de la izquierda y consideremos uno de estos listones, con el que vamos a trabajar, y que se presenta en la Fig.2. Supongamos, para simplificar, que el lado del hexgono es 1. Es evidente que al mover el rombo sobre la diagonal AB (fig.2) el volumen del liston romboidal no vara, pues el volumen que se aumenta al desplazarse v hacia arriba (extremo superior diagonal azul) queda compensado al desplazarse m (extremo inferior diagonal azul) en sentido contrario. Por tanto, el volumen permanece invariable al realizar esta operacin. Supongamos, por tanto la diagona AB fija. Como el lado del hexgono hemos admitido que vale 1, el valor de dicha diagonal es 3 1/2. Deseamos calcular el ngulo alfa de forma que la superficie sea mnima.

Fig.1

Fig.2

Fig.3

Nota En las Fig.2 y Fig.3 la planta est dibujada completa, pero para mayor claridad en el alzado slo se ha dibujado el listn romboidal considerado

Sea L la longitud de la diagonal (diagonal azul) y x la proyeccin de la misma (Fig.3). Podemos establecer la siguientes igualdad L 2 = (2x) 2 + 1 (Fig.4) La superficie lateral de la clula est formada por seis trapecios. El rea de uno de ellos es (Fig.5)

(Fig.4) y por tres rombos, siendo el rea de uno de ellos por lo que la superficie de la clula es A T = 6.A 1 + 6.A 2. Efectuando operaciones y sustituyendo los valores obtenidos anteriormente resulta

Derivando e igualando a 0 tendremos

y de ah obtenemos x 2 = 0.125. LLevando este valor a L, (fig.5) tendremos Relacionando, por ltimo, ambas diagonales.

de donde 31 43.606

por lo que el valor de alfa resulta 70

Sphirolandia: un lugar muy formal


Prof. Mario Peral Manzo.

En una regin muy prxima a la imaginacin que hace frontera con la realidad, se encuentra un hermoso lugar matemtico en donde viven eternamente un infinito nmero de individuos llamados "sphiros", seres esfricos de idnticas dimensiones y con una vocacin indomable por agruparse. Este lugar es un medio que est muy reglamentado con el fin de mantener un constante equilibrio entre los individuos que lo habitan y asegurar su existencia "por siempre y para siempre jams". Sphirolandia, pues, se basa en las siguientes REGLAS 1. Los "sphiros" pueden permanecer aislados si as los desean. (Por supuesto no lo desean; pues simplemente no pueden evitar querer agruparse) 2. Cuando estnn aislados y buscan reunirse, solamente pueden agruparse en cadenas; jams podrn formar conglomerados o amontonamientos que contravengan la tercera regla (ver siguiente regla). 3. Toda vez que los "sphiros" formen cadenas (filas) a. No les es permitido agruparse con un idntico nmero de elementos a cualquier cadena ya existente. Es decir, no debe haber ms de una cadena con idntico nmero de elementos. Cuando por accidente (o por el caso advertido en la quinta regla; expresada lneas ms abajo) esto suceda, las cadenas con idntico nmero de elementos se unirn de acuerdo con lo previsto en esta ltima regla mencionada. b. Pueden unirse a otra cadena, alineando todos y cada uno de sus elementos para formar hileras y siempre comenzando este alineamiento por uno de los extremos de la cadena. (Convengamos en llamar filas a los elementos ordenados de manera horizontal e hileras a los ordenados de manera vertical).

c. Si lo desean pueden permanecer as unidos de manera indefinida pero, como ya lo sabemos, no lo desean; pues sencillamente no pueden evitar querer seguir agrupndose. 4. Toda vez que los inquietos y gregarios "sphiros" hayan alineado sus filas en hileras, podrn unirse a otros conjuntos de "sphiros" ordenados del mismo modo que se menciona en la tercera regla.

5. Toda vez que se junten cuatro filas de "sphiros", y muy a su pesar, se separarn en hileras, y cada una de las hileras cuyo nmero de "sphiros" sea idntico, se unirn de acuerdo con la tercer regla.

6. Los "sphiros" que formen hileras de un solo elemento (suena extrao esto de "hileras de un nico elemento" pero abusemos un poco del lenguaje para una mayor claridad) necesariamente se separarn unos de otros durante el proceso descrito en la quinta regla aunque, posteriormente al restablecimiento de la legalidad y, dada su terquedad, no se les prohibe que decidan volver a unirse como lo estaban poco antes de la desintegracisn o como se les di su muy regalada gana sin violar la segunda regla. 7. Las filas producidas durante la desintegracisn de las hileras (descrita en la quinta regla) reiniciarn el proceso indefinidamente.

Problemas Como en el ambiente que hemos creado hay un infinito nmero de "sphiros" en interaccin, suponemos que hay un infinito nmero de procesos en los que interactan "sphiros" solos o en cadena y esto garantiza la eternidad de stos y sus interacciones. Pero si nos limitamos a un conjunto finito como el ejemplificado en la quinta regla, habr un retorno al lmite que es el origen para estas interacciones o continuarn indefinidamente?. Y si continan indefinidamente ser porque el proceso se convierte en un ciclo peridico o un proceso cclico con infinidad de posibles historias? Qu otras variantes sern posibles sin violar las reglar enunciadas? Para quienes no les da la gana quebrarse la cabeza con los anteriores planteamientos sugerimos que aborden el siguiente problema que slo se detiene en el lmmite que reinicia el proceso.

Caso I Si ... 53 = 3(2) + 2(1) Y ... 74 = 4(2) + 3(1) Entonces [53][74] = 3(4) + 1(3) + 1(2) + 2(1) 53 = 3(2) + 2(1)

74 = 4(2) + 3(1)

[53][74] = 3(4) + 1(3) + 1(2) + 2(1)

Las lnes se usan para indicar con mayor claridad las soluciones, pero se entiende que las filas estn alineadas formadas por "sphiros" que estn unidos en hileras. Por 53 indicamos dos filas de "sphiros" una de longitud 5 y otra de longitud 3, que es igual a 3(2) + 2(1) es decir 3 hileras de 2 spiros ms 2 hileras de uno.

Caso II Si ... 116 = 6(2) + 5(1) Y ... 132 = 2(2) + 11(1) Entonces [116][132] =

Caso III Si ... 412 = 4(2) + 8(1) Y ... 105 = 5(2) + 5(1) Entonces [412][105] = Caso IV Si ... 57 = 5(2) + 2(1) Y ... 23 = 2(2) + 1(1) Entonces [57][23] =

Generalizacin (A quien no le interesen estos asuntos tan serios, contine con las conclusiones, pero no saben lo que se pierden). En el siguiente cuadro aparecen una serie de formalizaciones que nos permiten comprender de mejor manera la dinmica de nuestros sphiros. Primeramente entendemos que las literales a, b, c, d representan a nmeros naturales desiguales ordenados de menor a mayor. Observamos que las interacciones que realizan los sphiros pueden ser expresados segn las propiedades conmutativa y asociativa de la suma y que adems, en el contexto de las reglas que hemos formulado, es posible enunciar una proposicin que se refiere al lmite permitido para el agrupamiento de estos entes y el comienzo de un nuevo ciclo. (Sera interesante disear algn programa de ordenador que permitiera experimentar con otras variables para llegar a generalizaciones ms complejas y, por tanto, ms interesantes). {a, b, c, d} nmeros Naturales a<b<c<d ab = ba Propiedad conmutativa ab = a(2) + (b - a)(1) = a + b [ab] [cd] = [cd] [ab] Propiedades conmutativa y asociativa [ab] [cd] = a(4) + (b-a)(3) + (c-b)(2) + (c-c)(1) = (a+b) + (c+d)

Conclusin; Antes de concluir: es nuestro deseo que estos ejercicios sirvan (sobre todo para los ms jvenes) como un aliciente para abordar de manera divertida los temas de la teora de conjuntos. Ahora s, concluyamos que:

en los casos observados las cantidades se conservan; pero las combinaciones entre los "sphiros", segn las reglas enunciadas, permiten o garantizan un movimiento perpetuo (interacciones entre los elementos, de manera indefinida) y la existencia del sistema como tal.

sphiros.zip (172 bytes) Si lo deseas puedes comprobar tus soluciones de los Casos II, III y IV

Trigonometra Hiperblica. Unas combinaciones particulares de la funcin exponencial da lugar a un interesante tipo de funciones denominadas funciones hiperblicas presentes en muchas ramas de la ciencia. Se definen las funciones hiperblicas, que denominaremos coseno hiperblico y seno hiperblico como

El punto P(x,y), siendo

Ambas, estn relacionadas de forma parecida a como se relacionan las funciones trigonomtricas usuales. Lo mismo que aquellas se identifican con un punto sobre la circunferencia goniomtrica (de radio unidad) x 2 + y 2 = 1) estas se identifican con un punto P(x, y) de la hiprbola unidad: x 2 - y 2 = 1) En efecto, basta comprobar, con un poco de clculo, que el punto dado por P(cosh(x), senh(x)) verifica dicha expresin. De ah resulta que cosh(x) 2 - senh(x) 2 = 1 en lugar de la popular expresin fundamental de la trigonometra sobre la circunferencia. Las grficas de dichas funciones se deducen fcilmente a partir de las funciones exponenciales y son

describe la rama derecha de la hipbola. Para t = 0 resulta P(x,y) = (0,0); para valores de t positivos dicho a punto recorre la rama superior (rama derecha azul); para valores de t negativos la inferior (en rojo). El punto Q(-x,y) recorrer la rama izquierda de la hiprbola.

La grfica de la funcin y =

En dichas grficas puede observarse que la funcin y = cosh(x) es una funcin par (simtrica respecto del eje

tagh(x)

de ordenadas), y que y = senh(x) es impar (simtrica respeco del origen). Es importante observar que estas funciones no son, como ocurre con las funciones trigonomtricas, peridicas.

Comparacin entre las grficas de las funciones exponenciales e x y e - x con las grficas de las funciones hiperblicas y = cosh(x) e y = senh(x)

A partir de la definicin, y con algo de clculo, podemos obtener senh(x + y) = senh(x)cosh(x) + cosh(x)senh(y) cosh(x + y) = cosh(x)cosh(y) + senh(x)senh(y) Haciendo y = x tendremos senh(2x) = 2senh(x)cosh(y) cosh(2x) = cosh 2(x) + senh 2(x) De las expresiones (1) 1 = cosh 2(x) - sen 2(x) (2) cosh(2x) = cosh 2(x) + senh 2(x) obtenemos, sumando 1 + cosh(x) = 2cosh 2(x) y despejando cosh(x) resulta a igualdad lgicamente equivalente

Restando las expresiones (1) y (2) y despejando, resultar

Diofanto, la Aritmtica y algunas Ecuaciones Diofnticas Poco se conoce sobre la vida de Diofanto. Las investigaciones ms creibles lo situan hacia la segunda mitad del siglo III, siendo contemporneo de Pappo. Es clsico el epitafio en la Antologa de Metrodoro. El mismo, con las debidas reservas, nos lleva a calcular una edad de 84 aos. De l ha llegado hasta nosotros Sobre los nmeros poligonales (o Numeris Multangulis), Porismas (que se cree formaba parte de la Arithmetica), Sobre los nmeros fraccionarios y naturalmente la Arithmetica
"Esta es la tumba que guarda las cenizas de Diofanto. Es verdaderamente maravillosa porque, gracias a un artificio geomtrico, descubre toda su existencia. Dios le permiti ser nio durante 1/6 de su vida; luego de 1/2 sus mejillas se cubrieron de barba; despus de 1/7 se encendi la llama del matrimonio, del que, a los cinco aos, tuvo un hijo; pero este nio, desgraciado aunque amado apasionadamente, muri apenas lleg a la mitad de la vida alcanzada por su padre, el cul vivi cuatro aos ms mitigando su dolor con investigaciones sobre la ciencia de los nmeros"

"Como s, muy honorable Dionisio, que quieres aprender a resolver problemas numricos, he emprendido la tarea de exponer la naturaleza y el poder de los nmeros, empezando por las bases que sustentan estas cuestiones. Es posible que parezcan ms difciles de lo que son por ser desconocidas an y que los principiantes duden de conseguir alcanzarlas, pero las comprenders fcilmente gracias a tu actividad y a mis demostraciones, pues que el deseo unido a la enseanza conduce rpidamente al conocimiento [...]"

La Arithmetica fu un tratado de 13 libros del que slo se conocen los seis primeros. Fu encontrada en Venecia por Johann Mller (Regiomontanus, matemtico y astrnomo alemn) hacia 1464 y la primera traduccin latina pertenece a Wilhelm Holzmann (1532-1576) Diophanti Alexandrini Rerum libri sex, Basilea, 1575. En 1621 aparece la edicin de Bachet de Mziriac con el siguiente ttulo: Diophanti Alexandrini Arithmeticorum libri sex; et de Numeris multangulis liber unus. Nunc primun graece et latini editi atque absolutissimis commentariis illustrati, Paris 1621 (que contine adems del texto griego y la traduccin latina aclaraciones y notas). En el grfico puede verse una edicin realizada por Fermat hijo (sobre la traduccin de Bachet) que incluye impresas las anotaciones de su padre. La Arithmetica no es propiamente un texto de lgebra sino una coleccin de problemas (150). No se sabe cuantos de ellos son originales o tomados de otros tratados de la poca; Diofanto presenta en todos ellos una solucin nica y no establece distincin entre problemas determinados e indeterminados. Tampoco existe ningn orden en cuanto a la naturaleza de los problemas o los mtodos de resolucin Arithmetica Libro I Contiene 25 problemas de

Diofanto contina en el prefacio presentando las normas indispensables para leer la obra.

primer grado y 14 de segundo. Arithmetica Libro II Consta de 35 problemas. El problema 8, sin duda el ms famoso, di lugar al llamado "teorema de Fermat" II. 8 Descomponer un cuadrado en dos cuadrados "Si queremos descomponer 16 en dos cuadrados y Diofanto, Fermat y la Arithmetica suponemos que el primero es 1 aritmo, el otro han estado estrechamente tendr 16 unidades menos un cuadrado de aritmo, y, relacionados a lo largo de la historia por tanto, 16 unidades menos un cuadrado de aritmo de las matemticas. Todo empez son un cuadrado. cuando Fermat, en su ejemplar de la Formenos un cuadrado de un conjunto cualquiera de Arithmetica, escribi al lado del aritmos disminuido en tantas unidades como tiene la problema 8 del Libro II: raiz de 16 unidades, y sea el cuadrado de 2 aritmos Cubum autem in duos cubos, aut menos 4 unidades. Este ciadrado tendr cuatro quadratoquadratum in duos cuadrados de aritmo y 16 unidades menos 16 quadratoquadratos, et generaliter aritmos, que igualaremos a 16 unidades menos un nullam in infinitum ultra quadratum potestatem in duos eiusdem nominis cuadrado de aritmo y sumando a uno y otro lado los fas est dividere cuius demostrationem tminos negativos y restando los semejantes, resulta mirabilem sane detexi. Hanc marginis que 5 cuadrados de aritmo equivalen a 16 aritmos y, exiguitas non caperet por tanto, 1 airtmo vale 16/5; luego uno de los nmeros es 256/25 y otro 144/25, cuya suma es Es decir, que la ecuacin 400/25, es decir 16 unidades y cada uno de ellos es xn + yn = zn no tiene soluciones enteras para n > un cuadrado" 2. Diofanto resuelve la ecuacin En el caso n = 2 una solucin es (x, y, z) = (3, 4, 5) y ya se conoca x 2 + x 2 = 16 desde la Grecia clsica. 2 haciendo y = 16 - a 2 que identifica con una En general pueden obtenerse estas 2 ternas, denominadas pitagricas, a expresin de la forma (ka - 4) y haciendo k = 2 obtiene partir de la expresin y 2 = 16 - a 2 = (2a - 4) 2 x = 2n + 1 e identificando llega a a = 16/5 de donde x = 16/5 e y = 2n 2 + 2n 2 y = 12/5 z = 2n + 2n + 1 para n = 1, 2, 3, ... Arithmetica Libro III Consta de 21 problemas. El En Euclides. Elementos X 28 Lema ms famoso es el 19 en el que por primera vez I aparece la expresin general de acude a la geometra para solucionarlo. estas ternas: 2 2 III. 19 Encontrar cuatro nmeros tales que el x=a -b cuadrado de la suma de los cuatro, aumentado o y = 2ab 2 2 disminuido en cada uno de ellos, forma un z=a +b cuadrado. Sin embargo, la demostracin de Arithmetica Libro IV Casi todos los problemas de esta proposicin ha sido, hasta hace este libro (40) se refieren a nmeros cbicos. Como poco, el problema ms famoso, al lo griegos no conocan las frmula de la ecuacin menos ms popular, de las cbica, la sagaz eleccin de los datos por parte de matemticas y a su resolucin se Diofanto hace que se llegue a una solucin

haya unido el nombre de grandes matemticos. Al mismo Fermat se le atribuye una demostracin para el caso n = 4 y a Euler una para n = 3. Dirichlet (1805-1859) y Legendre (17521833) tambin intevinieron y probaron la proposicin para n = 5 Y muchos otros como Sophie Germain, Lam, Kummer, Gerd Faltings (que por sus aportaciones recibi en 1986 una medalla Fields) pero esta columna es demasiado estrecha para contenerlos a todos. Por fin, en 1995 el ingls Andrew Wiles lo logr (despus de algunos sustos).

aceptable. Y como muestra un botn IV. 1 Descomponer un nmero dado en dos cubos cuya suma de races sea dada "Si el nmero es 370 y la suma de las races 10, supongamos que la raz del primer cubo es 1 aritmo y 5 unidades, o sea: la mitad de la suma de las races. Por tanto, la raz del otro cubo ser 5 unidades menos 1 aritmo; luego la suma de los cubos valdr 30 cuadrados de aritmo ms 250 unidades que igualaremos a las 370 unidades del nmero dado, de donde se deduce que 1 aritmo tiene 2 unidades; la raz del primer cubo tendr entonces 7 y la del segundo 3, y, por consiguientes, los cubos sern 343 y 27" Con la notacin actual, Diofanto resuelve el sistema formado por las ecuaciones x 3 + y 3 = 370 x + y = 10 Para lo que supone que x = aritmo + 5 y que y = 5 aritmo . (en lo que sigue designaremos el aritmo por a). Sustituyendo estas expresiones en la primera ecuacin y desarrollando tendremos: (a + 5) 3 + (5 - a) 3 = 30 a 2 + 250 = 370 y para a = 2 obtiene x = 7, y = 3.

Arithmetica Libro V La mayora de los problemas propuestos (28 de los 30 que tiene el libro) son problemas de segundo y tercer grado. En el ltimo, el 30, Diofanto se aparta de su costumbre y propone un problema de los que hoy denominaramos de "mezclas" V. 30 Una persona se embarc con sus sirvientes, quienes le encargaron que les fuera til. Mezcl Puedes leer la historia sobre garrafas de vino, unas de 8 dracmas y otras de 5, este apasionante teorema en El enigma de Fermat de Simon Singh y pag por todo un nmero cuadrado que, aumentado en el nmero de unidades que se te (Planeta) indicar, 60, har que tengas otro cuadrado cuya El 17 de julio de 2000, la raz es el nmero total de garrafas. Averigua Vanguardia Digital public una cuntas haba de 8 y cuntas de 5 dracmas" entrevista con Wiles a la que puedes ir desde aqu Arithmetica Libro VI . Dedicado a resolver tringulos rectngulos de lados racionales; consta de 24 problemas.

"No hay otro problema que pueda justificar lo mismo para m. Fue la ilusin de mi infancia. Nada puede reemplazar eso. Lo he resuelto. Intentar resolver otros problemas, estoy seguro. Algunos sern muy difciles y tendr una sensacin de realizacin otra vez, pero no hay ningn problema matemtico que me pueda cautivar como lo hizo Fermat [...]"

En honor de Diofanto las ecuaciones con coeficientes enteros cuyas

soluciones son tambin enteras se denominan ecuaciones diofnticas. Las ms sencillas son las ecuaciones lineales con dos incgnitas de la forma Ax Bx = C Ejemplo Hemos comprado libros de una oferta por 86 el volumen y en otra oferta libros a 76 volumen pagando en total 1176 . Deseamos saber cutos libros se han comprado de cada oferta Si x es el nmero de libros del primer lote e y del segundo podemos plantear la ecuacin 68 x + 76 y = 1176 La condicin necesaria para que este tipo de ecuaciones admita solucin es que C sea divisible por el m.c.d(A,B), en nuestro caso m.c.d(68, 76) = m.c.d(68, 76) = 4 con lo que la ecuacin inicial quedar de la forma 17 x + 19 y = 294 y los coeficientes de x e y, 17 y 19, son primos entre s. Veremos a continuacin que este tipo de ecuaciones de la forma ax by = c admite siempre soluciones enteras. Por ejemplo, despejando ax y dando a y los valores 0, 1, 2, ..., a - 1, resultan los a nmeros y ax = c + by 0 c 1 c+b 2 c + 2b ... ... ... k c + kb ... ... ... h c + hb ... ... ... a-1 c + b (a - 1) En donde nos hemos ajustado al caso ax - by = c. Al dividir cada uno de los a nmeros de la segunda columna por a, para obtener x, obtendremos siempre restos distintos. En efecto. Supongamos que para los valores k y h diesen el mismo resto. Entonces los nmeros c + kb y c + hb sern congruentes mdulo a, es decir

Teniendo en cuenta el Teorema Fundamental de las Congruencias (La condicin necesaria y suficiente para que dos nmeros sean congruentes mdulo m es que su diferencia sea un mtiplo de m) resultar que la diferencia de los nmeros c + kb y c + hb debe ser un mltiplo de a, es decir

y como a y b son primos entre s debera ser la diferencia k - h un mltiplo de a, pero eso no es posible pues h y k son distintos y menores que a. Apliquemos dicho razonamiento al problema y tendremos y 17 x = 294 - 19y x = (294 - 19y)/17 0 17x = 294 x = 294/17 1 17x = 294 - 19 = 275 x = 275/17 2 17x = 294 - 38 = 292 x = 291/17 3 17x = 294 - 57 = 237 x = 237/17 4 17x = 294 - 76 = 218 x = 218/17 ... ... ... ... ... Puede comprobarse que para los valores5, 6, 7, 8, 9 y 10 no se obtiene solucin entera, pero ... 11 17x = 294 - 209 = 85 x = 85/17 = 5 Luego se han comprado x = 5 libros de 68 e y = 11 libros de 76. Ya vemos la laboriosidad de este mtodo, pero el gran Euler propuso el prctico y elegante que ahora se expone. Consideramos el coeficiente ms pequeo de x e y, en nuestro caso 17, el coeficiente de x. Despejamos x, efectuamos el cociente y asociando tendremos:

y como x debe ser entera, para y = 11 resulta - 11 - 1 = 5 Medidas indirectas.

y se obtiene x = 17

"...pero no se puede entender si antes no se aprende a entender la lengua, a conocer los caracteres en los que est escrito. Est escrito en lengua matemtica" Los tringulos MNP y ABP son semejantes: sus lados homlogos son proporcionales y sus ngulos iguales. Corolario Dos tringulos rectngulos que tengan un ngulo agudo igual son semejantes.

Clculo de la altura de una pirmide

Se cuenta que Thales de Mileto (aprox. 611-545 a.C), uno de los "siete sabios de Grecia", utilizando la semejanza resolvi dos problemas: calcul la altura de una pirmide en Egipto determin la distancia de una embarcacin a la costa

Puesto que los rayos del Sol inciden paralelamente sobre la Tierra, los tringulos rectngulos determinados por la altura de la pirmide y su sombra y el determinado por la altura del bastn y la suya son semejantes. Podemos, por tanto, establecer la proporcin H/S = h/s Tambin se le atribuyen las primeras de donde demostraciones geomtricas H = (h.S)/s utilizando un lenguaje lgico. (Por ejemplo, que todo ngulo inscrito en Clculo de la distancia de una embarcacin a la costa una circunferencia es recto,

que los ngulos opuestos por el vtices son iguales,

Colocado un observador en P (frente a la embarcacin) y lanzando desde M una visual a B pueden determinarse los tringulos rectngulos OPB y MOP'. Como los ngulos etc.) MOP' y POB son opuestos por el vtice son iguales y por tanto los dos tringulos rectngulos son semejantes y " ... fu primero a Egipto y desde all podemos establecer la proporcin siguiente: introdujo este estudio en Grecia". distancia (PB) es a la distancia (P'B) como Proclo distancia (OP) es a la distancia (OP') "Es el primer hombre en la historia al y todas esas distancias, excepto la buscada, pueden medirse que se le atribuyen descubrimientos directamente. matemticos concretos" Boyer Con el lenguaje actual de las proporciones podemos escribir:

es decir:

Sello de la Repblica Griega dedicado a "Thales el Milesio"

ERATSTENES de Cirene (aprox. 276-194 a.C.). Director de la Biblioteca de Alejandra y contemporneo de Arqumedes y Apolonio. Fu el primer matemtico de la historia del que se tiene noticia que midi el radio de la Tierra. Se bas en dos hiptesis muy atrevidas para su poca: Los rayos del Sol inciden paralelamente sobre la Tierra La Tierra es redonda. (Una observacin peligrosa si tenemos en cuenta que siglos despus la verdad popular pona en duda este hecho). Estableci tambin la oblicuidad de la Eclptica (en 23 51' 20") y de casi todos es conocida su famosa criba para calcular nmeros primos. Determinando el radio de la Tierra Cuenta la historia que Eratstenes observ que cuando un poste en Siena (actualmente Assuan) no proyectaba sombra, en Alejandra el mismo poste proyectaba una sombra de aproximadamente 7 grados y 12 segundos. Eratstenes midi la distancia entre Siena y Alejandra (medidicn nada fcil para la poca) y obtuvo que aproximadamente era de unos 5000 estadios (una medida de longitud griega). Pudo entonces establecer que si para un ngulo de 712' (ver figura) la distancia era de 5000 estadios para 360, aproximadamente 50 veces ms, debera ser de 5000 x 50 = 250000 estadios. No conocemos, con las medidas actuales, cul es la longitud de un estadio. A travs del historiador Plinio se ha podido establecer que es de, aproximadamente, unos 157,5 metros, por lo que 250000 (estadios) x 157,5 (m) = = 39375000 (m) = 39375 (km) Teniendo en cuenta que la circunferencia total de la Tierra se estima actualmente en unos 40000 km, es de admirar el resultado obtenido por Eratstenes. Ese error se debe, presumiblemente, a las tcnicas primitivas de medicin de la poca (Siena y Alejandra no estn a 5000 estadios exactamente) y la segunda a que ambas ciudades no se encuentran sobre el mismo meridiano, pero a pesar de todo ...

Con la notacin actual tendremos: tag (x) = s/h de donde: x = arc tag (s/h) Un bastn de 2 metros arrojara en Alejandra a las 12 una sombra de s = 2 . tag (7 12') = 0,25 m aproximadamente (es decir, unos 25 cm) y en Siena el Sol incidira ortogonalmente sobre el bastn y no arrojara sombra.

ARISTARCO DE SAMOS (310-230 a.C.). Slo se sabe, a ciencia cierta, que naci en Samos, que fu el primer cientfico griego que plante y resolvi problemas astronmicos con sentido matemtico, lo que hizo que tuviese la audacia de apartar los prejuicios de considerar a los astros como dioses (o al menos con rango divino) y que fu director del Liceo (284-269). Podemos situarlo cronolgicamente entre Euclides y Arqumedes.

Comparando distancias Relacin entre las distancias de la Tierra, Sol y la Luna. Aristarco observ que cuando la mitad de la Luna est iluminada el tringulo SLT es recto. Estim que el ngulo LTS = x era de unos 87 (aunque obtuvo valores comprendidos entre 86 49' y 87 8'). Teniendo esto en cuenta y con la notacin actual de la trigonometra resulta que Una pequea variacin en la medida del ngulo LTS lleva a grandes variaciones ngulo x tang (x) 87 19,08 87 30' 22,90 88 28,64 88 30' 38,19 89 57,29 89 30' 114,59 89 40' 171,89 89 50' 343,77

siendo d(TL) y d(TS) las distancias de la Tierra a la Luna y al Sol respectivamente. Como 1/cos (87) es aproximadamente igual a 19, resulta que distancia (TS) = 19 distancia (TL) Hoy sabemos que ese resultado no es cierto, pues dicha relacin es de aproximadamente unas 400 veces, d(SL) = 400 d(TL). A pesar de lo ingenioso del mtodo, los instrumentos de medidas de la poca jugaron a Aristarco esta mala pasada.

Aristarco observ que desde la Tierra, la Luna y el Sol se ven bajo un ngulo de 0,5. Como d(TS) = 19 d(TL) los dimetros DS y DL, del sol y la Luna se encontrarn en la misma porporcin, es decir DS = 19 DL

Sus obervaciones astronmicas le hicieron elaborar el siguiente modelo al observar que el dimetro de la Luna recorra dos veces la sombra terrestre durante un eclipse. En las figuras DT, DL y DS son los dimetros del Sol, la Tierra y la Luna respectivamente y d(TS), d(TL) las distancias entre el Sol y la Tierra y la Tierra y la Luna. d es una variable auxiliar que desaparecer en clculos posteriores.

Como los tringulos VCC' y VAA' son semejantes (1) Anlogamente, al ser VCC' y VBB' semejantes (2)

Despejando d en (1) y teniendo en cuenta que DS = 19 DL resulta Sustituyendo en (2) llegaremos a que

de donde DS = 6,67 DT

De esta forma obtuvo Aristarco, y pudo calcular, la relacin de los dimetros del Sol y la Luna en funcin del dimetro de la Tierra. Veamos, por ltimo, cmo obtuvo la distancia entre la Tierra, la Luna y el Sol. Como podemos colocar 360 * 2 = 720 veces el dimetro de la Luna sobre la rbita que sta describe alrededor de la Tierra resulta: 2 pi d(TL) = 720 DL de donde d(TL) = 40 DT y como d(TS) = 19 d(TL) resulta d(TS) = 760 DT

Hoy sabemos que estas medidas no son ciertas, pero el mtodo elaborado por Aristarco ha sido, y es, un ejemplo de cmo el hombre mediante la observacin y el ingenio pudo llegar a obtener un mtodo de trabajo riguro, esto es un mtodo cientfico. Aristarco puede ser considerado como uno de los precursores de la teora helioctrica, pero sus modelos cayeron, frente a los de Aristteles, en el olvido y no seran reedescubiertos hasta mucho tiempo despus.

Los trigonometra es una poderosa herramienta matemtica que nos permite hacer infinidad de medidas indirectas. Para ello slo necesitaremos un metro y un aparato que nos permita medir ngulos. Desde aqu puedes ver una sencilla aplicacin de cmo el prncipe Redondity salv a la princesa Floripondia. Algunas medidas indirectas Deseamos medir la distancia entre A y B siendo B inaccesible. Fijamos una posicin cualquiera N y medimos su distancia a A. Sea d. Medimos los ngulos x e y (por lo tanto podemos conocer z). Aplicamos el teorema del seno al tringulo obtenido

Deseamos medir la distancia entre A y B siendo ambos inaccesibles. Fijamos los puntos M y N y medimos la distancia entre ellos; sea d Desde M podemos medir el ngulo x y desde N el ngulo y; de esta forma podemos conocer al ngulo A aplicando el teorema del seno al tringulo NMA podemos conocer MA Desde N realizamos la misma operacin, ahora para el tringulo MNB, y medimos x' e y' (con lo que conocemos el B) y aplicamos a dicho tringulo el teorema del seno y calcular MB Para calcular la distancia entre A y B basta aplicar el teorema del coseno al tringulo MAB

La sucesin de Fibonacci y la Bolsa.


Prof. Francisco Gallego Puche (un apasionado de la Bolsa). En el mundo de la Inversin la faceta ms atrayente es la toma de decisiones sobre cundo, cmo y en qu invertir. Una y otra vez se ha intentado encontrar un mtodo que permita tomar la decisin de invertir siempre correcta, pero hasta ahora nadie la ha encontrado. Hasta la fecha slo se conoce un mtodo que se pueda acercar a este ideal. Esto es que, aunque no hace predicciones exactas, si que ha incrementado significativamente el nmero de decisiones

correctas y ha disminuido el de errneas, tanto en la eleccin de Mercados y Productos, como en la seleccin del momento de realizar la inversin. Este mtodo es el Anlisis Tcnico. El Anlisis Tcnico puede considerarse como la unin entre: Anlisis Grfico y Anlisis Cuantitativo. El Anlisis Grfico es el estudio de los grficos de precios y la interpretacin de las figuras que en ellos se forman. Respecto al Anlisis Cuantitativo engloba, el desarrollo, interpretacin y aplicacisn de modelos matemticos y estadsticos ( indicadores y osciladores tcnicos). Uno de los indicadores ms usados, y que generan mejores resultados en el Anlisis Tcnico, son los medias mviles. Las medias mviles "suavizan" la curva de precios de los valores mobiliarios amortiguando las bruscas variaciones que se producen por efecto de la volatilidad, y ofrecen un perfil ms adecuarlo para el seguimiento de la tendencia y para la generacisn de seales de compra y venta. Existen distintos tipos de medias mviles, segn su utilizacin en la generacin de seales, y hay varios mtodos de optimizacin que pueden aplicarse para adecuar las medias a los distintos horizontes temporales. En su concepcin ms simple la media de un conjunto de datos es el resultado de sumar el conjunto de datos y dividir el resultado por el nmero de unidades que constituyen el conjunto de datos. Si este clculo se repitiera sucesivamente, descartando el primero de los datos del conjunto y aadiendo un nuevo dato, se obtendra una sucesin de valores medios que representados grficamente constituiran una lnea que se denominara media mvil. El trmino "mvil" deriva obviamente del hecho de que en cada clculo se elimina el primero de los datos del conjunto y se incorpora un nuevo dato. Segn la naturaleza del mercado que se analice, la finalidad que persiga el analista tcnico, y la proyeccin temporal que se contemple, puede que unas veces sea ms ventajoso utilizar medias largas (100-200 sesiones), en tanto que en otras pueda ser ms aconsejable la aplicacin de medias cortas (5-25 sesiones) ms sensibles a la variacin de los precios. La experiencia ha demostrado con rotundidad que en la prctica las medias funcionan mejor cuando los periodos de tiempo elegidos para el clculo de las medias son nmeros de la Serie de Fibonacci, una serie numrica en la que cada trmino es la suma de los dos anteriores (1, 1, 2, 3, 5, 8, 13, 21, ...). Estos nmeros de Fibonacci se ajustan bastante bien a periodos y ciclos burstiles.

Grficos Matemticos.
Mariposas Matemticas

e s. paramtricas: x = sen (5t) cos (t); y = sen (5t) sen (4t)

e s. paramtricas: x = sen (5t) cos (t); y = sen (5t) cos (4t)

e s. paramtricas: x = sen (5t) cos (t); y = sen (5t) sen (6t)

e s. paramtricas: x = sen (5t) cos (t); y = sen (5t) sen (8t)

Ptalos

r = sen (3x)

r = 2sen (2x)

r = 2sen (5x/3)

r = k sen (tx); t > 0 Si t es impar hay t ptalos Si t es par hay 2t ptalos Si t = a/b es racional y a y b son impares hay a ptalos; en caso contrario 2a Si t es irracional existe un nmero infinito de ptalos que se van recubriendo

r = 2sen (4x/3)

r = 2sen (sqrt(20)x)

Sobre la longitud de la circunferencia y el rea del cculo. Una de las formas ms difundidas de la Naturaleza es la circular. Casi todas las formas tienden a hacerse ms o menos "redondeadas". Cuando en matemticas un conjunto de puntos tiene una propiedad comn dicho conjunto se denomina lugar geomtrico. El lugar geomtrico de los puntos del plano que equidistan de otro, que se denomina centro, es una circunferencia. El segmento de recta que une el centro con cualquier punto de la circunferencia es el radio de la circunferencia. La porcin de plano limitada por una circunferencia (includa la misma) se denomina crculo y el centro de la circunferencia es el centro del crculo.

Si dividimos la longitud entre el dimetro de la rueda obtenemos un valor que es independiente del tamao de la rueda. Es decir, cualquier rueda, del tamao que sea, al dar una vuelta completa recorre un camino de una determinada longitud. Si dividimos Una rueda, al dar una vuelta completa, describe una trayectoria cuya longitud es el dicha longitud entre el dimetro de la rueda permetro de la circunferencia de la rueda. siempre obtenemos el mismo valor. Este hecho era conocido por los babilonios y ya se encuentran noticias sobre el mismo en los papiros egipcios que se conservan en el Museo Britnico. Esta relacin entre la longitud de la circunferencia y su dimetro es, posiblemente, la ms popular de todas las constantes matemticas: el nmero . Dicho nmero, irracional, ha ocupado a generaciones de matemticos y su atractivo perdura en nuestros dias. Uno de los primeros trabajos fiables que se realizaron fu debido a Arqumedes

Comenz inscribiendo y circunscribiendo en una circunferencia un hexgono, a continuacin un dodecgono y as, doblando sucesivamente el nmero de lados, cuentan las crnicas que lleg hasta un poligono de 96 lados. Si designamos por I6, I12, ... I96, los permetros de los polgonos regulares inscritos y por C6, C12,... C96 los de los polgonos regulares circunscrtos, Arqumedes lleg a la conclusin de que

es decir, los permetros de los polgonos regulares inscritos y circunscritos de doble nmero de lados vienen dados por las medias armnica y geomtrica.

Veamos cmo lleg Arqumedes a este resultado. Consider el hexgono inscrito y circunscrito a la circunferencia. Resulta: I6 = 6 AB = 12 AH C6 = 6 CD = 12 CG En el tringulo COG, al ser OE la bisectriz de dicho ngulo, resulta

(pues la biscetriz interior de cualquier ngulo de un tringulo determina sobre el lado opuesto segmentos proporcionales a los lados de dicho gulo; la ltima igualdad resulta de OG = OA = r) Como los tringulos COG y AOH son semejantes y multiplicando y dividiendo por 12

Por tanto, resulta:

Sumando 1 a ambos miembros de dicha proporcin y operando

(pues EC + EG =CG)

A continuacin Arqumedes determina el lado del hexgono circunscrito para as obtener el permetro C6 En OA'B' aplicando el teorema de Pitgoras resulta:

Como los tringulos OAB y OA'B' son semejantes

de donde

A partir de dichas expresiones, tomando como valor de r = 0,5 (tomaremos aproximacin hasta las milsimas) tendremos: I6 = 6 r = 3 C6 = 12 x = 3,464 I12 = 3, 105 C12 = 3, 215 I24 = 3, 132 C24 = 3, 159 I48 = 3, 139 C48 = 3, 146 I96 = 3, 141 C96 = 3, 142

Aqu, en la Gacetilla, mediante un programa, hemos obtenido las siguientes aproximaciones para Lados In Cn 192 384 768 1536 3072 6144 3, 141 4 3, 141 55 3, 141 58 3, 14159 0 3, 14159 2 3, 141592 5 3, 141 8 3, 141 66 3, 141 61 3, 14159 7 3, 14159 3 3, 141592 9 3, 141592 7 3, 1415926 7

12288 3, 141592 6 24576 3, 1415926 4

49152 3, 14159265 1 3, 14159265 7 98304 3, 14159265 3 3, 14159265 4 196608 3, 141592653 4 3, 141592653 8 393216 3, 141592653 5 3, 141592653 6

Sobre el rea del crculo. Podemos considerar el crculo como un polgono regular de infinitos lados en el que la apotema se va convirtiendo en el radio. Esta consideracin hace que podamos justificar fcilmente el rea de un crculo de radio R a partir de la expresin que nos proporciona el rea de un polgono regular, sin ms que sustituir el permetro por la longitud de la circunferencia.

Como el rea de un polgono regular viene expresado por el producto del semipermetro por la apotema y el semipermetro de la circunferencia (la mitad de su longitud) es R rea del Cculo = R . R = R 2

El clculo integral es una poderosa herramienta matemtica que permite formalizar estos resultados.

El rea de la superficie limitada por la funcin continua y = f(x) , las rectas x = a, x = b y el eje de abscisas viene dada por la La expresin analtica de la circunferencia de centro (0,0) y radio R es integral x2 + y2 = R2 Vamos a calcular el rea que limita dicha circunferencia y los ejes coordenados positivos. La misma viene dada por

Efectuando el cambio de variable x = R sen (t) resulta

La longitud de un arco de curva dado por la funcin continua y = f(x) (y con Por lo tanto, el re del crculo limitado por una circunferencia derivada continua) entre las

rectas x = a y x = b viene dado por la integral

de radio R es rea = 4 A 1 = R2

La longitud del arco que la circunferencia dada intercepta en el primer cuadrante viene dada por

Por lo tanto, la longitud de la circunferencia de radio R es 4.L= 2 R Y eso es todo, por ahora. Quedan varias cosas en el tintero, pero para comenzar no est mal, eh! :-)

Copo de Nieve. En 1904 el matemtico sueco Niels F. Helge von Koch (1870-1924) construye la curva que lleva su nombre. Parte de un segmento unidad [0,1], poligonal P0, que divide en tres partes sustituyendo la parte central por los dos segmentos que, junto con dicha parte (anulada), formara un tringulo equiltero. Se obtiene as la poligonal P1. A continuacin se repite el proceso con cada una de las partes resultantes y se obtiene la poligonal P2. Este proceso se repite indefinidamente obtenindose en cada etapa k una poligonal de longitud (4/3)k. La curva de Koch se define como la curva lmite de la poligonal Pk cuando k tiende a infinito. Si el iniciador del proceso es un tringulo equiltero y se utiliza como generador la curva de Koch, se obtiene la Isla de Koch o Copo de Nieve cuando n crece indefinidamente.

Cuando n tiende a infinito, el permetro de dicha curva es infinito y sin embargo el rea limitado por dicho permetro son las 8/5 partes del rea inicial. Como un lado de copo de nieve est constituido por cuatro trozos y cada uno de ellos tiene la tercera parte de la longitud total, su dimensin fractal es log(4) / log(3) = 1,261 ...

Copo de Nieve n.2

No Copo de Nieve n.2

Puedes ver ms sobre Copo de Nieve en el problema (#082)

COPO DE NIEVE Esto es un fractal y se le conoce como "copo de nieve" generarlo es fcil (lo difcil es dibujarlo). 1 Se inicia con un tringulo equiltero original de lado L 2 Cada lado se segmenta en tres partes iguales. 3 Tomando como base el segmento medio de cada lado; se traza sobre el tringulo equiltero. En este caso, la nueva longitud de lado ser 1/3 del lado original. Esta operacin se repite para los otros dos lados del tringulo original. 4 Se borran los segmentos de enmedio y se tiene una nueva figura con puntas triangulares. (6 puntas) 5 A cada una de la puntas tringulares generadas se les aplican los pasos 2, 3 y 4. El fractal se genera repitiendo estos pasos sucesiva e indefinidamente a las puntas generadas. Se pide determinar el rea de dicho fractal.

Solucin Si partimos de un tringulo equiltero de rea A, en la primera iteracin obtenemos 3 triangulos de

area A/9 cada uno; en la segunda iteracin obtenemos 12 tringulos de rea A/92 cada uno; en la tercera iteracin obtenemos 48 tringulos de rea A/93; en la cuarta iteracin obtenemos 192 tringulos de rea A/94, etc. Asi pues si sumamos todas las reas parciales (nmero de tringulos por rea de cada uno), tendremos la siguiente progresin: 3.A/9, 3.4.A/92, 3.42.A/93,... 3.4(N-1).A/9N cuyo primer termino es a1= 3A/9 y cuya razn es r = 4/9. Si tenemos en cuenta la suma de los trminos de una progresinn geomtrica ilimitada y decreciente es entonces dicha suma es (3A/9) : (5/9) = 3A/5.

Si a esto le aadimos el rea del tingulo inicial obtenemos 3A/5 + A = 8A/5 que es el rea del "copo de nieve" que perseguamos, en funcin del rea del tringulo de partida A. Dado que el rea de un tringulo equiltero en funcin del lado es "copo de nieve" en funcin del lado es entonces el rea del

Ir a pgina de Copo de Nieve

Problema 1: Una palabra mgica de oscuro y antiguo origen es ABRACADABRA la cul escrita en amuletos permita al portador, segn los creyentes, prevenir enfermedades. La forma en que ella se escriba en estos casos era la indicada en la figura. De cuntas maneras se puede leer la palabra completa empezando desde la A de arriba y continuando hacia abajo, pasando siempre de una letra hacia otra?.

Solucin de JPinto Me he sentido atraido por el proyecto MECE y he copiado sus problemas. Creo que el primero tiene una respuesta muy sencilla: Puesto que desde cada letra hay dos caminos a elegir y hay 10 pisos bajo la primera A, habr 2 10 posibles lecturas de ABRACADABRA. Es correcto? Respuesta de MECE97

Es correcta, la cantidad de posibles caminos coincide y se relacionan con el triangulo de Pascal:

Si se suman los coeficientes que corresponden a la undcima fila se obtiene la solucin del problema. 1024 caminos distintos, o sea dos elevado a 10. En votacin de grupo, hemos considerado que si bien la respuesta es correcta, la consideramos inconclusa, por carecer de los fundamentos que antes esbozamos. Saludos desde Chilo. Problema 2 Demostrar que todas las soluciones Z+ (enteras positivas) de x2 - y2 = a3 son de la forma: x = n (2n + 1) y = n (2n - 1)

Solucin de JPinto La ecuacin puede escribirse: (x + y)(x - y) = a * a 2 y si x e y son naturales, x+y>x-y y por tanto se pueden escribir las ecuaciones: x + y = a2 x-y=a que nos dan como solucin x = (a + 1) (a/2) e y = (a - 1) a/2. Finalmente, llamando n a a/2 salen las formas dadas. Informacin adicional de Ignacio Larrosa Castro Pero a 3 se puede escribir de dos formas como producto de enteros positivos:

a 3 = a 7 a 2, que da lugar a la solucin del enunciado y correctamente desarrollada por JPinto. a 3 = 1 7 a 3, admito que pueda parecer sorprendente, pero debe considerarse. Entonces: (x + y)(x - y) = 17a 3, puede ser x - y = 1, x + y = a 3. Reemplazando en la segunda y = x - 1, tenemos 2x - 1 = a 3 ==> x = (a 3 + 1)/2, y = (a 3 - 1)/2.

Aqu a debe ser impar, a diferencia de la otra forma en que deba ser par, y mayor que 1 si queremos descartar la solucin trivial 1 2 - 0 3 = 1 3.. Entonces la solucin mnima es para a = 3 x = (3 3+1)/2 = 14 y = (3 3-1)/2 = 13 14 2 - 13 2 = 3 3

(... y ms precisiones) Respecto al problema, se me olvid otra precisin: Las soluciones expuestas por JPinto y en mi anterior mensaje son _todas_ nicamente si a es primo. Si no lo es, caben ms factorizaciones y crece el nmero de soluciones. Por ejemplo, si a = 6, tenemos todas estas posibilidades:
o o o o o o o o

1 7 216 2 7 108 3 7 72 4 7 54 6 7 36 ===> (la propuesta en el enunciado) 8 7 27 9 7 24 127 18

Como uno de los factores es (x - y) y otro (x + y) ambos deben tener la misma paridad. De las descomposiciones anteriores deben descartarse aquellas en que los factores tengan distinta paridad, pero an quedan tres adems de la nica considerada implcitamente en el enunciado del problema. Saludos. Bueno, pues ah queda eso. Problema 4 La hormiga escaladora Cunta distancia recorre una hormiga que sube por un cilindro vertial de radio 5 cm. y altura 12 cm. sabiendo que su recorrido forma una hlice y que en cada vuelta completa asciende 4 cm. de altura ?

Solucin Dividamos el cilindro en 3 cilindros de igual base pero de altura 4 cm cada uno. Cada uno de estos cilindros lo desrrollamos de forma que obtenemos una superficie rectangular con altura 4 cm y base el permetro del cilindro inicial. El permetro del cilindro inicial es 2 x pi x radio. Como el radio es 5 cm, el permetro es 31'415927 cm, que es a su vez la longitud de la base del rectngulo . El recorrido en forma de hlice, al realizar este proceso de particin, es la distancia entre ngulos opuestos de cada uno de los rectngulos obtenidos. Es decir que el camino de la base hasta la cima del cilindro inicial es tres veces esta diagonal (ya que hemos dividido el cilindro inicial en tres). La diagonal es, segn el teorema de Pitgoras, la raz cuadrada de la suma de los cuadrados de la base y de la altura del rectangulo. Es decir, Diagonal = 31'669551 cm . Como tenemos tres rectngulos iguales, la distancia total es 3 x 31'669551 cm = 95'008653 cm

Problema 5 La circunferencia arqueada Una circunferencia de radio 1 cm. se divide en cuatro arcos iguales para formar la figura que se indica. Cunto vale el rea de la figura?. Justificacin

Solucin [...] el problema es el de la circunferencia arqueada. Ah va: Est claro de que el rea que queremos calcular es la del cuadrado inscrito en la circunferencia de radio 1 cm, por lo tanto, sabemos que el dimetro de la circunferencia es 2 cm. Aplicando el teorema de Pitgoras, obtenemos el valor del lado que es sqrt(2). Sea D la diagonal del cuadrado (que es el dimetro de la circunferencia) y L el lado. Como D 2 = 2 L 2, resulta que el rea es A = L2 = D2 / 2 Como el dimetro es 2 cm, el rea es 2 cm cuadrados.

Otra forma Calculamos el rea del segmento circular AQBA, que es el rea del sector circular CAQBC menos el rea del tringulo ACBA rea tringulo: A T = 1/2 CA. CB. sen (90) = 1/2 u 2 rea sector: A S = 1/4 (rea del cculo de radio 1) = (pi) /4 De donde el rea del segmento circular buscado es A SC = A S - A T = (pi)/4 - 1/2 El rea buscada es, por tanto, A crculo - 4 A SC = 2 u 2

Problema 6 Divisibilidad Demostrar que n 7 - n es divisible por 42.

Solucin Veamos si es divisible por 2


Supongamos que n es par. Un nmero par elevado a cualquier nmero, es un nmero par. Si a un nmero par le restamos otro nmero par, obtenemos un nmero par. Supongamos que n es impar. Un nmero impar elevado a cualquier nmero, es un nmero impar. Si a un nmero impar le restamos otro nmero impar, obtenemos un nmero par. Por lo tanto, n 7 - n es par y por tanto divisible por 2.

Veamos si es divisible por 3 n 7 - n = n (n 6 - 1) = n (n 3 - 1) (n 3 + 1) Se debe cumplir que o bien n 3 - 1 bien n 3 + 1 bien n 3 sean divisibles por 3. Si n 3 - 1 n 3 + 1 es divisible por 3, queda demostrado que n 7 - n es divisible por 3. Supongamos que la condicin anterior no es cierta; entonces n 3 es divisible por 3. Por lo tanto n 3 Mod 3 = 0. Esto implica que n Mod 3 = 0 , y concluimos que en este caso n es divisible por 3. Cometario de la Gacetilla: si n es divisible por 3 estara demostrado la divisibilidad; si no lo es, entonces n 3 tampoco lo ser, pero lo sera n 3 + 1 n 3 - 1, ya que los mltiplos de 3 "van de tres en tres") Divisibilidad por 7 Segn el teorema de Fermat:"Sean s y a dos nmeros enteros cuyo mximo comn divisor es 1; entonces si s es un nmero primo, se cumple que a s - 1 Mod s = 1 para todo entero a". Supongamos que m.c.d (n,7) = 1, entonces, como 7 es primo, se cumple n 6 Mod 7 = 1. Por lo que (n 6 - 1) Mod 7 = 0. Como n 7 - n = n (n 6 - 1), entonces n 7 - n es divisible por 7 al serlo n 6 - 1. Supongamos por el contrario que m.c.d(n,7)<>1, entonces quiere decir que n es divisible por 7, y como n 7 - n = n (n 6 - 1), entonces n 7 - n es divisible por 7.

Como hemos demostrado que n 7 - n es divisible por 2, 3 y 7, entonces lo es por su producto. Y concluimos que n 7 - n es divisible por 2 x 3 x 7 = 42. Espero haber dado con la solucisn correcta. Un saludo. Jacinto Ruiz.

Otra demostracin de la divisibilidad por 7 Por induccin: Es divibible para n = 1, n = 2, etc. Lo suponemos cierto para n (Hiptesis de induccin) Lo demostramos para n + 1; hemos de probar que (n + 1) 7 - (n + 1) es mltiplo de 7. En efecto, desarrollando dicha expresin y asociando debidamente resulta: (n + 1) 7 - (n + 1) = = n 7 + 7 n 6 + 21 n 5 + 35 n 4 + 35 n 3 + 21 n 2 + 7 n + 1 - n - 1 = = (n 7 - n) + 7 (n 6 + 3 n 5 + 5 n 4 + 5 n 3 + 3 n 2 + 7 n) que es mltiplo de 7 por serlo ambos sumandos (El primero por la hiptesis de induccin). Problema 7 El prncipe Redondity El prncipe Redondity, deba salvar a la pricesa Floripondia, encerrada por el ogro Patiperro en una alta Torre construida sobre un cerro inaccesible. Para realizar esta empresa deba construir un puente y una escalera. Enorme empresa! ... pero el amor todo lo puede. Haba un bosque cerca y, adems, en las alforjas de su caballo Tranquilidad traa herramientas de carpintero e instrumentos de medir. Su problema era cmo calcular la altura de la torre y el ancho del acantilado para construir el puente y la escalera necesaria para salvar a su princesa.

Solucin Sabemos que el prcipe dispone de herramientas y aparatos de medida. Por lo tanto, debe disponer de algo para medir distancias y algo para medir ngulos (por ejemplo un teodolito). Nos ponemos a X metros del acantilado. Con el teodolito medimos el ngulo que forma la cima de la torre con la base y con nosotros mismos, llammosle b. Luego nos situamos en el filo del acantilado y medimos otra vez el ngulo, llammosle a. Por lo tanto, conocemos a, b y la distancia X. Llamemos L a la altura de la torre y llamemos P a la anchura del Planteamos las siguientes ecuaciones: tangente (b) = L / ( P + X ) tangente (a) = L / P de ah obtenemos los valores de P = (X x tangente (b)) / (tangente (a) - tangente (b)) y despus el de L = (X x tangente (b) x tangente (a)) / (tangente (a) - tangente (b))

Y ya hemos encontrado la solucin. Bueno, espero que sea correcta. Un saludo. Jacinto Ruiz.

Aadido de la G.M. La solucin es correctsima, pero... ya sabemos que en un descuido del prncipe, su caballo Tranquilidad haba roto el teodolito. A pesar de ello, el joven prncipe no se arredr y como era un lector de los clsicos (Euclides, Arqumedes,...), desempolv un antiguo libro en el que ley: "[...]la distancia entre la primera y la segunda posicin del agua dividida por la diferencia entre las distancias del hombre al agua, cuando se multiplica por la altura de los ojos es la altura, y cuando se multiplica por la distancia entre el agua y el hombre, es la distancia entre el agua y la casa." En el cercano bosque Redondity llen un recipiente de agua del cristalino arroyo.

Coloc el recipiente al borde del foso y se desplaz hacia atrs hasta que vi reflejada en la superficie del agua la parte superior del castillo. Resultaba entonces que los tringulos PBA y PMR eran semejantes. Midi la distancia PM = d1 y su altura r y estableci la proporcin

Seguidamente desplaz el recipiente una distancia D, procedi de la misma forma que lo haba hecho anteriormente hasta que vi reflejado el extremo de la fortaleza en el recipiente. Midi la distancia QM' = d2, dedujo que los tringulos QBA y QM'R' eran semejantes y estableci la proporcin:

A partir, de ahora, Redondity, experto astrlogo y matemtico, comenz a hacer sus clculos. Y obtuvo:

y de ah:

De esta forma pudo conocer la distancia del foso y la altura del castillo, cosntruir una

pasarela y una escalera y rescatar a su amada Floripondia. Redondity y Floripondia se casaron, fueron felices y comieron perdices. Y colorn colorado, este cuento se ha acabado. Moraleja: el amor y la lectura de los clsicos todo lo pueden. Problema 8 Matemticos Selenitas Los habitantes de la Luna utilizan una medida de longitud llamada lunario (Los Primeros Exploradores de la Luna. H.G.Wells), la cul fue adoptada pues el rea de la superficie lunar, medida en lunarios cuadrados, coindice con el volumen de la Luna, medido en lunarios cbicos. El dimetro de la luna es de 3475 km. Cunto mide un lunario en kilmetros?.

Solucin rea de la superficie esfrica (Luna) = 4.pi.R 2 Volumen de la esfera (Luna) = ( 4.pi.R 3 ) / 3 Igualando ambos tenemos: R 3 - 3 R 2 = 0 Resolviendo obtenemos R = 0 (absurdo) y R = 3 lunarios. Por otra parte sabemos que el dimetro de la Luna es 3475 km, por lo que 2 R = 3475 km. Por lo que si R = 3 lunarios y R = 1735,5 km. entonces 1 lunario = 1735,5 km / 3 = 579,1666... km. Problema 9 El viaje de Bilbo Bolson Se necesitan nueve das para cruzar el Bosque Negro. El Hobbit Bilbo Bolson tiene que llevar un mensaje hasta la Desolacin de Smaug, el dragn, en el otro extremo del Bosque Negro, donde no puede encontrar comida, y regresar. El enano Thorin, escudo de Roble, decide acompaarlo. Cada uno slo puede llevar comida para doce das. La comida se puede enterrar y recoger en el camino de regreso. Cmo se puede entregar el mensaje y regresar, de modo que el Hobbit y el Enano no dejen de comer cada da?.

Solucin Posible solucin al problema del Sr. Bilbo Bolson. Tienen que hacer un viaje de 18 das, 9 de ida y 9 de vuelta. Como slo pueden llevar encima comida para 12 das la solucin ser la siguiente. Cada uno sale con las 12 raciones de comida. Durante los tres primeros ds cada uno consume una racin de comida. Al final del tercer ds Thorin da tres de sus raciones a Bilbo (con lo que tiene de nuevo 12 raciones), otras tres raciones las esconde para la vuelta de Bilbo, y las tres reciones restantes se las lleva l mismo para su casa. Cuarto da: Thorin regresa hacia la casa con tres raciones de comida y tres jornadas de viaje. Bilbo contina la marcha con 12 raciones de comida. Cada ds va consumiendo una racin de comida. Una vez que ha llegado al final da el mensaje al dragn y ... comienza el regreso. Contina consumiendo las raciones que porta y cuando se le acaban (quedan tres das de viaje) dispone de las tres raciones que permanecan escondidas, por lo cual ha podido realizar el viaje. Da 1 Da 2 Da 3 Da 4 Da 5 Da 6 Da 7 Da 8 Da 9 B B B T T T B B B Ida Bilbo T T T B B B B B B Vuelta Bolson

T T T T T T T racin perteneciente a Thorn B racin perteneciente a Bilbo

Ida Thorn Vuelta

Problema 10 Hexaedro Celestial Un cubo de 4 cm de arista est pintado de toda su superficie exterior de color azul. Realizando tres cortes horizontales y verticales se obtienen 64 cubitos de 1 cm. de lado. Determinar el nmero de cubitos que tienen respectivamente 3, 2, 1 y ninguna cara azul.

Posible solucin al Hexaedro Celestial: 3 Caras: 8 Cubos, (los que se encuentran en los vrtices de los cubos). 2 Caras: 2 por cada arista ( Hay 12 aristas ) son 24 Cubos 1 Cara: 4 por cada lado del cubo, los cuatro centrales (Hay 6 lados) son 24 Cubos 0 Caras: Los cubos del centro 64 - ( 24 + 24 + 8) son 8 Cubos Problema 11 Problema y medio Se considera el siguiente conjunto ordenado de nmeros: 1/1, 2/1, 1/2, 3/1, 2/2, 1/3, 4/1, 3/2, 2/3, 1/4, ... 1/1992 Determinar el nmero de veces, y en qu posiciones se genera el nmero 1/2 (un medio).

solucin Mi definicin de serie para este problema es: Conjunto de nmeros tal que ... [ n/1, .... , 1/n ] La fraccin (sin simplificar) 1/2 slo aparece una vez; en la segunda serie y en la tercera posicisn absoluta. El nmero 1/2 = 0,5 aparece en 664 ocasiones. Aparece la primera vez en la serie 2 y de ah en adelante, aparece cada 3 series ms, es decir: 5,8,11,... (como se puede ver en la tabla adjunta).
(1) (2) (3) (4) (5) (6) 1/1 2/1, 1/2 (segunda serie, 2* posicin) 3/1, 2/2, 1/3 4/1, 3/2, 2/3, 1/4 5/1, 4/2, 3/3, 2/4, 1/5 (quinta serie, 4* posicin) 6/1, 5/2, 4/3, 3/4, 2/5, 1/5 . . . . . . . . . . . . . . .

El nmero de elementos que forman el conjunto es 1 + 2 + 3 + 4 + ... + 1991 + 1992 = 1.985.028 Las posiciones donde aparecen las ocurrencias son: X0 = 3 Xi = 2 + 9 * i + Xi - 1 para i = 1, 2, ..., 663 por considerar la primera ocurrencia como X0 Problema 12 Parejas perfectamente cuadradas

Magaly invit a nueve nios y nueve nias para su fiesta de cumpleaos. Su mam prepar poleras con los nmeros de los invitados del 1 al 18 y se los reparti a todos los participantes de la fiesta. Durante un baile, la mam observ que la suma de los nmeros de cada pareja eran un cuadrado perfecto. Cmo estaban conformadas las nueve parejas?

Solucin La verdad no s si he entendido muy bien el enunciado, pero ah os envo una posible solucin si el enunciado lo he interpretado bien. En la fiesta haba 9 chicos y 9 chicas. Un total de 18 personas con "poleras". El cuadrado perfecto de los primeros nmeros son los siguientes 1, 4, 9, 16, 25, 36,... Como la suma de los nmeros de cada "polera" tiene que ser un cuadrado perfecto, el nmero mayor que se puede obtener de la suma de dos de estos nmeros es: 17 + 18 = 35. Por lo cual de los posibles cuadrados perfectos que se pueden formar eliminamos el 36 y mayores a ste por no poderse dar. E igualmente eliminamos el 1 por no ser posible en este caso. Con lo cual nos queda los posibles cuadrados perfectos que son vlidos para nuestro problema: 4, 9, 16 y 25. Comenzamos a formar las parejas: Las primeras personas tendrn que ser las que tienen nmeros superiores o iguales a 16 ya que a estas la nica posibilidad que hay es que sumen con otra persona el nmero 25. Por lo cual las primeras parejas sern (7,18), (8,17), (9,16) Las siguientes parejas tendrn que ser (aplicando la misma lgica anterior) en las que est una persona que tenga un nmero mayor o igual 9 ya que a estas la nica posibilidad que hay es que sumen con otra persona el nmero 16. Estas parejas sern: (15,1), (14,2), (13,3), (12,4), (11,5) y (10,6) Con lo que tenemos todas las parejas formadas y sumando todas ellas cuadrados perfectos. Resumiendo, las parejas son: (18,7), (17,8), (16,9), (15,1), (14,2), (13,3), (12,4), (11,5), (10,6) Un saludo y espero haber entendido bien el enunciado del problema. Zubi. Problema 13 La cuenta ilegible En una cuenta ya algo borrosa por los aos transcurridos se puede leer lo siguiente: Por la compra de 72 vacunos se pag ...670... pesos . Si los puntos indican que la primera y ltima cifra del valor pagado son ilegibles, y si cada vacuno tena el mismo valor en pesos, cunto se pag por cada vacuno y cunto se pag en total?

Solucin Suponemos: a670b Descomponemos 72: 72 = 9 * 8 Para que un nmero sea divisible por 8, la suma de la ltima cifra, el doble de las decenas y el cudruple de las centenas tienen que ser igual a un mltiplo de ocho; luego:

b + 28 = 8 n con b < 10; es ms, tiene que ser 0, 2, 4, 6, 8. n tiene que ser 4, puesto que es el nico nmero que hace que b cumpla las condiciones. Resolvemos la ecuacin: b + 28 = 32 con lo que b = 4 Para que el nmero sea divisible por 9, la suma de sus cifras ha de resultar un mltiplo de 9. La ecuacin ser: a + 6 + 7 + 0 + 4 = 9 n ; a = 9 n - 17 a < 10 Para que a sea menor que diez, n tiene que ser igual a 2, luego: a = 18 - 17 ; a = 1 Solucin: El precio total es 16704 pesos. Cada vacuno cost 16704 : 72 = 232 pesos.

Al publicar el problema cometimos el error de escribir "...670..." en lugar de "...679..." que aparece en el original, sin embargo se obtiene una soluciexacta. Por qu? La solucin con la expresin original, "..679.." es que el nmero buscado es 36792 y por cada vacuno se pagaron 511 pesos. Problema 14 El plano y las circunferencias Una circunferencia divide al plano en dos regiones. Dos circunferencias pueden dividir al plano en cuatro regiones. Tres circunferencias pueden dividir al plano en ocho regiones como mximo. Y seis circunferencias?, y diez circunferencias?, y n circunferencias?.

Solucin El problema tiene truco, puesto que con 2 n se cumplen los tres primeros casos, pero a partir de ah hay que buscar otra frmula que se refiere al espacio de n dimensiones. Dibujando me encuentro: n = 1 => Regiones = 2 = 1 * 0 + 2 n = 2 => Regiones = 4 = 2 * 1 + 2 n = 3 => Regiones = 8 = 3 * 2 + 2 n = 4 => Regiones = 14 = 4 * 3 + 2 n = 5 => Regiones = 22 = 5 * 4 + 2 .................. n = 7 => Regiones = 44 = 7 * 6 + 2 Parece que la ley es Circunferencias = n => R = n * (n - 1) + 2 y no 2 n. Es curioso que los tres primeros trminos coincida. Gracias a la inestimable ayuda de Francisco J. Snchez.

Comentario adicional de la G.M.

Consideremos la sucesin 2, 4, 8, 14, ...y establezcamos las diferencias sucesivas: ai Di1 Di2 Di3 2 2 2 0 4 4 2 0 8 6 2 14 8 En donde a i son los trminos de la sucesin, D i 1 las diferencias de primer orden, es decir D 11 = a 2 - a 1 = 4 - 2 = 2 D 21 = a 3 - a 2 = 8 - 4 = 4 ................... D i2 son las diferencias de segundo orden, construdas a partir de las de primer orden de forma anloga a la anterior D 12 = D 21 - D 11 = 4 - 2 = 2 D 22 = D 31 - D 21 = 6 - 4 = 2 ................... Anlogamente llegamos a que todas las diferencias de tercer orden D i3 son nulas y todas las siguientes. Puede demostrarse, aunque no lo haremos, que cualquier elemento de la primera columna, es decir de los trminos de la sucesin, pueden expresarse en funcin de los de la primera fila, es decir, del primer tmino de la sucesin y de las diferencias de primero, segundo,... orden. Dicha expresin es, para nuestro caso:

para a1 = D 11 = D 12 = 2, resulta:

a n = 2 + 2 (n - 1) + (n - 1)(n - 2) = n 2 - n + 2. El rea de un tringulo.

Los tres tringulos MNA, MNA y MNA tienen la misma rea

Desde siempre sabemos que el rea de un tringulo es igual a la mitad del producto de un lado cualquiera por la altura relativa a dicho lado.

Vamos a justificar, en lo que es posible en una pgina web, otras expresiones para obtener el rea de un tringulo; para lo cual slo tendremos que tener algunos conocimientos bsicos de geometra y trigonometra. Dados dos lados y el ngulo comprendido Lados: c y b ngulo: A Si consideramos el tringulo rectngulo AMC resulta h c = b . sen (A) y sustituyendo en la expresin (1) resulta:

Teorema del seno En todo tringulo se verifica la relacin

Dicha relacin puede probarse que es igual al dimetro de la circunferencia circunscrita (es decir, igual a 2R).

Dados un lado y dos ngulos Lado: c ngulos: A, B Conocidos los ngulos A y B es inmediato calcular C C = 180 - (A + B) Como ya sabemos h c = b . sen (A) Adems, segn el teorema del seno

Sustituyendo estos valores en (1) tendremos:

Conocidos los tres lados y el radio de la cincunferencia circunscrita (Que ya es conocer!) A partir del teorema del seno resulta: y sustituyendo en la expresin (2)

Conocidos los tres lados y el radio de la Las bisectrices del tringulo, (en azul) se cortan en el incentro cincunferencia inscrita Las bisectrices dividen al tringulo en tres trigulos que es el centro de la AIB, BIC, CIA de altura r. circunferencia inscrita. El rea de cada uno de ellos es (a.r)/2, (b.r)/2 y (c.r)/2 por lo que el rea del tringulo es

siendo p la mitad del permetro del tringulo.

Conocidos los tres lados. Frmula de Hern En el tringulo AMC h 2c = b 2 - AM 2 Teniendo en cuenta el teorema del cuadrado opuesto a un ngulo agudo resulta:

por lo que Cuadrado del lado opuesto a un ngulo agudo a 2 = b 2 + c 2 - 2.c.AM El cuadrado del lado opuesto a un ngulo agudo es igual a la suma de los cuadrados de los otros dos lados menos el doble producto de uno de ellos por la proyeccin del otro sobre l Es decir Si a + b + c = 2p, siendo p la mitad del permetro, resulta: b + c - a = 2p - 2c = 2(p - a) a + c - b = 2p - 2b = 2(p - b) a + b - c = 2p - 2c = 2(p - c)

Siendo p la mitad del permetro del tringulo. Puede probarse tambin que si p es el

semipermetro de un tringulo y A, B y C son los ngulos del mismo el rea es

La frmula de Hern tambin es vlida si el tringulo es obtuso. Basta entonces aplicar el teorema del cuadrado del lado opuesto a un ngulo obtuso a 2 = b 2 + c 2 + 2.c.AM El cuadrado del lado opuesto a un ngulo agudo es igual a la suma de los cuadrados de los otros dos lados mas el doble producto de uno de ellos por la proyeccin del otro sobre l

Puedes ver una desmostracin de esta expresin en el problema (#057) El rea de un tringulo

Expresin vectorial del rea de un tringulo (En lo que sigue indicaremos las magnitudes vectoriales en negrita). Consideremos el vector AB perpendicular al vector AB y con el mismo mdulo [AB] = [AB] Como h = [AC] sen (x) = = [AC] cos (90 - x) = [AC] cos (y) el rea del tringulo es: Ejemplo. A = 1/2 [AB ] h = Sean A(3,2), B(1,4), C(2,5) los vrtices del = 1/2 [AB] [AC] cos (y) = tringulo ABC. = 1/2 [AB] [AC] cos (y) = El vector AB = (1-3, 4-2) = (-2,2) y el = 1/2 AB. AC vector AC = (2-3, 5-2) = (-1,3) siendo AB. AC el producto escalar de los Un vector perpendicular a AB y con el vectores AB y AC mismo mdulo es AB= (-2,-2) (pues el En general al rea de un trigulo es producto escalar de ambos es 0. A = 1/2 | AB. AC | El producto escalar de AB y AC (referidos es decir, el valor absoluto de dicho producto los vectores a una base ortonormal) es escalar. AB. AC = (-2,-2).(-1,3) = 2 - 6 = - 4 por lo que el rea del tringulo resulta: A = 1/2 |(-2,-2).(-1,3)| = 1/2 | - 4 | = 2 unidades 2 Resolviendo Ecuaciones. Cmo resolver la ecuacin x 4 - 8 x 3 - 36 x 2 + 288 x - 576 = 0

The Bernard Bolzano Pages

Bernard Bolzano filsofo, matemtico y sacerdote catlico checo (1781-1848). Sus ideas liberales y racionalistas lo llevaron a la expulsin de la universidad de Praga (1819) en la que imparta clases de filosofa y religin. Fu de los primeros en presentar definiciones rigurosas sobre funciones continuas y derivables. Su obra pstuma Paradojas del Infinito presenta, entre otras, las definiciones de cantidad, nmero, conjunto finito e infinito. En 1817 presenta un trabajo titulado Una prueba puramente analtica del teorema que establece que entre dos valores donde se garantice un resultado opuesto, hay una raz real de la ecuacin. Dicha prueba analtica se conoce hoy como teorema de Bolzano

El Teorema de Bolzano enunciado con la terminologa actual dice: Si f es una funcin continua en el intervalo [a,b] y f(a) y f(b) toman valores de signo opuesto (es decir, f(a) . f(b) < 0), entonces existe una raz de f en (a,b). (Es decir, existe un punto c del intervalo (a,b) en el que f(c) = 0). En dicho teorema nos vamos a basar para encontrar una raz de una ecuacin. Conviene observar que el teorema de Bolzano es un teorema de existencia, por lo que nos garantiza la existencia de la raz, pero no nos dice cul es. Tambin es importante sealar que el teorema indica que existe una raz en el intervalo, aunque pueden existir ms de una. Por ltimo diremos que todas las hiptesis del teorema son esenciales para la existencia de la raz, incluida la continuidad de f en los extremos del intervalo.

Vamos a disear una estrategia para resolver, mediante el teorema de Bolzano, la ecuacin f(x) = x 2 - 2 = 0 (con lo que tendremos, adems un mtodo para hallar la raz cuadrada de 2). Como f(1) = - 1 < 0 y f(2) = 2 > 0 y f es continua en [1,2], existir una raz de dicha ecuacin en (1,2). Dicha estrategia es muy sencilla; consiste en dividir el intervalo [1,2] incial en dos subintervalos iguales; en uno de los subintervalos la funcin cambiar de signo en sus extremos. Dicho subintervalo volvemos a dividirlo en dos y en uno de ellos, la funcin volver a cambiar de signo y as sucesivamente. Tenemos de esta forma una sucesin de subintervalos encajados cuya longitud tiende a 0. Puede probarse, que las sucesiones formadas por los extremos de los subintervalos convergen a la raz de f. Este procedimiento puede trasladarse facilmente a un programa de ordenador o calculadora. Para tener una bandera de salida basta establecer el clculo de la raz con una cota de error dada. (Por ejemplo que la longitud del intervalo sea menor que un cierto valor). f(x) = x 2 - 2 = 0 [a, b] [1, 2] Signo Signo f(a) f(b) + Punto medio xm 3/2 = 1,5 Signo f(x m) + Longitud intervalo 1

[1, 3/2] [5/4, 3/2] [11/8, 3/2] [11/8, 23/16] [45/32, 23/16] [45/32, 91/64] [181/128, 91/64]

+ + + + + + +

5/4 = 1,25 11/8 = 1,375 23/16 = 1,43750 45/32 = 1,40625 91/64 = 1,42188 181/128 = 1,41406 363/256 = 1,41797

+ +

1/2 (0,5) 1/2 2 (0,25) 1/2 3 (0,125) 1/2 4 (0,0625) 1/2 5 (0,03125) 1/2 6 (0,01563) 1/2 4 (0,00781)

Conviene resaltar lo lenta que es la convergencia hacia la solucin buscada.

Mtodo de Newton Un algoritmo sencillo y de, en ciertas condiciones, rpida convergencia para obtener la raz de una ecuacin, es el mtodo de Newton-Raphson, o simplemente mtodo de Newton. La idea intuitiva es muy fcil. Si hacemos una estimacin inicial, por ejemplo x 0, de la raz c, y trazamos en T (x 0, f(x 0)) la tangente a f, sta cortar al eje de abscisas en x 1 que es una mejor estimacin de c que la inicial. Repitiendo el proceso tendremos una sucesin x0, x1, x2..., xn que puede probarse que, si converge, lo hace a c. f (x) = x 2 - 2 Clculo de los x k La ecuacin de la tangente a f en T viene dada por f(x) - f(x 0) = f(x 0)(x - x 0) y la interseccin de ella con el eje de abscisas, si designamos dicha interseccin por (x 1,0) resulta - f(x 0) = f(x 0)(x 1 - x 0), de donde xn 3/2 0,25 f '(x) = 2x f '(x n) 3 x n+1 1,416667 f (x n)

1,416667 0,006944 2,833333 1,414216 1,414216 0,000006 2,828431 1,414214 Observaciones:

Reiterando el proceso para x1, x2,... obtenemos la expresin general:

Es preciso realizar una buena estimacin inicial, es decir elegir un x0 prximo a la raz de f. Claro que si no sabemos la raz ... (pero el teorema de Bolzano puede ayudar para localizarla). Como aparece f(x n) en el denominador, el mtodo fallar si algn valor lo anula; el problema

No siempre converge la sucesin x 0, x 1, x 2, ... x n, como puede comprobarse con la funcin f (x) = x 1/3 Entonces tenemos que x n + 1 = - 2 x n

puede soslayarse con la eleccin de otra estimacin inicial. Es preciso tener en cuenta unas mnimas condiciones de continuidad y derivabilidad de f y f ' como las funciones polinmicas las cumplen, no insistiremos sobre el tema. Una convergencia realmente rpida, S seor!, por algo, dicho algoritmo es utilizado por varias calculadoras para el clculo de la raz cuadrada.

Resolviendo la ecuacin f(x) = x 4 - 8 x 3 - 36 x 2 + 288 x - 576 = 0 Como f cumple las condiciones de continuidad y derivabilidad, mediante el teorema de Bolzano, obtenemos f(9)= -171 < 0 y f(10)= 704 > 0, luego en el intervalo (9,10) existe (al menos) una raz real. En lugar de realizar los clculos vamos a utilizar una calculadora Texas Instruments TI-82 para efectuarlos. Los pasos que seguimos estn indicados en la columna de la derecha

1: Introducimos en el editor de ecuaciones la expresin de f y de su derivada Y1= x 4 - 8 x 3 - 36 x 2 + 288 x - 576 Y2= 4x 3 - 24x 2 - 72 + 288 2: Asignamos a la variable alfanumrica A el valor inicial 19/2 19/2 A 3: Escribimos la expresin de x n + 1 particularizada para la funcin dada y la almacenamos en A (A - (Y1(A)/Y2(A)) A 9.272839 4: Cada vez que pulsamos Enter obtenemos 9.254112

9.253989

9.253989

La ecuacin propuesta tiene otra raz real. Cul? La otra solucin, facilitada por Paloma Pascual, es

-6,490345. Polgonos Estrellados. Si al dividir una circunferencia en partes iguales unimos los puntos de divisin de dos en dos, de tres en tres, etc. y al cerrarse la poligonal hemos recorrido la circunferencia un nmero entero de veces, obtenemos un polgono regular estrellado. Puede probarse que para obtener un polgono regular estrellado de n lados (la circunferencia estar dividida en n partes iguales) uniendo las divisiones de a en a, es necesario (y suficiente) que a y n sean primos. Como unir divisiones de a en a es igual que dividirlas de n - a en n - a (es decir de a en a en sentido contrario), se podrn construir polgonos estrellados considerando los nmeros menores que n/2, que sean primos con n.

Pentgono regular estrellado El nmero primo con 5 menor que 5/2 es 2; podemos construir el pentgono estrellado uniendo las divisiones de dos en dos. Obtenemos de esta forma el ms popular de los polgonos estrellados y, posiblemente, el emblema de la escuela pitagrica. En l el nmero ureo aparece por doquier.

No existen polgonos estrellados de 6 lados, ya que no existe ningn nmero primo con 6 menor que 6/2.

Heptgonos regulares estrellados Existen dos nmeros primos con 7 menores que 7/2, el 2 y el 3. Podemos, por tanto, construir dos heptgonos regulares estrellados uniendo las divisiones de 2 en 2 y otro de 3 en 3.

Octgono regular estrellado 3 es el nico nmero primo con 8 menor que 8/2. Uniendo las divisiones de 3 en 3 obtenemos el octgono regular estrellado.

Enegonos regulares estrellados 2 y 4 son primos con 9 menores que 9/2. Podemos construir dos polgonos regulares estrellados de 9 lados uniendo las divisiones de 2 en 2 y de 4 en 4.

Decgono regular estrellado Por ltimo, uniendo de 3 en 3 obtenemos el decgono regular estrellado. En l tambin "aparece" el nmero ureo.

El Pentagrama y el Nmero ureo El lema de la Escuela Pitagrica fue todo es nmero y su emblema el pentagrama o pentgono regular estrellado. En el pentgono estrellado figura el nmero ureo infinidad de veces. Veamos qu relacin existe entre el pentgono regular y el pentgono regular estrellado. Si consideramos el lado del pentgono la unidad, basta aplicar el teorema del coseno al tringulo ABC y resulta que AC es igual al nmero ureo. El teorema del coseno afirma que en todo trigulo un lado al cuadrado es igual a la suma de los cuadrados de los otros dos lados menos el doble producto de ellos

por el coseno del gulo comprendido. En nuestro caso, aplicando dicho teorema al tringulo ABC, tendremos: AC 2 = AB 2 + BC 2 - 2 AB. AC. cos (108) y como AB = BC = 1, efectuando operaciones resulta: AC 2 = 2 - 2 cos (108) Extrayendo la raiz cuadrada: AC = 1,6180340... Considerando el lado del pentgono regular la unidad, (AG = 1), pueden obtenerse de forma inmediata las siguientes expresiones:

Qu pudo hacer que los pitagricos sintieran tanta admiracin por el nmero ureo ?. Casi con toda seguridad, para la escuela pitagrica la consideracin del irracional 5 1/2, de cuya existencia tuvieron conciencia antes que de 2 1/2, tuvo que causar una profunda reflexin en las teoras de la secta. Si tienes alguna duda de las relaciones del nmero ureo con el pentgono estrellado ... mira!, y as hasta el infinito. Siempre que encuentres un pentgono regular podrs hacer lo mismo.

Dado un segmento AB, se dice que est dividido en media y extrema razn, cuando: "[...] si hay de la parte pequea a la parte grande la misma relacin que de la grande al todo" (Vitrubio). A partir del Renacimiento recibi el nombre de Divina Proporcin.

La Proporcin urea fascin como ideal de belleza a los griegos, a los renacentistas y perdura en nuestros das. Los pintores y escultores del Renacimiento la tuvieron muy en cuenta ... y tambin los impresores. En el grfico de la izquierda se puede apreciar el diseo de la caja y los mrgenes de un libro segn la normas de la Divina Proporcin. En el de la derecha aparece la reproduccin de un incunable impreso en Venecia (1495), segn dichas normas. Se trata del libro de Pietro Bembo De Aetna (Sobre el Etna). Exquisita tipografa romana, calidad de papel y tinta, proporciones divinas. Una joya.

Potencias del Nmero ureo

Ms sobre polgonos estrellados Relacin entre los lados del decgono regular convexo y el estrellado Sean AB y AD los lados de los polgonos regular convexo y el estrellado respectivamente. Los ngulos ABG y AMB son iguales pues el primero es un ngulo inscrito en la circunferencia que vale (la mitad del arco que abarca) 72 y el segundo es un angulo interior cuyo valor es

Igualmente los ngulos FAD y AOB son iguales y valen 36; por otra parte los tringulos DAO y AOM son semejantes por lo que

Como AM = AB y DO = AO = r resulta

De la primera igualdad tenemos r 2 = AB AD es decir, el radio de la circunferencia circunscrita es medio proporcional entre ambos lados. Conocido el radio podemos hallar ambos lados, para lo cual dividimos el radio en media y extrema razn, es decir en la proporcin urea. NOTA Podemos construir grficamente, muy facilmente, dado el radio de la cincunferencia circunscrita, ambos lados. Trazamos una circunferencia que tenga NM = radio como dimetro. Por el extremo M levantamos una perpendicular MT = NM y unimos T con el centro de la circunferencia determiando sobre ella los puntos P y Q. Entonces TP y TQ son los lados de cada uno de los polgonos. En efecto, pues NM 2 = TM 2 = TP TQ

Cnicas.
La filosofa est escrita en ese grandsimo libro abierto ante los ojos; quiero decir, el universo; pero no se puede entender si antes no se aprende a entender la lengua, a conocer los caracteres en los que est escrito. Est escrito en lengua matemtica y sus caracteres son tringulos, crculos y otras figuras geomtricas, sin las cuales es imposible entender ni una palabra; sin ellos es como girar vanamente en un oscuro laberinto. Galileo Galilei

De las dos obras de Apolonio de Prgamo (262190 a. d.C), Secciones en una razn dada y Cnicas, la zltima es, junto con los Elementos de Euclides, la obra ms importante de la geometra griega. Apolonio, Euclides y Arqumedes elevaron la geometra a niveles tales que su vigencia permanece en nuestros das. Las Cnicas constaban de ocho libros. Los cuatro primeros se han conservado, los tres siguientes se conservan mediante la traduccin al rabe y se ha perdido el octavo. Esta obra es el resultado de estudiar todas las particularidades de unas secciones a las que denomin cnicas. Apolonio descubri que se obtenan al cortar mediante una superficie plana un cono circular en diversas posiciones. Depende de cmo se corten, las secciones resultantes sern crculos, elipses, hiprbolas o parbolas. Aunque estos conceptos no tuvieron posibilidad de ser aplicados a la ciencia de su poca, su importancia ha quedado plenamente justificada con el paso del tiempo.

Galileo Galilei (1564-1642), estudiando el movimiento de un proyectil, con una componente horizontal uniforme y una vertical uniformemente acelerada, lleg a la conclusin que dicha trayectoria, despreciando la resistencia del aire, es una parbola. Galileo cambi el concepto que durante la Edad Media, se tena sobre la trayectoria de un proyectil.

siendo g la gravedad, v la velocidad inicial de la bala y alfa la inclinacin del tiro. Galileo estableci a partir de la expresin anterior la inclinacin para alcanzar la mxima distancia (x).

En 1609 Johannes Kepler (15711620) publica, utilizando las observaciones de su maestro Tycho Brahe, su obra Astronoma Nova en donde enuncia las dos primeras leyes referente a las bitas de los planetas. Posteriormente, en 1619, en el libro Harmonices, Mundi, Libri Kepler publicara la tercera.

Primera Ley Los planetas describen rbitas elpticas en uno de cuyos focos est el Sol. Segunda Ley Las reas barridas por la recta que une el sol con el planeta son directamente proporcionales a los tiempos empleados en barrerlas.

Aproximadamente 80 aos ms tarde, Isaac Newton (1642-1727) probaba que las rbitas elpticas de los planetas implicaban la ley de gravitacin universal. El concepto de cnica aparece no slo en las trayectorias de planetas y proyectiles, sino tambin en trayectorias de partculas atmicas elementales. La ley de los gases perfectos enunciada por el fsico irlands Robert Boile (1627-1691) dice: "A temperatura constante el producto P V = k". Una hiprbola equiltera.

(Si las reas dibujadas son iguales, entonces la velocidad del planeta es mayor en el perihelio que en el afelio) Tercera Ley Los cuadrados de los perodos de revolucin son proporcionales a los cubos de los semiejes mayores de las rbitas.

La relacin existente entre la distancia al origen del foco y el semieje mayor se denomina excentricidad de la cnica. En la elipse la excentricidad est comprendida entre 0 y 1 Si es 0, entonces la elipse es una circunferencia.

Estas son las excentricidades de las rbitas de los planetas del Sistema Solar

Mercurio Venus Tierra Marte Jpiter Saturno Urano Neptuno Plutn 0,206 0,007 0,017 0,093 0,043 0,051 0,046 0,004 0,250
La excentricidad de rbita de la Luna, con respecto de la Tierra es de 0,055. El famoso cometa Halley, que nos visitar prximamente en el ao 2061, tiene una rbita elptica, con foco en el Sol, cuya excentricidad es de 0,967 (aproximadamente).

La superficie engendrada al girar una parbola alrededor de su eje es una superficie parablica. Dichas superficies tienen la propiedad de ser reflectoras. Situado un punto luminoso en el foco, los rayos se proyectan paralelos al eje, y recprocamente, los rayos que inciden paralelos al eje, se concentran en el foco. Estas superficies son las nicas que gozan de esta propiedad. El primer reflector parablico de un faro de mar fue construido por William Hutchinson en 1752. La idea de un reflector parablico se difundi rpidamente y en la actualidad lo encontramos en faros de bicicletas, coches, proyectores de teatros, radares, antenas parablicas, etc.

Recomendaciones en la Red

Kepler"s Laws Edmond Halley & Johannes Kepler: Pioneers in Astronomy Galileo and the Inquisition The Galileo Project Biography of Galileo Galilei (Muy buena. Con versin en italiano

historia actual de la enseanza de las matemticas en Espaa hubiese sido muy distinta sin la aportacin de las Sociedades de Profesores de Matemticas surgidas en los ltimos aos. En 1975 en la revista Escuela 75, editada por el Colegio Oficial de Doctores y Licenciados de Valencia, en un artculo titulado Para qu las Matemticas? un grupo de profesores pona en cuestin "[...] los planteamientos habituales en la enseanza de las Matemticas". El artculo estaba firmado por los profesores que posteriormente formaran el Grupo Cero. De la colaboracin de los anteriores y un grupo de profesores catalanes surge el Grupo Zero en Catalua.

(F.E.S.P.M.) Federacin Espaola S.A.E.M. THALES de Sociedades de Profesores de Matemticas La influencia de estos dos grupos en el resto de Espaa, mediante grupos de trabajos, conferencias, publicaciones, etc. fue bastante importante y puede considerarse que son el origen de las asociaciones

posteriores. Estos grupos iniciales "[...] denunciaron el raquitismo de la enseanza tradicional de unos ejercicios estriles aportando problemas con significado y tiles [...] (y) valorando la iniciativa del alumno [...] dando un sentido ms integral e interdisciplinar a la enseanza de nuestra asignatura" (A. Prez Actas IV Jornadas Andaluzas) Fu necesario, para rentabilizar esfuerzos, formar asociaciones que agruparan el mayor nmero de profesores, de forma que estas ideas -no todas nuevas desde luego- no se dispersaran y pudieran ser conocidas por el mayor nmero de profesionales. Con esta intencin surgen las Asociaciones de Profesores de Matemticas

Sociedad Matemtica de Profesores de Cantabria En 1978 se crea la Sociedad Canaria de Profesores de Matemticas "Isaac Newton"; en 1981 la Sociedad Andaluza de Profesores de Matemticas "Thales" y la Sociedad Aragonesa "Pedro Ciruelo". En 1984 nace la Asociacin Andaluza de Profesores de Matemticas que se fusionara con "Thales" constituyendo la Asociacin Andaluza de Profesores de Matemticas "Thales" que posteriormente sera la Sociedad Andaluza de Educacin Matemtica "Thales". En 1987 se inicia el proceso de federar estas asociaciones culminando en 1989 con la creacin de la Federacin Espaola de Sociedades de Profesores de Matemticas. A partir de entonces surgen la Sociedad Castellana, la navarra Tornamira, la Asociacin Puig Adam, la extremea Ventura Reyes Prsper, la madrilea Emma Castelnuovo, la gallega Enciga, etc. La labor de todas ellas ha sido importantsima en los ltimos aos procurando mediante publicaciones, cursos, revistas, centros de documentacin, etc. promover la mejora y aprendizaje de las matemticas, adems de promocionar las olimpiadas matemticas. Como reconocimiento internacional de esta

Sociedad Asturiana de Educacin Matemtica

labor un botn. El 7 de abril de 1991 la Comisin Ejecutiva de la Comisin Internacional de Educacin Matemtica (ICMI) acepta la candidatura de Sevilla para el 8 I.C.M.E. (8 Congreso de Educacin Matemtica). Por primera vez, durante el mes de agosto de 1996, organizado por Thales, se celebr en un pais de habla espaola este congreso.

Societat dEducaci Matemtica AlSociedad CastellanoKhawarizmi Manchega de Profesores (Comunitat de Matemticas Valenciana)

Organizacin Espaola para la Coeducacin Matemtica Ada Byron

Sociedad de Educacin Matemtica de la Regin de Murcia

APRIMA. Sociedad Riojana de Profesores de Matemticas

Asociacin Galega de Profesores de Educacin Matemtica

Federaci d'Entitats Associaci de Professors per l'Enseyament de de Matemtiques de les Sociedad Castellano les Matemtiques a Comarques Meridionals. Leonesa de (APMCM) Catalunya. Profesores de Matemticas

Sociedad Extremea de Educacin Matemtica

"Ventura Reyes Prsper"

Sociedad Puig Adam de Profesores de Matemticas Otras asociaciones

Sociedad de Ensinantes de ciencia de Galicia (ENCIGA) Sociedad de Profesores de Matemticas "Tornamira" Sociedad Aragonesa de Profesores de Matemticas "Pedro Snchez Ciruelo"

Proporcin cordobesa En diversos trabajos de investigacin (de arquitectura, sobre pintura, etc.) aparece un rectngulo que no est en la proporcin urea, sino que la relacin entre sus lados es de 1,3... (Sin ir ms lejos, si la resolucin de tu ordenador es de 800x600, se encuandra en la misma proporcin) Si el nmero ureo puede establecerse como la relacin existente entre el lado del decgono regular y el radio de la circunferencia circunscrita al mismo, pareci lgico buscar una relacin de la misma naturaleza con la que dicha proporcin quedara geomtricamente fundamentada. La misma qued establecida al obtener la proporcin buscada como la relacin entre el radio de la circunferencia circunscrita al octgono regular y el lado de ste. Cualquier matemtico, o buen aficionado, sabe que esta relacin es:

Dicho cociente es c = 1,306562964 ... que se conoce como nmero cordobs Al ser ms fcil construir un octgono regular que un pentgono, dicha proporcin se extendi rpidamente quedando de manifiesto en mltiples obras pictricas y arquitectnicas. Como ejemplos podramos citar la bveda cordobesa, y nada digamos de las bellas arcadas de la mezquita de Crdoba. Segn los trabajos del alemn Fechner esta proporcin se establece en multitud de obras pictricas. Para el arquitecto Rafael de la Hoz Arderius (uno de los mximos investigadotres del tema) considerando las ltimas tcnicas de medicin obtenidas del Papiro Rhind (museo Britnico) entre las diagonales de un rectngulo con dicha proporcin queda perfectamente encajada la Gran Pirmide Y como los anteriores podramos citar ms ejemplos. Los estudios efectuados sobre el tema indican que la proporcin dicha est ms extendida de lo que hasta ahora se crea.

Consideremos la circunferencia de radio R. Si trazamos la bisectriz del primer cuadrante, el segmento NP = X es el lado del octgono regular inscrito en dicha circunferencia. Aplicando el teorema de Pitgoras al tringulo NOM resulta que (MN)2 = R2 + R2 por lo que

Por simetra OP' = MN/2 (ya que NP'= MN/2 y OP' = P'N) Como QNP es recto, aplicando el teorema del cateto resulta: X /QP = P'P/ X de donde X2 = QP. P'P = 2R (OP - OP') Fundamentos geomtricos sobre el octgono regular es decir:

De esta expresin deducimos (considerando la circunferencia de radio unidad, radio R = 1) que:

Co nstru Basta con trazar una circunferencia y la bisectriz del cci primer cuadrante. RT es un lado del rectngulo y el radio n del de la circunferencia el otro. rect ngul o cord obs.

Teniendo en cuenta el apartado anterior es muy fcil construir un rectngulo en la proporcin cordobesa.

Consideramos el segmento unidad y trazamos una circunferencia de radio (2)1/2 Determinacin sobre la recta La bisectriz del ngulo MOM' corta a dicha circunferencia real del nmero cordobs C en C'. Proyectando sobre la recta real obtenemos C.

En efecto

En el tringulo OCC'

Observa que

es la expresin trigonomtrica del nmero cordobs.

Dado un segmento MN pretendemos encontrar un x, interior a MN que verifique (MP)/(MN) = C Si MP = x y PN = 1 - x resulta:

Dividir un segmento dado en la proporcin cordobesa

Basta pues dividir el segmento dado proporcionalmente a c y a (1 + c)

Por ltimo proponemos algunas cuestiones para el lector interesado:


(1) Hallar las soluciones de la ecuacin 2x4 - 4x2 + 1 = 0 (2) Cmo podramos relacionar el rectngulo cordobs y el ureo?

Medallas Fields Parece ser que cuando D. Alfredo Nobel pregunt a sus asesores quien podra ser premio Nobel de Matemticas y le contestaron que posiblemente el matemtico sueco Gsta Mittang-Leffer, Nobel respondi: "No habr premio Nobel de Matemticas". Las malas lenguas de la poca aseguraban que las relaciones entre Don Alfredo y Don Gsta no eran demasiado buenas. En el Congreso Internacional de Matemticas de 1924, presidido por John Charles Fields, se present la propuesta de unas "medallas internacionales para destacados descubrimientos matemticos". Cada cuatro aos se otorga este premio a dos

Mittag-Leffer (1846-1927)

matemticos menores de 40 aos y, a partir de 1966, debido a la gran y buena produccin matemtica, a un mximo de seis. Dichas Medallas constituyen el Premio Nobel de las Matemticas. Charles Fields (1863-1932) Las medallas estn acuadas en oro. En el anverso, aparece la inscripcin latina "TRANSIRE SVVM PECTUS MVNDOQUE POTIRE" (sobrepasar su propio entendimiento y apoderarse del mundo) junto al busto de Arqumedes y su nombre en griego. En el reverso, figura la inscripcin "CONGREGATI EX TOTO ORBE MATHEMATICI OB SCRIPTA TRIBVERE" (reunidos los matemticos de todo el mundo para premiar las obras maestras), junto con el dibujo de la famosa inscripcin del cilindro y la esfera inscrita del gran Arqumedes. Las medallas fueron diseadas por el escultor canadiense Dr. Robert Tait McKenzie y las inscripciones redactadas por el profesor G. Norwood de la Universidad de Toronto.

Las medallas Fields se conceden, desde 1932 cada 4 aos Hasta el momento se han concedido 42 (1998) Desde 1936 hasta 1950 no se concedieron debido a la II Guerra Mundial La edad media de los premiados es de 34 aos (34,61)

GANADORES MEDALLAS FIELDS 1936 Lars Ahlfors - (29; Finlandia) Jesse Douglas - (39; USA) 1950 Laurent Schwartz - (35; Francia) Atle Selber - (33; Noruega) 1954 Kunihiko Kodaira - (39; Japn) Jean-Pierre Serre - (27; Francia) 1958 Klaus Roth - (32; Alemania) Rene Thom - (35; Francia) 1962 Lars Hormander - (31; Suecia) John Milnor - (31; USA) 1966 Michael Atiyah - (37; UK) Paul Cohen - (32; USA) Alexander Grothendieck - (38; Alemania) Stephen Smale - (36; USA)

1970 Alan Baker - (31; UK) Heisuke Hironaka - (39; Japn) Serge Novikov - (32; Rusia) John Thompson - (36; USA) 1974 1978 Enrico Bombieri - (33; Italia) David Mumford - (37; UK) Pierre Deligne - (33; Blgica) Charles Fefferman - (29; USA) Gregori Margulis - (32; USSR) Daniel Quillen - (38; USA) Alain Connes - (35; Francia) William Thurston - (35; USA) Shing-Tung Yau - (33; Hong Kong) 1986 Simon Donaldson - (27; UK) Gerd Faltings - (32; Alemania) Michael Freedman - (35; USA) Vladimir Drinfeld - (36; USRR) Vaughan Jones - (38; Nueva Zelanda) Shigefumi Mori - (39; Japn) Edward Witten - (38; USA) Pierre Louis Lions - (38; Francia) Jean Christophe Yoccoz - (36; Francia) Jean Bourgain - (40; Blgica) Efim Zelmanov - (39; Rusia) 1998 Maxim Kontsevich - (34; Rusia) Richard E. Borcherds - (39; Sudfrica) William Timothy Gowers - (33; UK) Curtis T. McMullen - (38; USA)

1982

1990

1994

MEDALLAS FIELDS 2002 El pasado 20 de agosto durante el 2002 I.C.M. (International Congress of Mathematics) celebrado en Beijing (China) se concedieron las Medallas

Fields La primera fue concedida a Laurent Lafforgue del Hautes tudes Scientifiques in Bures-sur-Yvette y la segunda a Vladimir Voevodsky del Institute for Advanced Study (Princeton). La Unin Matemtica Internacional concedi en dicho acto el Nevalinna Prize a Madhu Sudan del M.I.T.

Por nacionalidades se distribuyen de la siguiente manera USA 11 Francia 6 UK 5 Alemania 3 Japn 3</SP Rusia 3 Blgica 2 USSR 2 Finlandia 1 Hong Kong 1 Italia 1</SPANS< td> Noruega 1 Nueva Z. 1 Suecia 1 Sudfrica 1 Los Versos de Oro. Los Versos de Oro resumen la doctrina filosfica del filsofo de Samos. 1. Honra a los dioses inmortales del modo establecido por la ley. 2. Venera el juramento y tambin a los nobles hroes. 3. Y lo mismo a los genios subterrneos, de acuerdo con los ritos tradicionales. 4. Honra a tu padre y a tu madre as como a tus parientes. 5. Haz tu mejor amigo a quien sobresalga por sus virtudes. 6. S amable con tus palabras y til con tus obras. 7. No te enojes por las faltas leves que cometan tus amigos.

8. Acta segn tus facultades, teniendo en cuenta que el poder est muy cerca de la necesidad. 9. Aprende que, por una parte, las cosas son as; y por otra, acostmbrate a dominar lo siguiente: 10. Primero el estmago y despus el sueo, el impulso sexual y la ira. 11. No cometas ninguna accin vergonzosa 12. Con otro ni a solas, porque, ante todo, te debes respetar a t mismo. 13. S justo en palabras y actos 14. Y razonable y sensato en todo lo que hagas. 15. No olvides que la muerte es el destino de todos. 16. Y que es condicin de la fortuna aumentar y disminuir. 17. Los sufrimientos que la suerte proporciona a los hombres proceden de los dioses. 18. Soporta tu destino sin indignarte. 19. Aunque es conveniente que corrijas este destino segn tus facultades. 20. Ten presente que el destino no da ms sufrimiento a los buenos. 21. De las muchas palabras que pronuncian los hombres, unas son buenas y otras malas. 22. Que ellas no te turben ni ejerzan influencia sobre ti. 23. Soporta con paciencia y dulzura la mentira. 24. Procura cumplir siempre lo que te voy a decir ahora: 25. Que nadie, ni con palabras ni con actos, 26. Te convenza de que debes hacer o decir lo que no sea mejor. 27. Reflexiona antes de cometer una accisn estulta 28. Pues es propio de los hombres decir palabras necias y ejecutar actos malos. 29. Realiza ahora lo que no pueda perjudicarte despuis. 30. Abstente siempre de lo que no conozcas. 31. Aprende todo lo necesario para que tu vida sea ms feliz. 32. No conviene que descuides la salud de tu cuerpo 33. Para lo cual procurars descubrir la justa medida en comidas, bebidas y ejercicios fsicos. 34. Entiende por justa medida la que no te cause dolor. 35. Acostmbrate a llevar una vida pura, limpia y viril. 36. Procura no hacer nada que pueda traer la envidia sobre ti. 37. No gastes insensatamente, como los que ignoran la honesta proporcin de lo bello; 38. Pero tampoco seas avaro. Lo mejor en todo es la justa medida. 39. Haz lo que no te perjudique, pero reflexiona antes de obrar. 40. No permitas que el dulce sueo cierre tus ojos 41. Sin haber repasado contigo mismo lo que hayas hecho durante el da. 42. ?En qu he faltado? ?Qu he hecho? ?He omitido alguna obligacin?. 43. Repasa tambin todas las acciones que hayas realizado,empezando por la primera y sin olvidar ninguna. 44. Reprndete si has cometido algn acto malo y recocjate con los buenos. 45. He aq lo que debes hacer. He aqu la tarea que reclama tu cuidado. 46. He aqu lo que debes amar. He aqu lo que te encaminar por la senda divina. 47. Antes de empezar cualquier tarea 48. Pide a los dioses que santifiquen tu esfuerzo. 49. Si pones en prcticas estas normas, conocers los lazos que une a los dioses inmortales con los hombres mortales 50. Y aprenders a aconocer los elementos que pasan y los que permanecen. 51. Y conocers, como es justo que se conozca, que la Naturaleza es una y semejante en todo.

52. Y as no esperars lo que no puede esperarse, ni habr secreto alguno para ti. 53. Y sabrs tambin que los hombres padecen los males que ellos escogen 54. Porque son tan desgraciados que no ven los bienes que estn a su lado. 55. Ni los oyen, porque son muy pocos los que saben librarse del mal. 56. Tal es el destino que ciega su mente. Como cilindros que ruedan 57. Van de un sitio para otro padeciendo males infinitos, 58. Impotentes para reconocer la discordia funesta que les es innata, 59. A la que no voy a provocar, sino esquivarla huyendo de ella. 60. Padre Zeus; t podras liberar a los hombres de imnumeralbles males, 61. Mostrando a cada uno el genio que lo gua. 62. Y en cuanto a ti, hombre, ten confianza, porque la raza de los mortales es de origen divino, 63. Y su naturaleza sagrada le revela todas las cosas. 64. Practicando lo que te ordeno, disfrutar de sus beneficios 65. Y en cuanto sea curada tu alma quedars libre de todos los males. 66. Evita los alimentos indicados en los libros de las Purificaciones y de la Salvacin del alma. 67. Sin embargo, reflexiona sobre cada cosa 68. Tomando como gua del carro de tu alma la recta razn. 69. Y una vez que te hayas liberado de tu envoltura carnal, irs al ter impalpable 70. Y sers inmortal: un dios incorrupto en vez de mortal.

Nmeros primos Los primeros nmeros primos son: 2, 3, 5, 7, 11, 13, 17, 19, 23, 29, 31, 37, 41, 43, 47, 53, 59, 61, 67, 71, 73, 79, 83, 89, 97, 101, 103, 107, 109, 113, 127, 131, 137, 139, 149, 151, 157, 163, 167, 173, 179, 181, 191, 193, 197, 199, 211, 223, 227, 229, 233, 239, 241, 251, 257, 263, 269, 271, 277, 281, 283, 293, ... El mayor nmero primo conocido hasta el momento es: 2 3021377 - 1 (Abril de 1998) Son nmeros primos los que slo son divisibles por si mismos y, evidentemente, por la unidad.( Una definicin equivalente dice que un nmero a es primo sii slo tiene cuatro divisores (-1, 1, a, -a). El nico nmero primo par es el 2. Todo nmero primo (excepto el 2) es impar. Pero el recproco no es cierto Ya Euclides, en los Elementos [IX.20], prueba que la sucesin de nmeros primos es infinita ("Hay ms nmeros primos que cualquier cantidad propuesta de nmeros primos"). Razon de la siguiente manera: Supongamos todos los nmeros primos menores o igual a uno dado P. Consideremos el nmero entero N = 2.3.5.7...P + 1 Evidentemente N es mayor que P Al dividir N entre 2 el cociente ser 3.5.7...P y de resto sobrar 1. Igual pasar al dividir N entre 3, el cociente ser 2.5.7....P y de resto sobrar 1. Y as sucesivamente siempre que dividamos entre 2, 3, 5, ... P, de resto obtendremos 1 Consideremos dos posibilidades: N es primo o no lo es Si es un nmero primo, ser un primo mayor que P. Si no

es un nmero primo se puede descomponer en factores primos [VII,31], pero ninguno de dichos factores podr ser 2, 3, 5, ... P Por consiguiente ha de haber un nmero primo mayor que P. Que es lo que haba que hacer. Hasta n = 40 puedes obtener Sera bueno tener una fmula para obtener nmeros primos ... pero no existe (hasta el momento). nmeros primos. Para n = Fermat (en 1640) crey que haba encontrado una con la 41, es evidente que no. expresin 2k + 1, con k = 2n pero Euler, casi un siglo despus se lo estrope, cuando prob que para n = 5, el nmero n2 - 79 n + 1601 obtenido, 4.294.967.297, no es primo, sino el producto de 6.700.417 x 641 Hasta n = 79 se obtienen nmeros primos, pero falla para n = 80 Algunas expresiones para obtener nmeros primos son: n2 - n + 41

Y en esas estamos. En la red te recomendamos


La pgina personal de Carles Pina (En Castellano, Cataln e Ingls) Prime Numbers The Prime Pages prime numbers research, records, and resources

Humor biolgico matemtico. Las bacterias se multiplican dividindose Me di cuenta de que iba a suspender matemticas cuando un da el profesor dijo en clase: "Sea un epsilon menor que 100", y todo el mundo se ech a reir. Un ingeniero, un fsico y un matemtico se presentan a un examen. La nica pregunta es cunto es 1 + 1 El ingeniero escribe unos momentos en su papel y entrega: 1+1=2 A los cinco minutos entrega el fsico y escribe 2 (dos) El matemtico sigue escribiendo y solicita media hora ms de tiempo para terminar. Al cabo de la misma el tribunal le recrimina, Pero hombre, entregue ya!, y el matemtico dirigindose al tribunal le dice: Lo siento, slo he podido demostrar que la solucin existe y es nica

Evaluacin en la E.S.O. Acaba de llegar a mi poder este valioso documento, firmado por Inocencio Docente, y tal como lo recibo, os lo muestro

Comentario de la evaluacin 1. La grafa del signo seis es del todo correcta. 2. Se puede apreciar lo mismo con el siete. 3. El signo ms nos dice, acertadamente, que se trata de una suma. 4. En cuanto al resultado vemos que el uno es correcto. El segundo nmero, efectivamente, no es ocho. Bueno, si lo cortamos por la mitad de arriba abajo, observamos que el alumno ha escrito dos treses simtricos. Elegimos el bueno porque se ve que su intencin era buena.

Evaluacin El conjuntode estas observaciones evidencia que: (a) La actitud del alumno es positiva (lo intent) (b) Los procedimientos son correctos (los elementos estn ordenados correctamente). (c) En conceptos slo se equivoc parcialmente en uno de los seis elementos que forman el ejercicio. Esto es casi de sobresaliente. En Consecuencia podemos otorgarle un "Notable" y decir que "Progresa Adecuadamente"

Evolucin de un mismo problema matemtico a travs de los cambios a que ha estado sometido el Bachillerato en los ltimos aos.

Enseanza 1960 Un campesino vende un saco de patatas por 1000 pesetas. Sus gastos de produccin se elevan a los 4/5 del precio de venta. Cul es su beneficio?. Enseanza Tradicional 1970 Un campesino vende un saco de patatas por 1000 pesetas. Sus gastos de produccin se elevan a los 4/5 del precio de venta, esto es, a 800 pesetas. Cul es su beneficio?. Enseanza Moderna (LGE) Un campesino cambia un

conjunto "P" de patatas por un conjunto "M" de monedas. El cardinal del conjunto "M" es igual a 1000 pesetas y cada elemento de "M" vale una peseta. Dibuja 1000 puntos gordos que representen los elementos del conjunto "M". El conjunto de los gastos de produccin, es decir 200 puntos gordos menos que el conjunto "M", es el conjunto "F". Representa el conjunto "F" como subconjunto del conjunto "M" y responde a la siguiente pregunta: cul es el cardinal del conjunto "B" de los beneficios?. Dibujar "B" en color rojo. Enseanza renovada 1980 Un agricultor vende un saco de patatas por 1000 pesetas. Los gastos de produccin se elevan a 800 pesetas y el beneficio es de 200 pesetas. Subraya la palabra "patata" y discute sobre ella con tu compaero. Enseanza renovada (LODE) Un lavriego vurgus, capitalista insolidario, sanriquecio con 200 pesetas al bender especulando un saco de patatas. Analiza el texto y deseguio di lo que piensas de este avuso antidemocrtico. Bachillerato de Adultos (Comienzo de los 90) Para la prxima convivencia necesitamos patatas por valor de 1000 pesetas. Investiga. Conclusiones. Realiza una puesta en comn de los resultados obtenidos dando respuestas razonadas, claras y concisas sobre: (A) las patatas; (B) la tortilla; (C) la convivencia. Enseanza comprensiva (LOGSE) Tras la entrada de Espaa en el Mercado Comn, los agricultores no pueden fijar libremente el precio de las patatas. Suponiendo que quieran vender un saco de patatas por 1000 pesetas haz una encuesta para poder determinar el volumen de la demanda potencial de patatas en nuestro pas y la opinin sobre la calidad de nuestras patatas en relacin con las importadas de otros pases, y cmo se vera afectado todo el proceso de venta si los sindicatos del campo convocan una huelga general. Completa esta actividad analizando los elementos del problema, relacionando los elementos entre si y buscando el principio de relacin de dichos elementos. Finalmente haz un cuadro de doble entrada, indicando en horizontal arriba, los nombres de los grupos citados y abajo, en vertical, diferentes formas de cocinar las

patatas

Cmo demostraran algunas personas si es cierto o falso si todos los nmeros impares son primos. Fsico: 3 es primo, 5 es primo, 7 es primo, y por induccin todos los nmeros impares son primos. Nota: al llegar al 9 se obtiene un error experimental. Ingeniero: 3 es primo, 5 es primo, 7 es primo, y por induccin todos los nmeros impares son primos. Fsico terico especializado en renormalizaciones: 3 es primo, 5 es primo, 7 es primo, 9/3 es primo, 11 es primo, 13 es primo, 15/3 es primo ... Programador: 1 es primo, 1 es primo, 1 es primo, 1 es primo ,... Fsico Cuntico:Todos los nmeros son iguales y primos y no primos hasta que son observados. Vendedor de Software: 1 es primo, 2 es primo, 3 es primo, 4 es primo, 5 es primo, 6 es primo, 7 es primo, ... Ms primos que nadie en el mercado. Catedrtico: 3 es primo, 5 es primo, 7 es primo, los dems quedan de ejercicio para los estudiantes. Programador profesional: 3 es primo, 5 es primo, 7 es primo, 9 ser primo en la prxima versin. Programador de Basic: Qu es un nmero primo?. Programador de Cobol: Qu es un nmero impar?. Programador de Windows: 1 es primo. Ha ocurrido un error. Apriete cualquier tecla para empezar. Filsofo: Vayamos por partes, Qu es un nmero?. Economista: Acabo de leer que 3 es primo, 5 es primo, 7 es primo, pero 9 no es primo. La produccin est bajando.

Tenemos que contratar ms nmeros primos. Cristiano Evangelista: Seguro que est en la Biblia. Telogo: Dios cre a todos los nmeros iguales, por lo tanto los impares y los primos son los mismos. Psiclogo: 1 es primo, 3 es primo, 5 es primo, 7 es primo, 9 es primo reprimido. Socilogo: Hala, ya estamos clasificando nmeros. Profesor de Bellas Artes: 2 es primo, 4 es primo, 6 es primo impar. Abogado: 1 es primo, 3 es primo, 5 es primo, 7 es primo ... despus de descontar el 10% de impuestos y mi tasa legal. Poltico: 3 es primo, 5 es primo, 7 es primo, 9 tiene todo el derecho a tener su opinin, pero esto es una democracia y tristemente ha perdido. Poltico Liberal: 3 es primo, 5 es primo, 7 es primo, 9 ... Hmmmm ... bueno, lo importante aqui es adoptar una actitud constructiva. Mara Gaetana Agnesi (1718-1799) Nacida en Miln fue una excelente matemtica, filsofa y ling|ista. Con 30 aos publica un libro, primer tomo, dedicado a la geometra y que tuvo una amplia difusin en Europa (traducido al ingls y francs): Intituzioni Analitiche ad uso della gioven italiana. En un segundo tomo de las Instituciones, la parte dedicada al clculo diferencial fu considerada como una de las mejores de la poca. La Academia de Pars coment en el anlisis de dicho tomo: "... consideramos este tratado como la obra ms completa y mejor escrita en el gnero". En el mismo hace un estudio de las curvas que pueden escribirse de la forma.

Veamos, brevemente, cmo se generan.

Sea una circunferencia C y QT un dimetro. Desde Q trazamos una recta r que corta a C en A y desde T una tangente t. Ambas rectas se cortan, tal como indica la figura, en m. Por m trazamos una paralela al dimetro QT y por A una perpendicular a dicho dimetro. La interseccin de ambas es el punto P. Deseamos saber qu grfica describe el punto P al desplazarse A por la circunferencia. (Puedes ver otra posicin de P considerando r, determinado A y efectuando la misma construccin).

Como los tringulos QmT y QAH son semejantes podemos establecer la proporcin Tm/HA = QT/QH de donde x/HA = a/y es decir x y = a HA (1) Aplicando el teorema de la altura al tringulo rectngulo QAT resulta HA 2 = y (a - y) (1). Despejando HA en (1) sustituyendo en (2) y resolviendo en y la expresin obtenida llegamos a

Para a = 1 en la expresin hallada tenemos

Una forma fcil (y eficaz) de obtener la grfica de esta funcin es analizar el comportamiento de la expresin que aparece en el denominador: p(x) = x 2 + 1. Una funcin cuya grfica es una parbola con las ramas hacia "arriba" y vrtice en (0,1).

Como p(x) = x 2 + 1 1. es positiva 2. es par (es decir p(x) = p(-x), simtrica respecto del eje OY) 3. es decreciente cuando x < 0

...entonces f(x) = 1 / p(x) 1. 2. 3. 4. ... es positiva ... es par ... es creciente cuando x < 0 ... es decreciente cuando x > 0

4. es creciente cuando x > 0 5. tiene un mnimo en (0, 1) 6. tiende a +Infinito cuando x tiende a +Infinito 7. tiende a +Infinito cuando x tiende a Infinito

5. ...tiene un mximo en (0, 1) 6. ... tiende a CERO (con valores positivos) cuando x tiende a +Infinito 7. ... tiende a CERO (con valores positivos) cuando x tiende a -Infinito De 6 y 7 podemos deducir que el eje de abscisas (y = 0) es asntota horizontal de f.

Como p(x) no se anula en ningn punto de su dominio (real), podemos garantizar que f no tiene asntotas verticales. Suficientes propiedades de f como para esbozar su grfica, que figura en azul en el dibujo anterior.

Una interesante propiedad (para los amigos de pi, que somos todos) de f. Como y = arc tag(x) es una primitiva de f resulta que al rea limitada por f y su asntota es ... PI. Mira, mira ...

Para comprobar la ltima igualdad slo es necesario tener en cuenta la grfica de la funcin y = arc tag(x) y que pi/2 y -pi/2 son sus asntotas. Cuando x "tiende" a +Infinito entonces la funcin arc tag(x) se "aproxima" a pi/2 y cuando x "tiende" a -Infinito entonces la funcin arc tag(x) se "aproxima" a -pi/2.

Hija de una familia acomodada, y muy numerosa, public a los 9 aos una traducci en latn en defensa de la educacin y formacin de las mujeres. A edad muy temprana dominaba el latn y, bastante bien, el griego y hebreo. Su padre ejerci una gran influencia sobre ella. Cuando, parece ser que afectada por la muerte de su madre, le solicit hacerse monja ste se neg. Ella, a cambio, le pidi tres condiciones: asitir a la iglesia siempre que quisiera, vestir de una forma sencilla y no asistir a fiestas profanas. Se dedic a estudiar libros religiosos y de matemticas. En 1738 publica una coleccin de ensayos filosficos: Propositiones Philosoficae. Su profesor, un tal Rampinelli, la anim a que escribiera los libros antes mencionados. Su posicin acomodada hizo que preparase la impresin en su propia casa y entr en contacto con Riccati para comentar su trabajo. El Papa Benedicto XIV influy para que se le concediera la ctedra de matemticas de Bolonia, pero no la acept. Muri a los 81 aos, en Miln y en la Biblioteca Ambrosiana de dicha ciudad se conserva, en 25 volmenes, su obra. ... y como vers nunca hemos hablado de brujas

Al-Khwarizmi El ms conocido de los matemticos rabes es Mohammed Ibn Musa Al-Khwarizmi (780-850), conodido como padre del lgebra. Se sabe poco de su vida salvo que vivi en la primera mitad del siglo IX y que trabaj en la biblioteca del califa de Bagdad. Escribi libros sobre geografa, astronoma y matemtica. En su obra Artimtica ("Algoritmi de numero indorum") explica con detalle el funcionamiento del sistema decimal y del cero que usaban en la India. Obra de gran importancia pues contribuy a la difusin del sistema de numeracin indio y al conocimiento del cero. Debe destacerse la obra de contenido algebrico "Hisab al-yabr wa'l muqqabala", considerada uno de los primeros libros de lgebra. Obra eminentemente didctica con abundantes problemas para resolver y adiestrar al lector, principalmente, en la resolucin de ecuaciones de segundo grado. Es el autor de uno de los mtodos ms antiguos que se conocen para resolver ecuaciones de segundo grado. Dicho mtodo, geomtrico, se conoce como de completar cuadrado

Resolucin de la ecuacin x 2 + 6x = 7

Comenzamos por construir el cuadrado de lado x, ABCD, cuya rea es x 2. A continuacin prolongamos los lados AB y AC en 3 unidades respectivamente (observa que obtenemos dos rectngulos de rea 6x, segundo trmino de la ecuacin). Completamos el cuadrado construyendo un nuevo cuadrado de superficie 9 u 2. El rea total del cuadrado es x 2 + 6x + 9. Puesto que deseamos resolver la ecuacin x 2 + 6x = 7, tendremos, sumando 9 a ambos miembros: x 2 + 6x + 9 = 7 + 9 = 16 es decir (x + 3) 2 = 4 2 por tanto x=1

Arqumedes de Siracusa Arqumedes de Siracusa matemtico, fsico e inventor griego, nace en Siracusa (285-212 a.J.C). Su padre, Fidias, posiblemente astrnomo, parece que influy en su vocacin y formacin. Estudi en la famosa escuela de Alejandra, posiblemente fuera alumno de Euclides, y regres a su ciudad natal donde dedic su vida a la investigacin. Las aportaciones de Arqumedes a las matemticas fueron de gran categora cientfica. Su mtodo fue fundamentalmente geomtrico, obteniendo conclusiones que no slo representaron un gran avance sobre la geometra, sino que tambin llevan al clculo integral. Fue el primer matemtico conocido del que se tienen nocicias que calcul el rea limitada por un segmento parablico en el intervalo [0,1], determinando la suma de las reas de los rectngulos incritos y circunscritos. En Geometra sus escritos ms importantes fueron:

De la Esfera y el Cilindro, donde introduce el concepto de concavidad, que Euclides no haba utilizado, asi como ciertos postulados referentes a la linea recta. De los Conoides y Esferoides en donde define las figuras engendradas por la rotacin de distintas secciones planas de un cono. De las Espirales en donde analiza estas importantes curvas y analiza sus elementos ms representativos.

En Aritmtica son, fundamentalmente dos los escritos ms interesantes:

El Arenario en el que expone un mtodo para escribir

nmeros muy largos dando a cada cifra un orden diferente segn su posicin. De la medida del Crculo una de sus obras fundamentales, donde demuestra que la razn entre la circunferencia y el dimetro est comprendida entra 3 10/7 y 3 1/7; dicha relacin es conocida en la actualidad por . Demuestra adems la equivalencia entre el rea del crculo y un tringulo rectngulo cuyos catetos son el radio y el permetro (longitud) de la circunferencia.

Arqumedes comunic a Eratstenes (bibliotecario de Alejandra) los razonamientos seguidos en las custiones geomtricas. Los mismos se recogen en una obra fundamental: El Mtodo. (Algo as, segn algunas investigaciones, como una comunicacin entre colegas al ms alto nivel).

Las aportaciones ms importantes de Arqumedes a la Fsica se centran en la mecnica de slidos y en la Hidrosttica, en las que se vale para sus demostraciones de figuras geomtricas. En la mecnica de slidos es la Esttica la parte que m mereci su atencin. En sus escritos trata sobre el equilibrio de los cuerpos geomtricos, as como la forma de determinar el centro de gravedad de cualquier Sello del correo postal cuerpo (en estos escrito habla del centro de gravedad de las figuras espaol dedicado a planas) Arqumedes (pintado por Tambin enuncia la ley fundamental de la palanca, la cual produjo Rivera) gran sensacin en el mundo griego (Dadme un punto de apoyo y mover el mundo). La polea compuesta, basada en el principio de la palanca, y que emple para mover un gran barco, para sorpresa del escptico rey Hiern, fue otrode sus sorprendentes descubrimientos. El historiador Plutarco nos cuenta "[...] que no poda ser deslizado del muelle a no ser que se emplease un gran esfuerzo y muchos hombres; y, tras cargarlo con numeroso pasaje y mercancas a tope, se sent a una cierta distancia y, sin gran esfuerzo, sino slo sosteniendo el cabezal de la polea en su mano y tirando de las cuerdas gradualmente, arrastr el barco en lnea recta, de forma suave y por igual como si se estuviera moviendo en el mar" Probablemente el descubrimiento ms conocido de Arqumedes sea la ley sobre la prdida que sufren los cuerpos sumergidos en un lquido. Arqumedes descubri con dicho principio que el rey Hiern haba sido objeto de una estafa al encargar una corona de oro. Cuenta la tradicin que descubri la solucin mientras se estaba baando y sali corriendo desnudo de su casa gritando (lo he descubierto!).

Arqumedes aplic parte de sus descubrimientos en la defensa de su ciudad natal contra el asedio de los romanos. Citando nuevamente a Plutarco, las legiones romanas avanzaron hacia las murallas creyndose invencibles "[...] pero cuando Arqumedes comenz a maniobrar con sus mquinas, inmediatamente lanz contra las fuerzas terrestres toda clase de armas arrojadizas y unas masas inmensas de piedras que caan con un ruido y violencia terribles; contra las cuales ninguno pudo resistir, ya que abatan a cuantos les caan a montones, rompiendo toda formacin." Cuenta la tradicin, aunque no parece muy probable, que mediante unos espejos incendi la flota romana desde el interior de las murallas de Siracusa; parece ser que el artilugio consista en un conjunto de espejos planos con los que concentraba los rayos del sol sobre las velas de las embarcaciones.

Pero a pesar de todas sus invenciones y grandes armas, la verdadera pasin de Arqumedes fueron las matemticas puras "[...] sus palancas, poleas y catapultas fueron naderas en comparacin con los bellos teoremas que descubri". Nuevamente citamos a Plutarco: "Arqumedes posea un espritu tan elevado, un alma tan profunda y con tales tesoros de conocimientos cientficos que, aunque estos inventos le han trado hasta ahora el renombre de una gran sagacidad sobrehumana, no se ha dignado dejarnos ningn comentario o escrito sobre estas materias; sino que repudiando como srdido e innoble el mundo de la ingeniera y toda clase de tcnica que slo sirve para mero uso y provecho, situ sus afectos y ambiciones en aquellas especulaciones ms puras en las que no puede caber ninguna referencia a las vulgares necesidades de la vida" Su mximo legado fueron las matemticas y, en ese terreno, permanece como el ms grande de la antigedad. Sus resultados, que sobreviven en una docena de libros y fragmentos, tienen una calidad y un refinamiento lgico verdaderamente sorprendentes. Cuando Siracusa fue capturada por los soldados de Marcelo un destacamento entr en la casa de Arqumedes que se encontraba absorto en sus trabajos y le dio muerte. Plutarco nos relata, por ltimo, el epitafio que pidi a sus amigos que figurara sobre su tumba: "[...] sus descubrimientos fueron numerosos y admirables; pero se cuenta que le pidi a sus amigos y parientes que, cuando muriera, colocaran sobre su tumba una esfera dentro de un cilindro, incribindola en la proporcin del slido continente respecto al contenido, esto es, la razn 3:2"

Cuenta la tradicin que Arqumedes indic que sobre su tumba se esculpiera un cilindro y en l una esfera inscrita. La relacin entre los volmenes de ambos cuerpos es

V Cilindro = 3/2 V Esfera

Pare llegar a dicho resultado, Arqumedes compar una semiesfera con un cilindro y un cono recto de bases un crculo mximo de la semiesfera. Obtuvo sobre dichos cuerpos tres secciones al cortar por un plano paralelo a las bases y compar las reas obtenidas. Superficie Seccin Semiesfera S 1 = pi r 12 = pi (R 2 - x 2) = pi R 2 - pi x 2 Superficie Seccin Cilindro S 2 = pi r 22 = pi R 2 Superficie Seccin Cono S 3 = pi r 32 = pi x 2 (pues x = r 3) Es decir, que para una seccin dada se establece la proporcin S1 = S2 - S3 por lo que V Semiesfera = V Clindro - V Cono = 2/3 pi R 3 Como el volumen del cilindro circunscrito a la esfera de radio R es 2 pi R 3 resulta V Esfera = 2/3 VCilindro circunscrito
Anverso y reverso de una moneda siciliana acuada en honor de Arqumedes. En el reverso puede verse una esfera sobre un soporte. AR MD son las abreviaturas del nombre latinizado ("Archimedes")

Euclides de Alejandra Sobre la vida de Euclides slo sabemos dos cosas ciertas: que fue contemporneo de Tolomeo Ster (367-283 a.d.C), mayor que Arqumedes (nacido hacia el 287 a.d.C) y que ense en Alejandra. De su obra, se sabe que escribi ms de diez libros, slo han llegado dos a nuestros das: LOS ELEMENTOS y LOS DATOS. No es exageracin afirmar que los Elementos ha sido el libro ms utilizado de la historia. Adems fue uno de los primeros libros impresos. Una primera impresisn aparece en Venecia. Hacia el siglo IV a.d.C. Alejandro de Macedonia conquista el mundo griego, muriendo en el 323 a.d.C. a lo 33 aos. Dos aos antes fund la ciudad de Alejandra en el delta del Nilo. Esta ciudad fue punto de encuentro de griegos, judos En Espaa, la primera versin y rabes. En ella se conserv lo mejor del pensamiento se realiza en Sevilla en 1576, heleno. A su universidad, se dice que con ms de 700.000 con el ttulo documentos, fue llamado Euclides por Ptolomeo I Ster (el "Los seis libros primeros de Grande), sucesor de Alejandro, y durante ms de 20 aos, la Geometra de Euclides, ejerci la labor docente y cientfica. En dicha ciudad, Traduzidos por Rodrigo Euclides escribi su obra cumbre Los Elementos. En trece gamorano Astrologo y libros recopila casi todo el saber matemtico de su poca; Mathematico, y realiza una tarea gigantesca axiomatizando dichos Cathedratico de conocimientos. Cosmografia por su Magestad en la casa de Sin lugar a duda, Euclides puede ser considerado el creador Contratacio de Seuilla, del mtodo axiomtico y sus Elementos el libro que ms 1576" influencia ha ejercido en las matemticas.

Los Postulados de Euclides Al comienzo de cada uno de los libros que componen los Elementos, Euclides presenta una definiciones y unas nociones comunes relativas a los temas desarrollados. En el Libro I expone adems los cinco postulados en los que basa la construccin axiomtica. 1. 2. 3. 4. 5. Postlese el trazar una recta desde un punto cualquiera hasta un punto cualquiera Y el prolongar continuamente una recta finita en linea recta. Y el describir cualquier crculo con cualquier centro y distancia. Y el ser todos los ngulos rectos iguales entre s. Y que si una recta al incidir sobre dos rectas hace los ngulos internos del mismo menores que dos rectos, las dos rectas prolongadas indefinidamente se encontrarn en el lado en el que estn los (ngulos) menores que dos rectos. Algunas proposiciones equivalentes al postulado de las paralelas (postulado 5) son: Playfair Por un punto exterior a una recta se puede trazar una paralela y slo una. Proclo Dos rectas parlelas estn entre si a una distancia finita. Legendre Existe un tringulo en el cual la suma de sus tres ngulos vale dos rectos.

Saccheri y Laplace Existen dos tringulos no congruentes, con los ngulos de uno respectivamente iguales a los del otro. Legendre y Lorentz Por un punto cualquiera interior a un ngulo menor que dos tercios de rectos pasa una recta que corta a ambos lados del ngulo. Gauss Si k es un entero cualquiera, siempre existe un tringulo cuya rea es mayor que k. Bolyai Por tres puntos no alineados pasa siempre una circunferencia. etc...

Los Elementos renen y sistematizan casi todo el conocimiento matemtico de su poca. A grandes rasgos la estructura de Los Elementos es:

Libro I Teoremas relativos a congruencias, rectas paralelas. 23 definiciones; 5 postulados; 9 nociones comunes; 48 proposiciones (las p.47 y 48 son el teorema de Pitgoras) Libro II Aritmtica de la Escuela Pitagrica. 2 definiciones; 14 proposiciones. Libro III Cculos, cuerdas, .... 11 definiciones; 37 proposiciones. Opus elementorum euclidis ... Libro IV Construcciones con regla y comps. Venecia 1482. 7 definiciones; 16 proposiciones. Pgina inicial de la primera edicin de Libro V Teora de la proporcin. 18 Los Elementos debida al impresor definiciones; 25 proposiciones. Erhard Ratdolt. Libro VI Estudio de figuras semejantes. 4 Para leer: definiciones; 33 proposiciones. Euclides. Biblioteca Clsica Libro VII Teora de nmeros; 22 Gredos. definiciones; 39 proposiciones. (la p.I es el algoritmo de Euclides). Tomo I Libros I-IV y una Libro VIII Teora de nmeros; 27 introduccin monumental de proposiciones. Luis Vega (180 pginas) Libro IX Teora de nmeros; 36 Tomo II Libros V-IX proposiciones; (p.XX "el conjunto de Tomo III Libros X - XIII nmeros primos es infinito"). Libro X Magnitudes; 36 proposiciones; (Se establece el mtodo de exhaucin). Libro XI Geometra de slidos y esfera; 39 proposiciones. Libro XII Geometra de slidos y esfera; 18 proposiciones. Libro XIII Geometra de slidos y esfera; 18 proposiciones.

En la red: http://aleph0.clarku.edu/~djoyce/home.html La pgina de David E. Joyce (En ingls) Leonardo de Pisa. Fibonacci
"En un patio cerrado, se coloca una pareja de conejos para ver cuntos descendientes produce en el curso de un ao, y se supone que cada mes a partir del segundo mes de su vida, cada pareja de conejos da origen a una nueva. Como la primera pareja de conejos tiene descendencia en el primer mes, dobla el nmero y, en este mes, se tienen dos parejas. De stas, una pareja, la primera, tambin tiene descendencia en el mes siguiente, de manera que en el segundo mes hay tres parejas. De sas, dos parejas tienen descendencia en el mes siguiente, de modo que en el tercer mes han nacido dos parejas adicionales de conejos, y el nmero total de parejas de conejos llega a cinco. En dicho mes tres de estas cinco parejas tienen hijos y, en el cuarto, el nmero de parejas llega a 8. Cinco de estas parejas producen otras cinco parejas, las cuales, junto con las 8 parejas ya existentes, hacen 13 parejas en el quinto mes. Cinco de estas parejas no tienen hijos en este mes en este mes, mientras que las restantes ocho parejas tienen descendencia, de modo que en el sexto mes se tienen 21 parejas. Simando a stas las 13 parejas que nacen en el sptimo mes, se obtiene un total de 34 parejas. Sumando a stas las las 21 parejas que nacen en el octavo mes, el total es de 55 parejas. Sumando a stas las 34 parejas que nacen en el noveno mes, se obtienen 89 parejas. Agregando a stas las 55 parejas que nacen en el dcimo mes, se tiene un total de 144 parejas. Agregando a stas las 89 parejas que nacen en el undcimo mes, se llega a un total de 233 parejas.
Parejas: primer mes segundo mes tercer mes cuarto mes quinto mes sexto mes sptimo mes octavo mes

Leonardo de Pisa (conocido como Fibonacci, contraccin de filius Bonacci, es decir el hijo de Bonacci) nace en Pisa, posiblemente hacia 1170 y muere sobre 1250. Al ser su padre representante comercial de la ciudad de Pisa en Argelia, estuvo en contacto con la cultura rabe, interesndose especialmente por sus matemticas. Su obra principal fue el Liber Abaci (o Libro acerca del baco), una extensa obra que contiene casi todo el conocimiento algebraico y aritmtico de la poca. En ella Fibonacci expona entre otras cosas, la importancia del sistema de numeracin indoarbigo. Escrito en 1202, slo se conserva la versin de 1228 (segunda versin). En l aparece (pgs. 123 y 124) un problema sobre el nacimiento de conejos y que nada tuvo de significativo hasta que, a comienzos del siglo pasado, fue objeto de numerosos estudios que permitieron descubrir muchas de las propiedades que tiene. Aunque anteriormente Kepler (De Nive Sexangula) ya haba relacionado la sucesin de Fibonacci con la seccin urea y el crecimiento de plantas. En honor de Fibonacci, la sucesin definida por f1 = f2 = 1 fn = fn - 1 + fn - 2 para n >= 3 recibe el nombre de sucesin de Fibonacci y sus trmino nmeros de Fibonacci. Los primeros tminos de la sucesin de Fibonacci son:

1 2 3 5 8 13 21 34 55

f1 = 1 f2 = 1 f3 = f2 + f1 = 2 f4 = f3 + f2 = 3

Finalmente, sumando a stas 144 parejas que dcimo mes nacen en el ltimo mes, undcimo mes se obtienen un total de 377 parejas. Este es el duodcimo mes nmero de parejas producidas por la primera pareja en el lugar dado, al trmino de un ao. Al examinar la tabla anterior, el lector puede ver cmo se llega a este resultado; a saber: se suma el primer nmero al segundo, o sea, 1 a 2; el segundo al tercero; el tercero al cuarto, el cuarto al quinto; y as sucesivamente, hasta que se suman el dcimo y el undcimo nmeros 144 y 233; as se obtiene el nmero total de parejas de los conejos en cuestin, es decir, 377."
noveno mes

89 144 233 377

f5 = f4 + f3 = 5 f6 = f5 + f4 = 8 f 7 = f 6 + f 5 = 13 ...

Es decir: 1, 1, 2, 3, 5, 8, 13, 21, 34, 55, 89, 144, 233, 377, 610, 987, 1597, 2584, ... En ella f 14 = 377 es el resultado buscado por Fibonacci.

Otras sucesiones interesantes. Nmeros de Lucas. En honor del matemtico francs F. Edouard A. Lucas (1842-1891) La sucesin definida por:

L 1 = 1, L 2 = 3 L n+2 = L n+1 + L n n >= 1 1, 3, 4, 7, 11, 18, 29, 47, ... Sucesiones generalizadas de Fibonacci. se obtienen con el mismo mtodo de recurrencia pero los dos primeros trminos son dos nmeros naturales cualesquiera. Si f 1 = 1 y f 2 = 5, obtenemos la sucesin generalizada de Fibonacci siguiente:

Otras obras de Fibonacci fueron: Practica Geometriae (1120) Liber quadratorum (1225) Flos (1225) La sucesin de Fibonacci presenta numerosas propiedades que la han hecho particularmente atractiva. Existe una publicacin denominada The Fibonacci Quarterly, publicada por la Fibonacci Association en la que, a partir de los aos 60, se recogen y estudian mltiples propiedades de esta sucesin y las derivadas de ella. En lo que sigue veremos algunas de estas propiedades

1, 5, 6, 11, 17, 28, 45, 73, 118,... Nmeros de Tribonacci Sucesin obtenida a partir de los nmeros

T 1 = 1; T 2 = 1; T 3 = 2 y sumando de tres en tres

T4 = T3 + T2 + T1 = 4 T5 = T4 + T3 + T2 = 7 T 6 = T 5 + T 4 + T 3 = 13 T 7 = T 6 + T 5 + T 4 = 24 T 8 = T 7 + T 6 + T 5 = 44 ...

Suma de n trminos f 1 + f 2 + f 3 + f 4 + ... + f n = f n + 2 - 1 Suma de trminos impares f 1 + f 3 + f 5 + f 7 + ... + f 2n - 1 = f 2n Suma de trminos pares f 2 + f 4 + f 6 + f 8 + ... + f 2n = f 2n + 1 - 1 Suma de los cuadrados de n trminos f 12 + f 22 + f 32 + f 42 + ... + f n2 = f n f n + 1 Diferencia de cuadrados La diferencia de cuadrados de dos nmeros de Fibonacci cuyos ndices difieren en dos unidades es otro nmero de Fibonacci f n + 12 - f n - 12 = f 2n Relacin de la sucesin de Fibonacci con los coeficientes binomiales

Dispuesto el Tringulo de Pascal tal como indica la figura y sumando las diagonales en el orden indicado (diagonales colororeadas) obtenemos los nmeros de Fibonacci. Si , los nmeros de Fibonacci tienen la siguiente expresin: La divisibilidad y los nmeros de Fibonacci.

f 1 = C0, 0 f 2 = C 1, 0 f 3 = C 2, 0 + C 1, 1 f 4 = C 3, 0 + C 2, 1 f 5 = C 4, 0 + C 3, 1 + C 2, 2 f 6 = C 5, 0 + C 4, 1 + C 3, 2 f 7 = C 6, 0 + C 5, 1 + C 4, 2 + C 3, 3 f 8 = C 7, 0 + C 6, 1 + C 5, 2 + C 4, 3 ...

Nmeros de Fibonacci consecutivos son primos entre si Si designamos por (a,b) el mximo comn divisor de a y b, entonces (f m , f n) = f (m, n)

Ejemplo: Si f 8 = 21 y f 12 = 144, entonces m = 8, n = 12, por lo que (m, n) = (8, 12) = 4 y (f 8 , f 12) = (21, 144) = 3 = f 4

Si n es divisible entre m, entonces f n es divisible entre f m Ejemplo: f 10 = 55; f 5 = 5 entonces f 10 / f 5 = 55 / 5 = 11 f n es par si y slo si n es mltiplo de 3. f 3 = 2; f 6 = 8; f 9 = 34; f 12 = 144...

La sucesin de Fibonacci y el nmero ureo.

La sucesin formada por los cocientes de nmeros de Fibonacci consecutivos converge, rpidamente, hacia el nmero ureo. Diferencia en valor absoluto con phi f2 / f1 = 1 0, 61 80 33 98 ... f3 / f2 = 2 / 1 = 2 0, 38 19 66 01 ... f 4 / f 3 = 3 / 2 = 1, 5 0, 11 80 33 98 ... f 5 / f 4 = 5 / 3 = 1, 66 66 66 66 ... 0, 04 86 32 67 ... f 6 / f 5 = 8 / 5 = 1, 6 0, 01 80 33 98 ... f 7 / f 6 = 13 / 8 = 1, 62 5 0, 00 69 66 01 ... f 8 / f 7 = 21 / 13 = 1, 61 53 84 61 ... 0, 00 26 49 37 ... f 9 / f 8 = 34 / 21 = 1, 61 90 47 76 ... 0, 00 10 13 63 ... f 10 / f 9 = 55 / 34 = 1, 61 76 47 05 ... 0, 00 03 86 92 ... ...

Es decir

Como f n = f n - 2 + f n - 1 resulta

Ahora bien Anlogamente

por lo que

Reiterando este procedimiento llegamos a obtener

Puesto que el nmero ureo tiene el mismo desarrollo en forma continua queda justificada la convergencia indicada.

Otras convergencias de los nmeros de Fibonacci

La sucesin f 1 / f 3, f 2 / f 4, f 3 / f 5, f 4 / f 6, ... f n / f n + 2 converge a La sucesin f 1 / f 2, f 2 / f 3, f 3 / f 4, f 4 / f 5, ... f n / f n + 1 converge a El nmero de Fibonacci f m es el entero ms prximo al tmino a m de la progresin geomtrica cuyo primer trmino es Ejemplo y razn

es decir f 14 = Parte Entera (a 14 ) = 377

Frmula de Binet Podemos obtener un nmero de Fibonacci mediante la expresin

expresin conocida por frmula de Binet como recuerdo de Jacques Philippe Marie Binet (1786-1856), matemtico que la descubri

Espiral de Fibonacci

En la red:

www-history.mcs.st-and.ac.uk./~history/Mathematicians/Fibonacci.html

www.ee.surrey.ac.uk/Personal/R.Knott/Fibonacci/fib.html Adems La Bolsa y la sucesin de Fibonacci.. Francisco Gallego Puche

Hern de Alejandra Entre los muchos "Hern" que existen en la historia de las ciencias tcnico-matemticas unos de los ms importantes fue el de Alejandra (que por cierto parece ser que tampoco naci all sino en Ascra). Si tiene ms fundamento el que era de origen humilde y fu, en su juventud, zapatero. Tampoco existen datos dignos de crdito respecto a su nacimiento (?126 a.C.) ni a su muerte (?50 a.C.). Fu el inventor de mquinas como la dioptra, el odmetro (sistema de engranajes combinados para contar las vueltas de una rueda) o, quizs el ms importante, la eolipila, un precursor de la turbina de vapor. Su obra, si es la de un solo autor, fu bastante amplia. (Marcaremos con (+) las que han llegado a nososotros)

Obras de carcter cientfico: o (+) Mtrica. Fragmentos dispersos en una veintena de manuscritos y algunos de origen dudoso, tiene una finalidad eminentemente prctica. Estuvo perdida hasta que fu descubierto, en 1896, un manuscrito de 1100. Libro I. Estudio de reas, cuadriltero, polgonos regulares, figuras circulares, elipse,...) Libro II. Dedicado al estudio de volmenes siguiendo una estructura parecida al Libro I. Libro III. Dedicado a la divisin de figuras en partes proporcionales. o Escolios de Euclides (citados por Proclo) o (+) Mecnica. Libro I. Se ocupa de las proporciones de figuras. Libro II. Trara de las mquinas simples (torno, palanca, polispasto, cua y tornillo). Libro III. Tratado de aplicaciones de la mecnica. Tcnicas: o (+) Neumticas. Ms conocidas por su nombre latino 'Pneumaticorum libri duo'. En el prefacio se trata el concepto de vaco de forma cientfica por primera vez. o Catptrica. que trata de los espejos planos, cncavos y convexos. (Esta obra fu atribuda durante bastante tiempo a Ptolomeo). o (+) Dioptra, donde trata el uso de este aparato que fu utilizado durante bastante tiempo en observaciones astronmicas. Mecnica aplicada: o Relojes hidrulicos (mencionados por Pappus). Slo

o o o o o

se conserva un fragmento en el que habla de la clepsidra (+) Mquinas de guerra (+) Quirobalista (+) Autmatas Los equilibrios Sobre los vasos hidrulicos

El camino ms corto Euclides en su ptica, enunci que la luz atraviesa el espacio en linea recta y en la Catptrica enuncia la ley de la reflexin. Cuatro siglos despus, Hern observ que dicha ley es consecuencia del hecho de que la luz debe tomar siempre el camino ms corto. En Dioptra. prop. 4. demuestra el teorema. Un ejemplo nos har ver mejor qu quera decir Hern. Supongamos un hombre que se encuentra en P y desea ir hasta Q, pero antes desea llenar un cubo de agua en r. Cul es el camino ms corto? Para averiguarlo determinamos P' el simtrico de P respecto de r; sea M el punto de interseccin con r de la recta P'Q y M'cualquier otro punto de r. Los ngulos x, y, z son iguales (luego P'M = MP y P'M'= M'P); como la suma de dos lados de un tringulo es mayor que el tercero resulta P'Q < P'M'+ M'Q y puesto que P'Q = P'M + MQ resulta PM + MQ < PM'+ M'Q La trayectoria que debe recorrer una pelota de billar (blanca) para dar en la roja, despus de rebotar en una banda es la indicada en amarillo. Si tira a cualquier otro punto de dicha banda no har carambola. La trayectoria de la bola blanca, rebotando primero en la banda B y posteriormente en la B' es la indicada en color amarillo. Cul sera la trayectoria rebotando primero en B' y luego en B? Y a tres bandas?

En la Mtrica, existen ejemplos sobre el clculo de races cuadradas, mtodo que posiblemente ya conocan los babilonios y tambin Arqumedes; tambin calcula la raz cbica de 100. Para calcular las races cuadradas, utiliza la

Clculo de la raz cbica de 100 1. Considera los cubos anterior y posterior a 100, esto es 64 y 125, luego la raz cbica buscada estar entre las de estas cantidades.

siguiente regla: si deseamos calcular (N) 1/2 y p es una aproximacin, entonces es una mejor aproximacin. No ocurre igual con el clculo de la raz cbica de 100 y deducir una regla es ms difcil (No?)

2. Determina las diferencias 125 - 100 = 25 (= 5 ) 100 - 64 = 36 3. Multiplica 36 x 5 = 180 y lo aade al nmero propuesto, es decir 100 + 180 = 280 4. Divide 180 entre 280 y obtiene 9/14 5. Aade este valor a la raz cbica de 64, es decir a 4, y obtiene 65/14 De esta forma determina la raz cbica de 100 con una aproximacin menor que 0,02.
2

Posiblemente la expresin matemtica ms conocida de Hern sea su frmula para determinar el rea de un trigulo conocidos sus lados. Algo realmente til en aquellos tiempos. Si bien parece que era conocida por Arqumedes, la primera demostracin que nos ha llegado figura en la Mtrica. El teorema nos garantiza, conociendo las lados de un tringulo, conocer su rea, mediante la expresin donde a, b y c son los lados del tringulo y p la mitad del permetro del mismo. Aunque ya conocemos una demostracin de este teorema en Calculando el rea de un tringulo, seguiremos ahora, con la notacin actual, el camino que siguin Hern para llegar a dicha expresin. Verdadero encaje de bolillo.! En primer lugar inscribi un crculo en el tringulo y dedujo que el rea del mismo era A = r.p (siendo r el radio del crculo y p la mitad del permetro del tringulo). Como la demostracin es, esencialmente, la misma que figura en Calculando el rea de un tringulo, la omitimos. A continuacin estudia los tringulos que se forman y llega a la conclusin de que son congruentes las siguientes parejas de tringulos: AOM y AOP, BON y CON, AOP y COP por lo que resulta: AM = AP, BM = BN, CP = CN y adems ngulo (AOM) = ngulo (AOP) ngulo (BOM) = ngulo (BON) ngulo (COP) = ngulo (CON)

Seguidamente prolong la base AB hasta C' de forma que AC'= PC (= CN) y argument BC' = BM + MA + AC'= BM + MA + CN = = 1/2 ( 2 BM + 2 MA + 2 CN) = = 1/2 ( (BM + AM) + (AM + AP) + (CN + CP) ) = Est clara la intencin de Hern al intentar = 1/2 ( (BM + AM) + (BN + NC) + (AP + tener sobre la recta base del tringulo la PC) ) = longitud del semipermetro. No slo eso, sino = 1/2 (a + b + c ) = p (semipermetro) que sobre la misma base determina p - a, p - b y p - c, con lo cul tiene sobre dicho segmento p - c = (C'A + AM + MB) - (AM + MB) = todos los elementos que intervienen en la C'A frmula. Genial! ... pero an falta mucho ... p - b = (C'A + AM + MB) - (CP + PA) = (C'A + AM + MB) - (C'A + AM) = MB p - a = (C'A + AM + MB) - (CN + NB) = (C'A + AM + MB) - (C'A + MB) = AM En este momento de la demostracin, Hern traza una perpendicular a la base por A y otra al segmento OB (por O). Ambas se cortan en T y une dicho punto con B. Obtiene de esta forma un cuadriltero TAOB tal que sus ngulos opuestos suman dos rectos (Euclides III.22: " Los ngulos opuestos de los cuadrilteros en los crculos son iguales a dos rectos". El cuadriltero dado se puede inscribir en un crculo por ser TO perpendicular a OL y AT perpendicular a AB. Cul es el centro de dicho crculo?); es decir ATB + AOB = 180. Como 2x + 2y + 2z = 360 resulta que x + y (Reconozco que cuando llegu aqu la primera + z = 180; puesto que y + z = AOB resulta vez que segu esta demostracin estaba que x + AOB = 180 = ATB + AOB y perdido.) concluye que ATB = x.

A continuacin Hern comienza a comparar parejas de tringulos semejantes

Son semejantes los tringulos POC y ATB. ( Por qu?) AB/AT = PC/r y como PC = C'A resulta AB/AC' = AT/r

Tambin son semejantes los tringulos KAT y KMO ( Por qu?) AT/AK = OM/KM = r/KM, por lo que AT/r = AK/KM

Teniendo en cuenta las dos proporciones anteriores resulta AB/AC' = AK/KM; sumando 1 a cada miembro de esta igualdad: AB/AC'+ 1 = AK/KM + 1 (AB + AC')/AC' = (AK + KM)/KM C'B/AC' = AM/KM, expresin equivalente a (C'B.C'B)/(AC'.C'B) = (AM.MB)/(KM.MB) o bien C'B 2.KM.MB = AC'.C'B.AM.MB A continuacin Hern considera el tringulo BOK, que es un tringulo rectngulo de altura r, que es precisamente la altura relativa a la hipotenusa; aplicando el teorema de la altura ("En todo tringulo rectngulo la altura relativa a la hipotenusa es media proporcional entre los dos segmentos que sobre la misma determina)", establece que r 2 = KM.MB. Sustituyendo en la expresin anterior tenemos: C'B 2.r 2 = AC'.C'B.AM.MB que son cada uno de los segmentos determinados sobre la base del tringulo; sustituyendo y manipulando las expresiones: C'B 2.r 2 = p.(p - a).(p - b). (p - c) (C'B.r) 2 = p.(p - a).(p - b). (p - c) Pero C'B.r = p.r = rea, segn se deduce al comienzo de la demostracin (Pareca que no servira para nada!), por lo que: rea 2 = p.(p - a).(p - b). (p - c) Q.E.D. Quod Erat Demonstrandum. Padro Puig Adam. Pedro Puig Adam (1900/1960) fue uno de los matemticos espaoles que mas trabajaron en la didctica de las matemticas. En cualquier pais europeo hubiese sido un lujo. En el nuestro, que tambin es europeo, con escasa tradicin cientfica y muy orgullosos de aquello de "...que inventen ellos", fue, salvo entre los cculos profesionales, un desconocido. Catedrtico del Instituto San Isidro de Madrid y de Metodologa de las Matemticas en aquella universidad, compaginaba su contacto real con la enseanza, con sus inquietudes pedaggicas influyendo en los nuevos profesores.

Su preocupacin por los problemas de la enseanza lo llev a ser un destacado miembro de la C.I.E.M. (Comisin Internacional para la Enseanza de las Matemeticas), logrando que la XI C.I.E.M. se celebrase en Madrid en 1958. En 1958 redact el Declogo del Profesor de Matemticas en el que recoga sus opiniones sobre la enseanza de las matemticas en los Institutos de Bachillerato. El Declogo, siempre en vigor, nos muestra cmo los actuales pontfices didcticos no nos descubren nada nuevo. Sirva esta pgina, que se ir actualizando, como homenaje a la ingente labor de un matemtico espaol de primera lnea.

Estructuras algebraicas en un juego de mosaico


Artculo de Puig Adam publicado en la revista belga Mathematica & Pedagogica, n. 10. 1956 Se encuentra tambin en el libro del autor La Matemtica y su Enseanza Actual. Publicaciones de la Direccin General de Enseanzas Medias. Ministerio de Educacin Nacional 1959.

Material: Una o dos cajas de mosaicos de colores, con piezas de dos clases; tringulos rectngulos issceles iguales entre s y rombos con ngulos agudos de 450 y lados iguales a los catetos de los tringulos. Estos mosaicos de juguete se venden en los bazares con el nombre de Rombo La inconmesurabilidad de las reas de estas piezas me sugiri una leccin activa sobre irracionales cuadrticos y su clculo, que conduje del siguiente modo:

Empec distribuyendo entre los alumnos (3 y 4 de Bachillerato) piezas de las dos clases, y preguntndoles los valores de sus ngulos, el de la hipotenusa del tringulo (tomando el cateto como unidad); y el del rea de una y otra pieza. No es raro que la del rombo ofrezca alguna pequea dificultad. Se puede ayudar a resolverla componiendo la figura adjunta o dejando simplemente que averigen la altura del rombo por aplicacin del teorema de Pitgoras, Resultados: rea del tringulo, 1/2; rea del rombo /2. Los escribo en el encerado y propongo la siguiente cuestin: Con estas piezas del mosaico se pueden construir multitud de figuras diversas. Si formamos, aparte, figuras slo con tringulos y figuras que solamente contengan rombos, ser alguna de las primeras equivalente a alguna de las segundas?. De otro modo: Se puede sustituir un nmero de tringulos por un nmero de rombos de modo que las reas sustitudas sean equivalentes?

Los alumnos con los que oper ya saban por aritmtica la inconmensurabilidad de Tuve, sin embargo, que recordar su significado: Imposibilidad de que

= m / n o, de otro

modo, imposibilidad de que n sea igual a m unidades (m, n enteros). Con este recuerdo, consegu ya que, algunos de los alumnos, vieran la impossibilidad anloga de que un cierto nmero de veces /2, rea del rombo, equivalga a otro cierto nmero de veces el rea del tringulo 1/2. Recalqu, diciendo: Las reas de los tringulos y las de los rombos son como dos mundos aparte no intercambiables. toda figura compuesta de tringulos y de rombos tendr un rea con una parte racional procedente solamente de los tringulos que contiene y una parte irracional, procedente de los rombos. Despus de llegar a esta consecuencia, abro ligeramente una de las cajas cuadradas del juego con objeto de dejarles ver solamente un borde. En l ven la constitucin de uno de los lados del cuadrado, cuya longitud consta de cuatro lados de rombos (1) y dos hipotenusas de tringulos e inmediatamente les propongo averiguar cuntas piezas de cada clase contiene la caja, es decir, hay en el cuadrado.

Tras breve reflexin calculan el cuadrado de 4 + 2 (4 + 2 ) 2 = 24 + 16 de lo que resulta que la caja contiene 48 tringulos y 32 rombos.

Repito la cuestin para cajas de distinto tamao y aun para rectngulos que los mismos alumnos pueden idear. Por ejemplo, el rectngulo de dimensiones 3 + 2 y1+ exige 14 tringulos y 10 rombos como se obtiene fcilmente calculando el producto. El clculo previo del nmero de piezas necesarias de una y otra clase facilita mucho la construccin efectiva, con lo que puede terminar en forma de juego instructivo la leccin.
El autor contina el artculo con la justificacin y generalizacin del juego. Si ests interesado en l puedes solicitarlo a la G.M. y te lo remitiremos por e-mail en el plazo ms breve posible

Aqu puedes conseguir el Declogo Decalogo.Zip (1 K.) Existe una Sociedad Puig Adam de Profesores de Matemticas con pgina en la red a la que puedes ir desde nuestra pgina dedicada a las Asociaciones de Profesores de Matemticas Declogo de la didctica matemtica media. (Pedro Puig Adam) 1. No adoptar una didctica rgida, sino amoldarla en cada caso al alumno, observndole constantemente. 2. No olvidar el origen concreto de la Matemtica, ni los procesos histricos de su evolucin. 3. Presentar la Matemtica como una unidad en relacin con la vida natural y social. 4. Graduar cuidadosamente los planos de abstraccin. 5. Ensear guiando la actividad creadora y descubridora del alumno, 6. Estimular la actividad creadora, despertando el inters directo y funcional hacia el objeto de conocimiento. 7. Promover en todo lo posible la autocorreccin. 8. Conseguir cierta maestra en las soluciones antes de automatizarlas. 9. Cuidar que la expresin del alumno sea traduccin fiel de su pensamiento. 10. Procurar que todo alumno tenga xito para evitar su desaliento.

También podría gustarte